Sunteți pe pagina 1din 186

Revision Through MCQs (RTM) Compilation (November 2019)

INSIGHTSIAS
IA SIMPLIFYING IAS EXAM PREPARATION

RTM COMPILATIONS
PRELIMS 2020

NOVEMBER 2019

www.insightsactivelearn.com | www.insightsonindia.com
Revision Through MCQs (RTM) Compilation (November 2019)
Revision Through MCQs (RTM) Compilation (November 2019)
Revision Through MCQs (RTM) Compilation (November 2019)

Table of Contents
RTM-QUESTIONS .............................................................................................................. 6
RTM- REVISION THROUGH MCQS - 1th -Nov-2019 ........................................................... 42
RTM- REVISION THROUGH MCQS - 2nd -Nov-2019 ........................................................... 47
RTM- REVISION THROUGH MCQS - 4th -Nov-2019 ............................................................ 52
RTM- REVISION THROUGH MCQS - 5th -Nov-2019 ............................................................ 57
RTM- REVISION THROUGH MCQS - 6th -Nov-2019 ............................................................ 63
RTM- REVISION THROUGH MCQS - 7th -Nov-2019 ............................................................ 68
RTM- REVISION THROUGH MCQS - 8th -Nov-2019 ............................................................ 74
RTM- REVISION THROUGH MCQS - 9th -Nov-2019 ............................................................ 80
RTM- REVISION THROUGH MCQS -10th -Nov-2019 ........................................................... 86
RTM- REVISION THROUGH MCQS - 12th -Nov-2019 .......................................................... 92
RTM- REVISION THROUGH MCQS - 13th -Nov-2019 .......................................................... 98
RTM- REVISION THROUGH MCQS - 14th -Nov-2019 ........................................................ 102
RTM- REVISION THROUGH MCQS - 15th -Nov-2019 ........................................................ 107
RTM- REVISION THROUGH MCQS - 16th -Nov-2019 ........................................................ 113
RTM- REVISION THROUGH MCQS - 18th -Nov-2019 ........................................................ 118
RTM- REVISION THROUGH MCQS - 19th -Nov-2019 ........................................................ 122
RTM- REVISION THROUGH MCQS- 20th -Nov-2019 ......................................................... 127
RTM- REVISION THROUGH MCQS - 21th -Nov-2019 ........................................................ 133
RTM- REVISION THROUGH MCQS - 22th -Nov-2019 ........................................................ 138
RTM- REVISION THROUGH MCQS - 23th -Nov-2019 ........................................................ 144
RTM- REVISION THROUGH MCQS - 25th -Nov-2019 ........................................................ 151
RTM- REVISION THROUGH MCQS - 26th -Nov-2019 ........................................................ 158
RTM- REVISION THROUGH MCQS - 27th -Nov-2019 ........................................................ 164
RTM- REVISION THROUGH MCQS - 28th -Nov-2019 ........................................................ 169
RTM- REVISION THROUGH MCQS- 29th -Nov-2019 ......................................................... 173
RTM- REVISION THROUGH MCQS - 30th -Nov-2019 ........................................................ 178

Telegram: https://t.me/insightsIAStips
4
Youtube: https://www.youtube.com/channel/UCpoccbCX9GEIwaiIe4HLjwA
Revision Through MCQs (RTM) Compilation (November 2019)
Revision Through MCQs (RTM) Compilation (November 2019)

(c) 2 and 3
(d) 1, 2 and 3
RTM-QUESTIONS 4. With reference to Indian polity,
consider the following statements
1. “This place lies on the right bank of 1. The present delimitation of
a distributary of river constituencies has been done
Swarnamukhi about eighty on the basis of 2001 census
kilometers from Tirupati and under the provisions of
Nellore. Recent Explorations Delimitation Act, 2002
conducted in and around this site 2. Delimitation Commission of
within a radius of 15 kilometers India’s orders can be
revealed important vestiges viz., challenged in court of law
fortified early historic settlement 3. Delimitation commissions have
Sunbrahmanya temple, unique rock been set up four times in the
cut laterite stepped well at past
Yakasiri, Vishnu temple” Which of the statements given
The above passage describes above is/are correct?
which of the following (a) 1 and 2
archeological site in India? (b) 3 only
(a) Keeladi (c) 1 and 3
(b) Gottiprolu (d) 1, 2 and 3
(c) Pavurallakonda 5. Recently PESA has been repeatedly
(d) Piklihal mentioned in media, related to
2. With reference to significance of (a) OECD
October 31 in Indian History, (b) UNDP
consider the following statements (c) UNICEF
1. On this day, Indian Prime (d) UNESCO
Minister Rajiv Gandhi was 6. Consider the following pairs
assassinated Disease Caused by
2. This day was the Birth 1. Lymphatic Filariasis Parasite
Anniversary of Sardar 2. Malaria Mosquito
Vallabhbhai Patel 3. Rabies Virus
Which of the statements given 4. Cholera Bacteria
above is/are correct? 5. Rubella Fungi
(a) 1 only Select the correct answer using the
(b) 2 only code below
(c) Both 1 and 2 (a) 1, 2, 3 and 4
(d) Neither 1 nor 2 (b) 2, 3, 4 and 5
3. With reference to Indian modern (c) 1, 3, 4 and 5
history, consider the following (d) 2, 4 and 5
statements 7. National Health Profile, 2019 has
1. In 1928, No Tax Campaign led been recently released by
by Vallabhbhai Patel in (a) NITI
Gujarath (b) NHFS
2. In1931, Vallabhbhai Patel (c) CBHI
elected as the President of (d) NHRC
Indian National Congress 8. Which of the following Indian states
3. Patel participated in the celebrate November 1 as their
famous Salt Satyagraha Formation Day?
movement and led it from front 1. Karnataka
Which of the statements given 2. Kerala
above is/are correct? 3. Chhattisgarh
(a) 1 and 2 4. Punjab
(b) 3 only 5. Haryana
Revision Through MCQs (RTM) Compilation (November 2019)

6. Madhya Pradesh 13. With reference to Graded


Select the correct answer using the Response Action Plan (GRAP),
code below consider the following statements
(a) 1, 2, 3 and 4 1. It works only as an emergency
(b) 1, 3, 4 and 5 and short term measure
(c) 1, 4, 5 and 6 2. GRAP does not include action
(d) 1, 2, 3, 4, 5 and 6 by various state governments
9. ENSURE Portal is primarily related to be taken throughout the
to year to tackle air pollution
(a) Public service Which of the statements given
(b) Subsidy payment above is/are correct?
(c) Data management (a) 1 only
(d) Education (b) 2 only
10. Who was the author of The (c) Both 1 and 2
Philosophy of Rabindranath (d) Neither 1 nor 2
Tagore? 14. With reference to Armed Forces
(a) Sarvepalli Radhakrishnan (Special Powers) Act (AFSPA),
(b) Subhas Chandra Bose consider the following statements
(c) Ras Bihari Bose 1. AFSPA was first enacted as an
(d) Jawahar Lal Nehru ordinance in the backdrop of
11. With respect to Electoral Bond’s, Quit India Movement
consider the following statements 2. AFSPA was first enacted to deal
1. Electoral bond is a bearer with the Naga insurgency in the
instrument in the manner of a Assam region.
promissory note 3. Governor of a State and the
2. Bonds shall not carry the Central Government are
donor's name, although the empowered to declare any part
payee will have to fulfil KYC or full of any state as a
3. Every party that is registered disturbed area
under section 29A of the Which of the given above
Representation of the Peoples statements is/are correct?
Act, 1951 are eligible to receive (a) 1 and 2
money through electoral bonds (b) 2 and 3
Which of the given above (c) 1 and 3
statements is/are correct? (d) All of the above
(a) 1 and 2 15. Recently Chrysaor, Framroot and
(b) 2 and 3 Zero-day vulnerability has been
(c) 1 and 3 repeatedly mentioned in media
(d) All of the above with apprehension. These are
12. Consider the following statements related to
1. Tiware dam is located in (a) Climate change
Maharashtra (b) Kerala mass killing
2. Polavaram is a multi-purpose (c) Cyber security
irrigation project on the river (d) Agri pesticide
Krishna 16. Consider the following statements
Which of the statements given 1. CERT-In is nodal government
above is/are Not correct? agency that deals with cyber
(a) 1 only security threats like hacking
(b) 2 only and phishing in India
(c) Both 1 and 2 2. Defence cyber agency focuses
(d) Neither 1 nor 2 on non-civilian cyber issues

Telegram: https://t.me/insightsIAStips
7
Youtube: https://www.youtube.com/channel/UCpoccbCX9GEIwaiIe4HLjwA
Revision Through MCQs (RTM) Compilation (November 2019)

Which of the statements given 1. Currently Inner Line Permit is


above is/are correct? operational in Arunachal
(a) 1 only Pradesh, Mizoram and Tripura
(b) 2 only 2. Assam has the largest and
(c) Both 1 and 2 smallest river islands in the
(d) Neither 1 nor 2 world
17. Serious Fraud Investigation Office 3. Population of all the
(SFIO) falls under which ministry Northeastern states is close to
(a) MoCA the population of Odisha
(b) MoF Which of the above statements
(c) MHA is/are correct?
(d) MoD (a) 1 and 2 Only
18. With reference to diffused light, (b) 1 Only
consider the following statements (c) 2 and 3 Only
1. Light is diffused due to its dual (d) 1,2 and 3
nature of wave and particle 22. With reference to East Asia
2. Light diffuses when it gets Summit(EAS), consider the
scattered following statements
3. In atmosphere light gets 1. EAS is an initiative of SAARC
diffused by gas molecules 2. It is a forum held annually by
Which of the above statements leaders of 18 countries from all
is/are correct? over the world
(a) 1 and 2 Only 3. EAS acts as an alternative to
(b) 1 Only the APEC in which India
(c) 2 and 3 Only doesn’t enjoy the membership
(d) 1,2 and 3 Which of the above statements
19. Consider the following events is/are correct?
1. First Famine Commission (a) 1 and 2 Only
2. Arms Act to restrict only (b) 3 Only
Indians to carry arms (c) 2 and 3 Only
3. Introduced uniform salt tax (d) 1,2 and 3
throughout British India 23. Recently SCOJTEX- 2019, has
4. War with Afghans been in news sometimes related to
Under whose rule the above given (a) SCO
events happened? (b) ASEAN
(a) Lord Ripon (c) SAARC
(b) Lord Lytton (d) RCEP
(c) Lord Curzon 24. Which of the following pairs are
(d) None of the above shared all correctly matched?
four events simultaneously 1. Global Microscope on Financial
20. Recently Gandhipedia has been in Inclusion report- Economist
news for sometimes is launched by Intelligence Unit
(a) Wikipedia 2. Global Innovation Index-World
(b) Ministry of culture Economic Forum
(c) UNESCO 3. Southeast Asia Energy Outlook-
(d) National Council for Science ASEAN
Museums Find the correct answer using the
code below
21. With reference to North Eastern (a) 1 and 2 Only
India, Consider the following (b) 1 Only
statements (c) 3 Only
(d) 2 and 3

Telegram: https://t.me/insightsIAStips
8
Youtube: https://www.youtube.com/channel/UCpoccbCX9GEIwaiIe4HLjwA
Revision Through MCQs (RTM) Compilation (November 2019)

25. Consider the following statements 29. In ancient India, ‘Kutagarashala’


1. ‘Red Atlas Action Plan Map’ referred to
atlas was prepared by NDRF (a) A place where children of high
2. Coastal Flood Warning Brahman family were given
System(CFLOWS) app for education.
Chennai is the first integrated (b) A place where the head of a
coastal flood warning system tribe resided.
Which of the statements given (c) A place where the debate
above is/are correct? between philosophers took
(a) 1 only place.
(b) 2 only (d) None of the statements (a), (b)
(c) Both 1 and 2 and (c) are correct.
(d) Neither 1 nor 2 30. With reference to Chola Education
26. Consider the following and Literature, consider the
National Waterways State Served following statements:
1. NW-2 Meghalaya 1. Temples and mathas served as
2. NW-4 Kerala the only educational centres
3. NW6 Assam 2. Not only Vedas and Epics, but
Which of the above is/are correctly subjects like mathematics and
matched? medicine were also taught
(a) 1 and 3 3. Endowment of lands was made
(b) 2 only to run educational centres
(c) 3 only Select the correct answer using the
(d) 2 and 3 codes below.
27. With Reference to National (a) 1 and 3
Institute of Ocean Technology (b) 2 and 3 only
(NIOT), consider the following (c) 3 only
statements (d) 1 only
1. It is an autonomous society 31. With reference to Welfare policies
2. It falls under Department of in India, consider the following
Ocean Development statements
Which of the statements given 1. Skills Build platform is
above is/are correct? launched by Directorate
(a) 1 only General of Supplies under the
(b) 2 only aegis of Ministry of Skill
(c) Both 1 and 2 Development &
(d) Neither 1 nor 2 Entrepreneurship (MSDE)
28. Recently Dustlik 2019 has been in 2. Program of Skills Build Platform
news sometimes related to facilitates co-operation and
(a) It is the first-ever Joint collaboration with private
Exercise between India and player
Uzbekistan 3. Pradhan Mantri Kaushal Vikas
(b) It is the first-ever Joint Yojana (PMKVY) is a flagship
Exercise between India and scheme of MSDE
Kazakhstan Which of the given above
(c) It is the first-ever Joint statements is/are correct?
Exercise between India and (a) 1 and 3
Turkmenistan (b) 2 only
(d) It is the first-ever Joint (c) 2 and 3
Exercise between India and (d) 1, 2 and 3
Afghanistan

Telegram: https://t.me/insightsIAStips
9
Youtube: https://www.youtube.com/channel/UCpoccbCX9GEIwaiIe4HLjwA
Revision Through MCQs (RTM) Compilation (November 2019)

32. With reference to Regional (a) It refers to the proportion of


Comprehensive Economic highly liquid assets held by
Partnership (RCEP), consider the companies to ensure their
following statements ongoing ability to meet short-
1. India’s trade deficit with the term obligations.
RCEP nations is $105 billion (b) It mean liquid assets that can
2. RCEP is a preferential trade be readily sold or immediately
agreement originally devised to converted into cash at little or
consist of 16 countries across no loss of value, or used as
the Asia-Pacific region collateral to obtain funds in a
3. RCEP’s administrative core is range of stress scenarios
ASEAN (c) It allows banks borrow money
Which of the given above through repurchase
statements is/are correct? agreements
(a) 1 and 2 (d) None of the above
(b) 2 and 3 36. With reference to NAVIC, consider
(c) 3 only the following statements
(d) 1 and 3 1. It is an independent global
33. Recently, India negotiating a navigation satellite system
agreement named Agreement on 2. It is designed to provide
Reciprocal Logistics Support (ARLS) accurate position information
with which of the following nation service to users in India as well
(a) USA as the region extending up to
(b) RUSSIA 5500 sqkm from its boundary
(c) JAPAN 3. The space segment consists of
(d) CHINA the IRNSS with an operational
34. With reference to NBFC, consider name of NAVIC constellation of
the following statements eight satellites
1. NBFC is a company registered Which of the above statements
under the Companies Act,1956 given is/are Not correct?
or 2013 (a) 1 and 3
2. NBFC conducts financial (b) 3 only
activity as principal business ie (c) 1 and 2
when a company’s financial (d) 1, 2 and 3
assets constitute more than 50 37. Recently, ICEDASH and ATITHI
per cent of the total assets and has been in news sometimes
income from financial assets related to
constitute more than 50 per (a) New IT Initiatives to regulate
cent of the gross income custom clearance in airport
3. Deposit insurance facility of only
Deposit Insurance and Credit (b) New IT Initiatives to regulate
Guarantee Corporation is not custom clearance in port only
available to depositors of (c) New IT Initiatives to regulate
NBFCs, unlike in case of banks custom clearance in airport
Which of the above given and port
statements is/are correct? (d) New IT Initiatives to regulate
(a) 1 and 3 custom clearance in airport,
(b) 3 Only border bus station and port
(c) 2 and 3 38. Polypedates bengalensis is a
(d) 1, 2 and 3 newly discovered frog species from
35. What do you mean by the term (a) Western Ghats
called “LCR”? (b) West Bengal

Telegram: https://t.me/insightsIAStips
10
Youtube: https://www.youtube.com/channel/UCpoccbCX9GEIwaiIe4HLjwA
Revision Through MCQs (RTM) Compilation (November 2019)

(c) Western Himalaya 1. Alzheimer’s disease is a non-


(d) West Meghalaya communicable disease
39. Consider the following statements 2. There is no cure/treatment
1. White Biotechnology is applied available for Alzheimer’s
to industrial process disease
2. Red Biotechnology is concerned Which of the given above
with development of Drugs statements is/are correct?
3. Green Biotechnology is related (a) 1 only
Agriculture (b) 2 only
Which of the above given (c) Both 1 and 2
statements is/are correct? (d) Neither 1 nor 2
(a) 1 and 2 43. Recently JCPOA has been in news
(b) 3 only sometimes commonly referred to
(c) 2 only (a) USA Nuclear Deal
(d) 1, 2 and 3 (b) Iran Nuclear Deal
40. With reference to ISRO, consider (c) Iraq Nuclear Deal
the following statements (d) Both A and C
1. Indian National Committee for 44. With reference to Paris Agreement,
Space Research (INCOSPAR) consider the following statements
was established under 1. Global stocktake established
Jawaharlal Nehru government under the Paris Agreement, it
in 1962 under the Department takes place every five years
of Atomic Energy 2. Nationally determined
2. first Indian-made satellite was contributions should be
the Rohini-75 reported every five years and
3. first Indian-built launch vehicle are to be registered by the
was ASLV UNFCCC Secretariat
Which of the above given 3. India pledged to reduce the
statements is/are correct? emissions intensity of its GDP
(a) 1 and 2 by 33 to 35 per cent by 2030
(b) 2 and 3 from 2005 level as NDC
(c) 2 only Which of the above given
(d) 1 and 3 statements is/are correct?
41. With reference to Indian History, (a) 1 and 3
consider the following statements (b) 3 only
1. First Battle of Panipat laid the (c) 2 and 3
foundation of the Mughal (d) All of the above
Empire in India 45. With reference to Indian Economy,
2. Third Battle of Panipat fought consider the following statements
between Maratha forces and 1. Trade deficit represents an
invading armies of Iran general outflow of domestic currency to
Abdali foreign markets
Which of the given above 2. In the long run, Trade deficit
statements is/are correct? may lead to fewer jobs
(a) 1 only 3. Trade deficit signifies demand
(b) 2 only in the domestic economy is not
(c) Both 1 and 2 being met by the domestic
(d) Neither 1 nor 2 producers
42. With reference to Alzheimer’s Which of the above given
disease, consider the following statements is/are correct?
statements (a) 1 and 2
(b) 1 and 3

Telegram: https://t.me/insightsIAStips
11
Youtube: https://www.youtube.com/channel/UCpoccbCX9GEIwaiIe4HLjwA
Revision Through MCQs (RTM) Compilation (November 2019)

(c) 2 and 3 (a) 1 and 2


(d) All of the above (b) 2 and 3
46. Recently Wasteland Atlas has (c) 1 and 3
been released by (d) All of the above
(a) Ministry of Home Affairs 49. Which of the following established
(b) Ministry of Rural Development the network of Population Research
(c) Ministry of Environment, Centres across India
Forest and Climate Change (a) NITI Aayog
(d) National Institution for (b) MoH&FW
Transforming India (c) MHRD
47. “It lies about 125 m below sea (d) MHA
level, and is one of the hottest and 50. With reference to welfare policies
most inhospitable places on Earth— in India, consider the following
temperatures average 34.5 Celsius statements
but have been recorded above 50 1. Bhavantar Bhugtan Yojana is a
Celsius. Numerous sulfur springs, scheme of the Government of
volcanoes, geysers, acidic pools, Odisha
vast salt pans, and colorful 2. Pradhan Mantri Kisan Samman
mineral-laden lakes dot the area, Nidhi is a Central Sector
which formed above the divergence scheme
of three tectonic plates. Volcanic 3. Rythu Bandhu scheme of
activity heats spring water, Andhra Pradesh provides
bringing sulfur and iron to the ₹4,000 per acre for every
surface, leaving behind yellow, season to all the farmers of the
green, and orange deposits. For state
centuries, locals have been trekking Which of the given above
in with camel caravans to mine the statements is/are not correct?
salt by hand, and in recent years, a (a) 2 only
few have been guiding tourists into (b) 1 and 3
the alien-looking landscape” (c) 2 and 3
The above given passage describes (d) All of the above
which of the following geographic 51. With reference to Indian
location? Constitution, consider the
(a) Ethiopia's Danakil Depression following statements
(b) Eastern part of Sahar Desert 1. President can disqualify MLAs
(c) Somalia’s Guban Desert for holding an ‘office of profit’
(d) Libyan Desert 2. Office of profit seeks to enforce
48. With reference to Super the principle of separation of
Conductivity, consider the power between the legislature
following statements and the executive
1. It is achieved when electric 3. State legislatures have power to
resistance in a medium enact laws for exempting
becomes zero certain offices from the purview
2. A superconductor can carry a of office of profit
current indefinitely without Which of the given above
losing any energy statements is/are correct?
3. 100 times more magnetic field (a) 2 and 3
is generated in a (b) 2 only
superconductor as compared to (c) 3 only
a normal conductor. (d) 1, 2 and 3
Which of the above given 52. With reference to Bay of Bengal
statements is/are correct? Initiative for Multi-Sectoral

Telegram: https://t.me/insightsIAStips
12
Youtube: https://www.youtube.com/channel/UCpoccbCX9GEIwaiIe4HLjwA
Revision Through MCQs (RTM) Compilation (November 2019)

Technical and Economic above passage, consider the


Cooperation (BIMSTEC), consider following statements
the following statements 1. Sarvajna and Tiruvalluvar were
1. BIST-EC was founded after contemporaries
Asian Financial crisis 2. Sarvajna famous for his pithy
2. 5 member nations of BIMSTEC three-lined poems called tripadi
are also member of SAARC 3. Manimekalai is a Tamil epic
3. BIMSTEC Permanent composed by Tiruvalluvar
Secretariat is at Katmandu Which of the given above
4. In 2004, Nepal and Bhutan statements is/are correct?
become full members (a) 1 and 3
Which of the above given (b) 2 only
statements is/are correct? (c) 3 only
(a) 1, 2 and 4 (d) 1, 2 and 3
(b) 2 and 4 56. Recently MHRD has launched
(c) 2, 3 and 4 Shaala Darpan portal for
(d) 1, 2, 3 and 4 (a) Navodaya Vidyalaya Samiti
53. What do you mean by Core (NVS)
investment companies? (b) Central Board of Secondary
(a) CICs are non-banking financial Education(CBSE)
companies (c) Vidya Bharati of Rashtriya
(b) CICs are special purpose Swayamsevak Sangh (RSS)
vehicles with asset size of less (d) Morarji Desai Residential
than ₹100 crore Schools
(c) CICs are non-banking financial 57. Which of the following is the
companies with asset size of World’s southernmost city?
less than ₹500 crore (a) Puerto Williams, Chile
(d) CICs are non-banking financial (b) Puerto Williams, Argentina
companies with asset size of (c) Puerto Williams, Falkland
₹100 crore and above island
54. Which of the following fund is not (d) Puerto Williams, Antarctica
included under SEBI regulated 58. Cotigao Wildlife sanctuary is
Alternative investment fund? located in
(a) Angel Funds (a) Goa
(b) Mutual Funds (b) Karnataka
(c) SME Funds (c) Maharashtra
(d) Social Venture Funds (d) Assam
55. “Sarvajna and Tiruvalluvar are 59. “It is the largest active volcano in
popular Kannada and Tamil poets, Europe and one of the world’s most
respectively. Statues of Sarvajna in frequently erupting volcanoes. It is
Tamil Nadu and Tiruvalluvar in also the volcano with the longest
Karnataka has been unveiled in record of continuous eruption. The
August 2009 as a symbolic effort to mountain’s largest feature is the
bolster ties between the two Indian Valle del Bove (Valley of the Ox), a
states, whose relationship has large horseshoe-shaped caldera on
been strained by issues related to the eastern slope. In June 2013, it
sharing of Kaveri water and was added to the list of UNESCO
Hogenakkal water supply power World Heritage Sites. Due to its
project” history of recent activity and
With reference to Sarvajna and nearby population, it has been
Tiruvalluvar mentioned in the designated a Decade Volcano by
the United Nations”

Telegram: https://t.me/insightsIAStips
13
Youtube: https://www.youtube.com/channel/UCpoccbCX9GEIwaiIe4HLjwA
Revision Through MCQs (RTM) Compilation (November 2019)

The above given passage refers and 5% blending of bio-diesel


which of the following volcano in diesel by 2030.
(a) Mount Etna 2. Government has reduced GST
(b) Mount Fuji on ethanol for blending in fuel
(c) Mayon Volcano from 18% to 5%
(d) Mount Stromboli 3. Repurpose Used Cooking Oil
60. Recently ‘UdChalo’ has been in (RUCO) launched by MNRE
news sometimes is related to aims for an ecosystem that will
(a) Travel portal for the personal enable the collection and
travel of the military and conversion of used cooking oil
paramilitary forces to biodiesel.
(b) Skill development training for Which of the following above
disables statements is true?
(c) Happy curriculum program for (a) 1 and 2
children (b) 2 and 3
(d) New portal for STEM program (c) 1 and 3
61. Which of the following are not (d) 1, 2 and 3
necessarily the consequences of 64. Recently India Justice Report (IJR)-
the proclamation of the President’s first-ever ranking of Indian states
rule in a State? on justice delivery has been
1. Dissolution of the State released. Which is prepared by
Legislative Assembly (a) Bar council of India
2. Removal of the Council of (b) Association of all India lawyers
Ministers in the State (c) Tata trusts with Daksh
3. Dissolution of the local bodies (d) Supreme courts Centre for
Select the correct answer using the Research and Planning (CRP)
code given below: 65. With reference to Feni River,
(a) 1 and 2 only consider the following statements
(b) 1 and 3 only 1. It is a transnational river
(c) 2 and 3 only 2. It originates in Tripura
(d) 1, 2 and 3 3. Recently there is a river sharing
62. Which one of the following dispute between Tripura state
suggested that the Governor should and west Bengal
be an eminent person from outside Which of the given above
the State and should be a detached statements is/are correct?
figure without intense political links (a) 1 and 3
or should not have taken part in (b) 2 and 3
politics in the recent past? (c) 1 and 2
(a) First Administrative Reforms (d) 1, 2 and 3
Commission (1966) 66. Which of the following is/are not a
(b) Rajamannar Committee(1969) Credit Rating Agency in India
(c) Sarkaria Commission (1983) 1. Brickwork Ratings
(d) National Commission to Review 2. SMERA
the Working of the 3. Moody
Constitution(2000) Find the appropriate answer using
63. With reference to Government the code below:
Policies on Energy, Consider the (a) 3 only
following statements (b) 1 and 3
1. National Policy on Biofuels-2018 (c) 2 and 3
envisages target of 20% (d) All of the above
blending of ethanol in petrol

Telegram: https://t.me/insightsIAStips
14
Youtube: https://www.youtube.com/channel/UCpoccbCX9GEIwaiIe4HLjwA
Revision Through MCQs (RTM) Compilation (November 2019)

67. What is common to places known (c) 2-1-4-3


as Barna, Tawa, Matatila and (d) 1-2-4-3
Tilaiya? 71. Which of the following statements
(a) Recently discovered uranium is/are true with respect to Steel
deposits Scrap Recycling Policy
(b) Sites of Sangam age in 1. Policy that aims to reduce
Tamilnadu imports, conserve resources
(c) Hots spots of Naxalism in and save energy
central India 2. Policy is based on 6Rs
(d) Water reservoirs principles
68. “The name of this community 3. It contains the provision of
means people living in the woods, Extended Producer
these are an ethnic group living in Responsibility
Thailand, Laos, India and Vietnam. Select the correct answer using the
They settled mostly along code below
waterways. Traditionally they live (a) 1 only
in small houses that are built on (b) 1 and 2
stilts. The houses are arranged (c) 2 and 3
around a central meeting building (d) 1,2 and 3
around a circle. These tribals are 72. Consider the following statements
mainly believe in Animism, but 1. Kalapani, a disputed territory
some (especially in Thailand), are between India and Nepal is
adherents of Theravada Buddhism, located in Uttar Pradesh
which is observed along with 2. According to Indian mythology,
Animism, which includes worship Great sage Vyasa meditated at
of ancestors, the spirits of the rice kalapani, giving the region its
and fire spirits. Sacred objects to name
this community include relics and 3. Kalapani is controlled by
fragments of ancient weapons and India's Indo-Tibetan Border
household objects” Police
The passage describes which of the Which of the given above
following tribal community? statements is/are correct?
(a) Brus Tribe (a) 1 only
(b) Kuki Tribe (b) 2 and 3
(c) Angami Tribe (c) 3 only
(d) Garo Tribe (d) 1 and 3
69. 2019 edition of SDG Gender Index 73. Which of the following is not a
has been developed by member of Quad countries
(a) UNDP (a) Japan
(b) WEF (b) India
(c) Equal Measure 2030 (c) Australia
(d) Amnesty international (d) South Korea
70. Arrange the following from South 74. “It is the only spacecraft to have
to North direction visited all four gas giant planets —
1. Jerusalem Jupiter, Saturn, Uranus and
2. Baghdad Neptune — and discovered 16
3. Sana moons, as well as phenomena like
4. Muscat Neptune’s mysteriously transient
Select the correct answer using the Great Dark Spot, the cracks in
code below: Europa’s ice shell, and ring
(a) 3-4-2-1 features at every planet”
(b) 3-4-1-2

Telegram: https://t.me/insightsIAStips
15
Youtube: https://www.youtube.com/channel/UCpoccbCX9GEIwaiIe4HLjwA
Revision Through MCQs (RTM) Compilation (November 2019)

The given above passage describes (d) 1, 2 and 3


which of the following Space 79. With reference to Indian Polity,
Mission? consider the following statements
(a) Voyager 1 Mission 1. Cabinet committees are extra-
(b) Voyager 2 Mission constitutional in nature
(c) New Horizon Mission 2. Presently, there are 8 Cabinet
(d) Cassini-Huygens Mission Committee under the
75. With reference to Fiber Optic Transaction of Business Rules
Network, consider the following 3. Newly constituted, Cabinet
statements Committee on Employment &
1. Fiber is preferred over electrical Skill Development will be
cabling when high bandwidth, headed by Prime minister
long distance, or immunity to Which of the given above
electromagnetic interference statements is/are correct?
are required (a) 1 only
2. Optical fiber has large (b) 2 only
advantages over existing copper (c) 1 and 2 only
wire in long-distance due to (d) 1, 2 and 3
much lower attenuation and 80. Recently New START treaty has
interference been in news sometimes is related
3. Fiber optics transmit data in to
the form of light particles (a) Nuclear arms reduction treaty
Which of the given above between the USA and Russia
statements is/are correct? (b) Nuclear arms reduction treaty
(a) 1 only between the Turkey and Syria
(b) 2 and 3 only (c) Nuclear arms reduction treaty
(c) 1 and 2 only between the Iran and Isreal
(d) 1, 2 and 3 (d) Nuclear arms reduction treaty
76. No Money For Terror Conference is between the Saudi Arabia and
organised by Yeman
(a) FATF 81. With reference to Thunder and
(b) FIU Lightning, consider the following
(c) OECD statements
(d) Transparency International 1. Lightning is an electrical energy
77. Tiger TRIUMP, It is a first tri- while thunder is a sound energy
services amphibious exercise 2. Lightning is seen first before
between India and one can hear the sound of
(a) UK thunder
(b) USA 3. Lightning is more dangerous
(c) Singapore and destructive than thunder
(d) Japan Which of the given above
78. Which of the following pairs is/are statements is/are correct?
correctly matched? (a) 1 and 2
1. Cyclone Bul Bul- Bay of Bengal (b) 2 only
2. Cyclone Pabuk - South China (c) 1 and 3 only
Sea and Andaman Sea (d) 1, 2 and 3
3. Cyclone Vayu - Arabian Sea 82. Consider the following statements
Select the correct answer using the with reference to Ayodhya Dispute
code below 1. Three-domed mosque(Babari)
(a) 1 only built by Mir Baqi of Mughal
(b) 1 and 3 kingdom in 1528 is in the
(c) 2 and 3 Jaunpuri style

Telegram: https://t.me/insightsIAStips
16
Youtube: https://www.youtube.com/channel/UCpoccbCX9GEIwaiIe4HLjwA
Revision Through MCQs (RTM) Compilation (November 2019)

2. In this case, Supreme Court 2. Office of profit seeks to enforce


invoked doctrine of adverse the principle of separation of
possession power between the legislature
3. Ayodya case is the second and the executive
longest herd case in the history 3. State legislatures have power to
of supreme court after the enact laws for exempting
landmark Keshvanand Bharti certain offices from the purview
case in 1973 of office of profit
Which of the given above Which of the given above
statements is/are correct? statements is/are correct?
(a) 1 and 2 (a) 2 and 3
(b) 1 and 3 (b) 2 only
(c) 3 only (c) 3 only
(d) 1, 2 and 3 (d) 1, 2 and 3
83. Recently HS-code has been in 87. With reference to Bay of Bengal
news sometimes was developed by Initiative for Multi-Sectoral
(a) World Customs Organization Technical and Economic
(b) World Trade Organization Cooperation (BIMSTEC), consider
(c) Reserve Bank of India the following statements
(d) Ministry of Finance 1. BIST-EC was founded after
84. With reference to Fall army worm, Asian Financial crisis
consider the following statements 2. 5 member nations of BIMSTEC
1. It is also known as Spodoptera are also member of SAARC
frugiperda or fruit destroyer 3. BIMSTEC Permanent
2. It is native of the tropical and Secretariat is at Katmandu
sub-tropical of Asia 4. In 2004, Nepal and Bhutan
3. It was reported in India for the become full members
first-time in Maharashtra Which of the above given
4. Pest can attack at least 80 statements is/are correct?
types of crops including bajra, (a) 1, 2 and 4
jawar, ragi, paddy, wheat and (b) 2 and 4
vegetables (c) 2, 3 and 4
Which of the given above (d) 1, 2, 3 and 4
statements is/are correct? 88. What do you mean by Core
(a) 1 and 4 investment companies?
(b) 1, 2 and 4 (a) CICs are non-banking financial
(c) 1, 3 and 4 companies
(d) All of the above (b) CICs are special purpose
85. Which of the following is often vehicles with asset size of less
dubbed as “sea monsters” and than ₹100 crore
largest aquatic carnivorous reptiles (c) CICs are non-banking financial
that have ever lived on this earth? companies with asset size of
(a) Pliosaur less than ₹500 crore
(b) Plesiosaur (d) CICs are non-banking financial
(c) Dinosaur companies with asset size of
(d) Caiman ₹100 crore and above
86. With reference to Indian 89. Which of the following fund is not
Constitution, consider the included under SEBI regulated
following statements Alternative investment fund?
1. President can disqualify MLAs (a) Angel Funds
for holding an ‘office of profit’ (b) Mutual Funds
(c) SME Funds

Telegram: https://t.me/insightsIAStips
17
Youtube: https://www.youtube.com/channel/UCpoccbCX9GEIwaiIe4HLjwA
Revision Through MCQs (RTM) Compilation (November 2019)

(d) Social Venture Funds 3. In conformity with seed treaty,


90. “Sarvajna and Tiruvalluvar are India enacted Protection of Plant
popular Kannada and Tamil poets, Varieties and Farmers’ Rights
respectively. Statues of Sarvajna in (PPV&FR) Act, 2001
Tamil Nadu and Tiruvalluvar in Which of the given above
Karnataka has been unveiled in statements is/are correct?
August 2009 as a symbolic effort to (a) 1 and 3
bolster ties between the two Indian (b) 2 only
states, whose relationship has (c) 1 and 2
been strained by issues related to (d) All of the above
sharing of Kaveri water and 93. 2019 Brown to Green Report has
Hogenakkal water supply power been published by
project” (a) German Watch
With reference to Sarvajna and (b) New Climate Institute
Tiruvalluvar mentioned in the (c) Climate Action Network
above passage, consider the International
following statements (d) Climate Transparency
1. Sarvajna and Tiruvalluvar were 94. Consider the following statements
contemporaries 1. Largest natural fresh water
2. Sarvajna famous for his pithy lake of India is Dal Lake
three-lined poems called tripadi 2. Chilaka Lake is the largest
3. Manimekalai is a Tamil epic inland salt water lake of India
composed by Tiruvalluvar 3. Largest brackish water lake of
Which of the given above India is Sambar Lake
statements is/are correct? 4. Pulicat Lagoon is considered to
(a) 1 and 3 be the second largest brackish
(b) 2 only water body in India
(c) 3 only Which of the given above
(d) 1, 2 and 3 statements is/are not correct?
(a) 2, 3 and 4
91. With reference to Swachh – (b) 1, 2 and 3
Nirmal Tat Abhiyaan, consider the (c) 2 and 4
following statements (d) All of the above
1. It is a pan India beaches 95. Recently MHRD has launched
cleaning drive Shaala Darpan portal for
2. The collected waste under this (a) Navodaya Vidyalaya Samiti
initiative will be processed as (NVS)
per extant Waste Management (b) Central Board of Secondary
Rules, 2016 Education(CBSE)
Which of the given above (c) Vidya Bharati of Rashtriya
statements is/are correct? Swayamsevak Sangh (RSS)
(a) 1 only (d) Morarji Desai Residential
(b) 2 only Schools
(c) Both 1 and 2 96. Which of the following is the
(d) Neither 1 nor 2 World’s southernmost city?
92. With reference to Seed Treaty, (a) Puerto Williams, Chile
consider the following statements (b) Puerto Williams, Argentina
1. Access and benefit sharing is (c) Puerto Williams, Falkland
the one of the provision of the island
treaty (d) Puerto Williams, Antarctica
2. Treaty recognizes Farmers’ 97. Cotigao Wildlife sanctuary is
rights located in

Telegram: https://t.me/insightsIAStips
18
Youtube: https://www.youtube.com/channel/UCpoccbCX9GEIwaiIe4HLjwA
Revision Through MCQs (RTM) Compilation (November 2019)

(a) Goa 101. With reference to President Rule,


(b) Karnataka consider the following statements
(c) Maharashtra 1. Proclamation imposing
(d) Assam President’s Rule in states has
98. Consider the following about been originally mentioned in
subsidies the constitution of India
1. Subsidies distort price signals 2. Upon the imposition of this
and thus demand for goods rule, Vidhan Sabha is either
and services dissolved or prorogued
2. Subsidies are faulty as they do Which of the given above
not follow principle of equity statements is/are correct?
3. Subsidies should be phased (a) 1 only
out by the government of India (b) 2 only
Select the correct answer using the (c) Both 1 and 2
codes below: (d) Neither 1 nor 2
(a) 1 and 2 only 102. With reference to discretionary
(b) 2 only powers of the President of India,
(c) 1 and 3 only consider the following statements
(d) 1, 2 and 3 1. Veto power
99. What do you mean by Inward 2. Right to be informed of all
looking trade strategy? important deliberation of
(a) To follow a policy of import council of ministers
substitution 3. Appointment of Prime Minister
(b) To not be part of the trade in case of a hung parliament
system by staying independent Which of the given above
and disconnected from world statements is/are correct?
markets in all aspects. (a) 2 only
(c) To enable a greater interaction (b) 1 and 3 only
amongst centre and various (c) 2 and 3 only
parts of India (d) 1, 2 and 3
(d) None of the above 103. The state level minister of
100. Consider the following about parliament can hold the portfolio
Total Fertility Rates in India during pleasure of the
1. since not many women use (a) Governor of the state
methods of reversible (b) The Prime Minister
contraception, they have little (c) The President of India
control over when they start (d) None of above
having children, but only seem 104. Which of the following statement
to have control over when they is true?
stop having children (a) Commutation means exchange
2. the average number of children of one thing for another
born to a women over her (b) Remissions means awarding a
lifetime has dropped below lesser punishment on some
replacement levels for all special grounds
religious communities barring (c) Respite means the reduction of
Hindus and Muslims the amount of sentence without
Which of the above statements changing its character
is/are correct? (d) All of the above statements are
(a) 1 only true
(b) 2 only 105. Which of the following state
(c) 1 and 2 only recently draft a law to protect
(d) Neither 1 nor 2 journalist?

Telegram: https://t.me/insightsIAStips
19
Youtube: https://www.youtube.com/channel/UCpoccbCX9GEIwaiIe4HLjwA
Revision Through MCQs (RTM) Compilation (November 2019)

(a) Bihar 2. Non-Resident Indians (NRIs)


(b) Kerala cannot file RTI applications to
(c) Odisha seek governance-related
(d) None of the above information
106. Recently Project Zero has been in 3. Right to Information is a part of
news is introduced by fundamental rights under
(a) Amazon Article 19(1) of the Constitution
(b) Flipkart Which of the given above
(c) Wallmart statements is/are correct?
(d) Microsoft (a) 1 and 2
107. Recently Raman 1.0 has been in (b) 2 and 3
news sometimes is related to (c) 1 and 3
(a) Upcoming movie on Sir C V (d) All of the above
Raman 112. With reference to Green climate
(b) Physics laboratory set up by fund, consider the following
IISC statements
(c) New metrics to quantify 1. The fund was set up under
Scattering of light world Bank
(d) None of the above 2. GCF was formally established
108. Recently IAF signed a deal to during the COP-16 in Cancun
procure SPICE 2000 guided bombs 3. Fund’s investments can be in
from the form of grants, loans,
(a) Israel equity or guarantees
(b) USA Which of the given above
(c) Russia statements is/are correct?
(d) South Korea (a) 1 and 3
109. What do you understand by the (b) 2 only
term “Buoyancy in tax” (c) 2 and 3
(a) Efficiency and responsiveness (d) 1, 2 and 3
of revenue mobilization in 113. Vaippar river is a river in the
response to growth in the Indian state of
Gross domestic product or (a) Karnataka
National income. (b) Tamil Nadu
(b) Effect on a decrease in tax (c) Kerala
rates on tax collection (d) Andhra Pradesh
(c) Streamlining of various taxes 114. With reference to BHIM, consider
under one Tax such as GST the following statements
(d) None of the above 1. It was developed by RBI
110. In India, which of the following is 2. BHIM named after B R
having highest share in the Ambedkar
disbursement of credit to 3. BHIM allow users to send or
agriculture and allied activities? receive money to non-UPI
(a) Cooperative Banks based accounts
(b) Regional Rural Banks Which of the given above
(c) Commercial Banks statements is/are correct?
(d) Microfinance institutions (a) 1 and 3
(b) 2 only
111. With reference Right to (c) 2 and 3
Information Act in India, consider (d) 1, 2 and 3
the following statements 115. Principle of Indicative Planning
1. Under the act, Chief Justice of was adopted for the first- time in
India is a public authority the

Telegram: https://t.me/insightsIAStips
20
Youtube: https://www.youtube.com/channel/UCpoccbCX9GEIwaiIe4HLjwA
Revision Through MCQs (RTM) Compilation (November 2019)

(a) 6th five year plan 1. The accord provided for


(b) 7th five year plan expulsion of all foreigners, who
(c) 8th five year plan entered Assam after 1965
(d) 9th five year plan 2. Clause 6 of the Accord provides
116. Which of the following is the for the protection and
main objectives of the 12th Five promotion of Assamese culture
Year Plan? Which of the given above
(a) Inclusive growth and poverty statements is/are correct?
reduction (a) 1 only
(b) Inclusive and sustainable (b) 2 only
growth (c) Both 1 and 2
(c) Faster, sustainable and more (d) Neither 1 nor 2
inclusive growth 120. Which of the following are in line
(d) Sustainable and inclusive with the larger ideology of
growth to reduce poverty and ‘Pacifism’?
unemployment (a) Opposition to war or violence
117. With reference to Disputes raised as a means of settling disputes
by India in WTO forum, consider (b) Sticking to principle rather
the following statements than pragmatism
1. Certain measures on imports of (c) Aversion to modern nation-
iron and steel states military institutions and
2. Countervailing duty by United their intrusion in other nations
States on Indian steel products (d) Both (a) and (c)
3. United States Sub-Federal 121. With reference to Tribunal’s in
Renewable energy programme India, Consider the following
Which of the given above statements
statements is /are correct? 1. Article 323-A of the constitution
(a) 1 and 2 only provides for the establishment
(b) 1 and 3 only of administrative tribunals by a
(c) 2 and 3 only parliament law
(d) 1, 2 and 3 2. Administrative tribunals are
118. With reference to UNNATI constitutional body
Initiative, consider the following 3. Constitutional Status to the
statements Tribunals were given by the
1. It is an initiative of ISRO 42nd (amendment) act, 1976
2. It is open only for developing Which of the given above
countries statements is/are correct?
3. It provides opportunities to (a) 1 and 2
strengthen in assembling, (b) 1 and 3
integrating and testing of (c) 1 only
Nanosatellite (d) 2 and 3
Which of the given above 122. Money Bill is defined by article _
statements is/are correct? of the Indian Constitution
(a) 1 and 2 only (a) Article 108
(b) 1 and 3 only (b) Article 110
(c) 2 and 3 only (c) Article 111
(d) 1, 2 and 3 (d) Article 112
119. With reference to Assam Accord 123. Recently ‘Nadu-Nedu’ programme
of 1985, consider the following has been in news sometimes is
statements launched by
(a) Telangana
(b) Andra Pradesh

Telegram: https://t.me/insightsIAStips
21
Youtube: https://www.youtube.com/channel/UCpoccbCX9GEIwaiIe4HLjwA
Revision Through MCQs (RTM) Compilation (November 2019)

(c) Karnataka resource such as the Moon or a


(d) Tamil Nadu planet
124. The government cannot impose 4. It allows state parties to hold
restrictions on the practice of bilateral or multilateral military
freedom of religion on which of the exercises in space or on
following grounds? celestial bodies
(a) Public order 5. As per the treaty, if a State
(b) Public health launches a space object, and
(c) Morality retains jurisdiction and control
(d) National Integrity over that object, it is also liable
125. Consider the following for damages caused by their
statements about Surang Bawadi space object
1. It is an important Karez system Which of the given above
was built by Bahmani kings statements is/are correct?
2. Suranga Bawadi used for rapid (a) 3 and 5 only
mobilization of soldiers (b) 1, 2, 3, 4 and 5
Which of the given above (c) 2, 3 and 5 only
statements is /are correct? (d) 1, 4 and 5 only
(a) 1 only 128. It is a compulsory termination of
(b) 2 only Indian citizenship by the Central
(c) Both 1 and 2 government, if
(d) Neither 1 nor 2 1. The citizen has shown
126. With reference to Coalition for disloyalty to the Constitution of
Disaster Resilient Infrastructure, India
consider the following statements 2. The citizen has been ordinarily
1. It is an international coalition resident out of India for four
of countries, private sector and years continuously
academic institutions Which of the given above
2. The headquarters of CDRI statement is/are correct?
located in New Delhi (a) 1 only
3. It was launched by the Indian (b) 2 only
Prime Minister at the 2019 UN (c) Both 1 and 2
Climate Action Summit (d) Neither 1 Nor 2
Which of the given above 129. The major purpose of the IndIGO
statements is/are correct? consortium, with support from the
(a) 1 and 3 USA, is to
(b) 3 only 1. Set up the LIGO-India detector,
(c) 1 and 2 which would help enhance the
(d) 1, 2 and 3 network of gravitational wave
127. With reference to the Outer Space detectors worldwide
Treaty, its obligations and 2. Make findings about cosmic
provisions, consider the following radiation widely available so
statements. that it strengthens research
1. It prohibits the launching of synergy in theoretical physics
ballistic missiles, which could Which of the given above
be armed with WMD warheads statements is/are correct?
through space (a) 1 only
2. China is not a party to the (b) 2 only
treaty (c) Both 1 and 2
3. The treaty explicitly forbids any (d) Neither 1 Nor 2
government to claim a celestial 130. The Constitution confers which
of the following rights and

Telegram: https://t.me/insightsIAStips
22
Youtube: https://www.youtube.com/channel/UCpoccbCX9GEIwaiIe4HLjwA
Revision Through MCQs (RTM) Compilation (November 2019)

privileges on the citizens of India 1. NISHTHA is the world’s largest


but denies the same to aliens? teachers’ training programme
1. Right to education 2. As per the Right to Education
2. Right to freedom of speech and (RTE) Act, to be eligible for
expression appointment as a teacher in a
3. Right against exploitation State, a person would be
Select the correct answer using the required to possess the
codes below. minimum qualification laid
(a) 2 and 3 only down by the concerned State
(b) 1 and 3 only Council of Teacher Education.
(c) 2 only 3. As per the RTE Act, for
(d) 1, 2 and 3 teaching primary classes, a
131. With reference to Indian candidate is required to pass a
Judiciary, consider the following Teacher Eligibility Test
statements conducted in accordance with
1. Articles 125 and 219 give the the National Council of Teacher
power of contempt of court to Education guidelines.
the higher judiciary 4. In India, more than 90% of
2. Contempt of Courts Act, 1971, teacher education institutions
clearly states that fair criticism are directly under the State
of any case which has been Governments.
heard and decided is not Which of the statements given
contempt above is/are correct?
Which of the given above (a) 2, 3 and 4
statements is/are correct? (b) 1 and 3
(a) 1 Only (c) 1, 2 and 4
(b) 2 Only (d) 1, 3 and 4
(c) Both 1 and 2 136. Recently which of the following
(d) Neither 1 Nor 2 has set up an Online Child Sexual
132. Which of the following acts as Abuse and Exploitation (OCSAE)
Insolvency regulator in India? Prevention/Investigation Unit?
(a) NCLT (a) CBI
(b) IBBI (b) MHA
(c) RBI (c) MWCD
(d) MoF (d) Delhi Police
133. Which of the following is 137. Consider the following
responsible for refinance support statements about BRICS forum.
and inspection of Co-operative 1. The Chairmanship of the forum
Banks in India? is rotated annually among the
(a) RBI members, in accordance with
(b) NABARD the country that is hosting the
(c) State Government conference.
(d) Union Government 2. BRICS forum is well known to
134. Mukhyamantri Nishulk Dava engage all “tracks” of
Yojana has been in news for interaction such as Track I,
sometimes is launched by Track II and Track III.
(a) Rajasthan Which of the given above
(b) Odisha statements is/are correct?
(c) Uttara Pradesh (a) 1 only
(d) Delhi (b) 2 only
135. Consider the following (c) Both 1 and 2
statements (d) Neither 1 Nor 2

Telegram: https://t.me/insightsIAStips
23
Youtube: https://www.youtube.com/channel/UCpoccbCX9GEIwaiIe4HLjwA
Revision Through MCQs (RTM) Compilation (November 2019)

138. Consider the following Initiative, consider the following


statements. statements
1. UNISPACE+50 is an initiative of 1. Program is launched by MWCD
the G-77 to coordinate to control Pneumonia
effectively in the domain of 2. It is a central sector scheme
satellite supervision and 3. Under the scheme, child
disaster management. suffering from Pneumonia will
2. The Space 2030 agenda of the be treated with pre-referral
U.N. will map out how dose of Amoxicillin by ASHA
spaceflight and space-related 4. The initiative also aims to
activities can help achieve the create mass awareness about
Sustainable Development Goals the most effective solutions for
(SDGs). pneumonia prevention like
Which of the given above breast feeding, immunization,
statements is/are correct? good quality air
(a) 1 only Which of the given above
(b) 2 only statements is/are correct?
(c) Both 1 and 2 (a) 1, 3 and 4
(d) Neither 1 Nor 2 (b) 3 and 4
139. The kingdom of Khurda is mainly (c) 2, 3 and 4
associated to which state (d) 1, 2, 3 and 4
(a) Maharashtra 144. Consider the following
(b) Bihar statements:
(c) Odisha 1. India have bilateral free trade
(d) West Bengal agreement with all members of
140. Ease of Living Index is released ASEAN nations
by 2. India has signed bilateral FTA
(a) Niti Ayog with Sri Lanka, Bhutan, Nepal
(b) World Bank and Bangladesh
(c) Ministry of Rural Development Which of the given above
(d) Ministry of Housing and Urban statements is/are correct?
Affairs (a) 1 only
141. ‘England’s Debt to India’ and (b) 2 only
‘India’s Will to Freedom’ are (c) Both 1 and 2
important works of (d) Neither 1 Nor 2
(a) Veer Savarkar 145. ADMM-Plus meeting has been in
(b) Dadabhai Naoroji news for sometimes is related to
(c) Ramesh Dutt (a) ASEAN
(d) None of the above (b) SAARC
142. He wrote biographies of Mazzini, (c) RCEP
Garibaldi, Shivaji and Shrikrishna; (d) SCO
stayed in America for some time; 146. With reference to Bureau of
and was also elected to the Central Indian standards, consider the
Assembly. He was following statements
(a) Aurobindo Ghosh 1. BIS is the national Standards
(b) Bipin Chandra Pal Body of India working under
(c) Lala Lajpat Rai the aegis of MoC&I
(d) Motilal Nehru 2. BIS head will be appointed by
143. With reference to Social Cabinet secretary
Awareness and Action to Neutralise 3. BIS is a statutory body
Pneumonia Successfully (SAANS) Which of the given above
statements is/are not correct?

Telegram: https://t.me/insightsIAStips
24
Youtube: https://www.youtube.com/channel/UCpoccbCX9GEIwaiIe4HLjwA
Revision Through MCQs (RTM) Compilation (November 2019)

(a) 1 and 2 (c) 1, 2 and 4 only


(b) 2 only (d) 3 and 4 only
(c) 1 and 3 only 150. Due to some reasons, if there is a
(d) All of the above huge fall in the population of
147. Consider the following species of butterflies, what could
statements: be its likely
1. The Standard Mark of Bureau consequence/consequences?
of Indian Standards (BIS) is 1. Pollination of some plants
mandatory for automotive tyres could be adversely affected.
and tubes. 2. There could be a drastic
2. AGMARK is a quality increase in the fungal
Certification Mark issues by infections of some cultivated
the Food and Agriculture plants.
Organisation (FAO). 3. It could lead to a fall in the
Which of the statements given population of some species of
above is/are correct? wasps, spiders and birds.
(a) 1only Select the correct using the code
(b) 2 only given below:
(c) Both 1 and 2 (a) 1 only
(d) Neither 1 nor 2 (b) 2 and 3 only
148. Which of the following Oil (c) 1 and 3 only
Refinery/Refinery’s located in (d) 1, 2 and 3
Assam? 151. The Parliament of India acquires
1. Bongaigaon Refinery the power to legislate on any item
2. Haldia Refinery in the State List in the national
3. Guwahati Refinery interest if a resolution to that
4. Barauni Refinery effect is passed by the
Select the correct answer using the (a) Lok Sabha by a simple majority
code below of its total membership
(a) 3 only (b) Lok Sabha by a majority of not
(b) 1 and 3 only less than two thirds of its total
(c) 2, 3 and 4 only membership
(d) All of the above (c) Rajya Sabha by a simple
149. Which of the following are the majority of its total
objectives of ‘National Nutrition membership
Mission’? (d) Raja Sabha by a majority of not
1. To create awareness relating to less than two thirds of its
malnutrition among pregnant members present and voting
women and lactating mothers. 152. Consider the following
2. To reduce the incidence of statements:
anaemia among young 1. The Rajya Sabha has no power
children, adolescent girls and either to reject or to amend a
women. Money Bill
3. To promote the consumption of 2. The Rajya Sabha cannot vote
millets, coarse cereals and on the Demands for Grants
unpolished rice. 3. The Rajya Sabha cannot
4. To promote the consumption of discuss the Annual Financial
poultry eggs. Statement
Select the correct answer using the Which of the statements given
code given below: above is/are correct?
(a) 1 and 2 only (a) 1 only
(b) 1, 2 and 3 only (b) 1 and 2 only

Telegram: https://t.me/insightsIAStips
25
Youtube: https://www.youtube.com/channel/UCpoccbCX9GEIwaiIe4HLjwA
Revision Through MCQs (RTM) Compilation (November 2019)

(c) 2 and 3 only (c) Quacquarelli Symonds


(d) 1, 2 and 3 Limited(QS)
153. With reference to Gram (d) None of the above
Panchayats, consider the following 157. Za’ir-Al-Bahr (the Roar of the
statements Sea)- It is the Joint Exercise
1. Term Gram Sabha is defined in between the Indian Navy and
the Constitution of India under (a) Qatari Navy
Article 243 (b) Royal Navy of Oman
2. The power to annul a decision (c) Royal Saudi Navy
of the Gram Sabha rests with (d) Yemeni Navy
the Gram Sabha only 158. Consider the following
3. According to the State statements regarding Pradhan
Panchayat Raj Acts, the Gram mantri awas yojana gramin
Sabha must meet at least two 1. Scheme implemented in rural
to four times in a year areas throughout India
Which of the given above 2. Beneficiary entitled to 100 days
statements is/are correct? of unskilled labour under
(a) 1 and 2 only MGNREGA
(b) 3 only 3. Beneficiary can avail a loan of
(c) 2 and 3 only Rs.70000/- for construction of
(d) 1, 2 and 3 house
154. Pradhan Mantri Matru Vandana 4. Unit size is 25 sq mts
Yojana (PMMVY) is a conditional Select the correct answer using the
cash transfer scheme for pregnant code below:
and lactating women of 19 years of (a) 1, 2 and 3 only
age or above for the first live birth. (b) 2 and 3 only
The scheme falls under the ages of (c) 3 and 4 only
(a) MWCD (d) All of the above
(b) MoHFW 159. Consider the following with
(c) MoSJE respect to DNA technology bill
(d) Both A and B 1. Bill provides DNA profiling for
155. Recently Kimberley Process has civil and criminal disputes
been in news for sometimes is 2. Written consent of individual is
related to needed to collect DNA samples
(a) It is an international in all cases
certification scheme that 3. Seeks to establish regional DNA
regulates trade in rough data bank one for each state
diamonds only
(b) It is an international Select the correct answer using the
certification scheme that code below:
regulates trade in Brent Crude (a) 1 and 2 only
Oil (b) 2 and 3 only
(c) It is an international (c) 1 only
certification scheme that (d) All of the above
regulates trade in 24 carat Gold 160. Sepahijala Wildlife Sanctuary is
(d) Both A and C in which state
156. The 2019 World Talent Ranking (a) Assam
has been released by (b) Meghalaya
(a) World Economic Forum (c) Arunachal Pradesh
(b) World Intellectual Property (d) Tripura
Organization

Telegram: https://t.me/insightsIAStips
26
Youtube: https://www.youtube.com/channel/UCpoccbCX9GEIwaiIe4HLjwA
Revision Through MCQs (RTM) Compilation (November 2019)

161. With reference to Indian Freedom (b) Humid sub-Tropical Monsoon


Struggle, consider the following Climate
statements (c) Dry sub-Tropical Monsoon
1. Rani Laksmi Bhai rapidly Climate
organized her troops and (d) Dry sub-Temperate Monsoon
assumed charge of the rebels in Climate
the Bundelkhand region 164. Article 371(G) of the Constitution
2. Rani Laksmi Bhai died while states that the Parliament cannot
fighting a squadron of the 8th decide on the matters of the
Hussars under Gen. Hugh Rose religious and social practices of the
Which of the given above (a) Mizos Tribe
statement is/are correct? (b) Naga Tribe
(a) 1 only (c) Garo Tribe
(b) 2 only (d) None of the above
(c) Both 1 and 2 165. “The term entered English in the
(d) Neither 1 Nor 2 late 15th century from French, it
162. Consider the following has been described as the
statements "crossroads of western Asia, the
1. Plague is a disease caused by eastern Mediterranean, and
the virus northeast Africa also region is an
2. Plague can be transmitted to approximate historical geographical
humans through sexual term referring to a large area in the
contact Eastern Mediterranean, primarily in
3. There is no treatment available Western Asia. In its narrowest
to cure Plague sense, it is equivalent to the
Which of the given above historical region of Syria.”
statements is/are not correct? The above passage describes
(a) 3 only which of the following geographical
(b) 2 and 3 only region?
(c) 1 and 3 only (a) West bank
(d) 1, 2 and 3 (b) Levant
163. Consider the following (c) Transcaucasia
temperature and rainfall data: (d) Sinai Peninsula
166. The term "two-state solution" is
sometimes mentioned in the news
in the context of the affairs of
(a) China
(b) Israel
(c) Greece
(d) Yemen
167. Xinjiang shares borders with
1. Mongolia
2. Russia
3. Kazakhstan
4. Nepal
5. Afghanistan
6. Pakistan
These are most likely to be found 7. India
in the natural regions of: Select the correct answer using the
(a) Humid sub-Temperate below code:
Monsoon Climate (a) 1, 2, 3, 4 and 6
(b) 1, 2, 3, 5 and 6

Telegram: https://t.me/insightsIAStips
27
Youtube: https://www.youtube.com/channel/UCpoccbCX9GEIwaiIe4HLjwA
Revision Through MCQs (RTM) Compilation (November 2019)

(c) 2, 3, 4, 5 and 6 2. IONS is a voluntary initiative


(d) 1, 2, 3, 5, 6 and 7 that seeks to increase maritime
168. Consider the following co-operation among navy’s of
statements the littoral states of the Indian
1. Deposit Insurance and Credit Ocean Region.
Guarantee Corporation Which of the above statements
(DICGC) under IRDAI is/are correct?
2. In India, every insured bank (a) 1 only
pays premium amounting to (b) 2 only
10% of its deposits to DICGC (c) Both 1 and 2
every year (d) Neither 1 nor 2
3. When a bank is liquidated, 171. With reference to National
depositors are entitled to Register of Citizens (NRC), consider
receive an insurance amount of the following statements
₹1 lakh per individual from 1. The register was first prepared
DICGC after the 9th in a series of
Which of the given above census held in India
statements is/are correct? 2. One of the eligibility criteria
(a) 3 only under NRC, D-Voters can apply
(b) 2 and 3 only for inclusion of their names in
(c) 1 only the updated NRC only when
(d) 1 and 2 only the appropriate Foreigner
169. Consider the following Tribunal declares them as non-
statements with respect to foreigners
NATGRID 3. Initially, NRC was specifically
1. NATGRID is a post Mumbai made for a state, which was
26/11 attack measure famous for Shirui Lily and
2. NATGRID, like a number of Cheiraoba festival
other government initiatives Which of the given above
(UIDAI), established through statements is/are not correct?
governmental notifications (a) 1 only
rather than legislation passed (b) 2 only
in Parliament (c) 3 only
3. Initially, no state agencies will (d) 1 and 3 only
be given direct access to the 172. The Hong Kong International
NATGRID data Convention for the Safe and
4. The data recovery centre of the Environmentally Sound Recycling of
NATGRID has been constructed Ships, 2009 (the Hong Kong
in Bengaluru Convention), was adopted by
Which of the given above (a) Directorate General of Shipping
statements is/are correct? (b) International Association of
(a) 1, 2 and 4 Ports and Harbors
(b) 1 and 4 (c) World Association for
(c) 1, 2 and 3 Waterborne Transport
(d) 1, 2, 3 and 4 Infrastructure
170. Consider the following in respect (d) None of the above
of Indian Ocean Naval Symposium 173. Arrange the following events in
(IONS): chronological order
1. Inaugural IONS was held in 1. Montagu-Chelmsford Report
India in 2015 under the 2. Jallianwala Bagh massacre
chairmanship of the Indian 3. Rowlatt Acts
Navy.

Telegram: https://t.me/insightsIAStips
28
Youtube: https://www.youtube.com/channel/UCpoccbCX9GEIwaiIe4HLjwA
Revision Through MCQs (RTM) Compilation (November 2019)

4. Rabindranath Tagore renounced 3. Allowing crop residue to remain


the knighthood in the field
Select the correct answer using the Select the correct answer using the
code below: code given below:
(a) 1-2-3-4 (a) 1 and 2 only
(b) 3-1-2-4 (b) 3 only
(c) 3-2-1-4 (c) 1 and 3 only
(d) 1-3-2-4 (d) 1, 2 and 3
174. Code on Wages, 2019 will 178. What is the application of
amalgamate Somatic Cell Nuclear Transfer
1. Payment of Wages Act, 1936 Technology?
2. Minimum Wages Act, 1948 (a) Production of biolarvicides
3. Payment of Bonus Act, 1965 (b) Manufacture of biodegradable
4. Payment of Gratuity Act 1972 plastics
Select the correct answer using the (c) Reproductive cloning of
code below: animals
(a) 1, 2 and 3 (d) Production of organisms free of
(b) 1, 3 and 4 diseases
(c) 2, 3 and 4 179. Ministry of Earth Sciences has
(d) All of the above unveiled state-of-the-art Air
175. Recently 3S programme has been Quality and Weather Forecast
in news for sometimes is primarily System–SAFAR (System of Air
related to Quality and Weather Forecasting).
(a) Banking reform’s It covers which of the following
(b) Skill development pollutants?
(c) Data management 1. PM2.5 and PM10
(d) Public Healthcare 2. Ozone
3. Carbon dioxide
4. Methane
Select the correct answer using the
176. Consider the following codes below.
statements (a) 1, 2 and 3 only
1. National Institute of Sowa-Rigpa (b) 1, 2, 3 and 4
(NISR) is autonomous national (c) 1, 2 and 4 only
institute under MoH&FW (d) 1 and 4 only
2. Sowa –Rigpa is a traditional 180. The Rajya Sabha does not have a
system of medicine originated procedure for moving of
in Bhutan 1. Adjournment motion
Which of the given above 2. Censure motion
statements is/are correct? 3. No-confidence motion
(a) 1 only Select the correct answer using the
(b) 2 only codes below.
(c) Both1 and 2 (a) 1 only
(d) Neither 1 Nor 2 (b) 2 and 3 only
177. Which of the following (c) 1, 2 and 3
statements can help in water (d) 1 and 3 only
conservation in agriculture?
1. Reduced or zero tillage of the 181. “Rights of Citizenship of Certain
land Persons of Indian Origin Residing
2. Applying gypsum before Outside India” has been enshrined
irrigating the field in the constitution of India under
the article

Telegram: https://t.me/insightsIAStips
29
Youtube: https://www.youtube.com/channel/UCpoccbCX9GEIwaiIe4HLjwA
Revision Through MCQs (RTM) Compilation (November 2019)

(a) Article 5 (d) 3 only


(b) Article 6 186. Consider the following
(c) Article 7 statements regarding Convention
(d) Article 8 on Cybercrime
182. With reference to K12 Education 1. It is the first international
Transformation Framework, treaty seeking to address
consider the following statements Internet and computer crime
1. It was rolled out by Microsoft 2. It is open for ratification even
with intention to improve high to states that are not members
school education of the Council of Europe
2. It comprises four pillars 3. It was drawn up International
strategy including parents Telecommunications Union
Which of the given above (ITU)
statements is/are not correct? Which of the given above
(a) 1 only statements is/are correct?
(b) 2 only (a) 1 and 2
(c) Both 1 and 2 (b) 1 only
(d) Neither 1 Nor 2 (c) 2 and 3
183. Recently Arundhati Swarna (d) 3 only
Yojana has been rolled out by 187. Which of the following are
(a) State of Assam correctly matched
(b) State of Gujarat 1. Prithvi-II missile- Short-range
(c) State of Maharashtra ballistic missile
(d) State of Karnataka 2. Agni missiles- Medium to
184. IMI 2.0 has been in news for Intercontinental range ballistic
sometimes is primarily related to missiles
(a) Restructuring of stressed 3. K family of missiles -
assets in banking sector Submarine-launched ballistic
(b) Debt swap agreement with missiles
Japan Select the correct answer using the
(c) Renaissance of ISIS terrorist code below:
group (a) 1 and 2 only
(d) Public Health care (b) 2 and 3 only
185. With reference to Avian botulism, (c) 1, 2 and 3
consider the following statements (d) 2 only
1. It refers to strain of botulism 188. With reference to Bacteriophage,
that affects wild and captive consider the following statements
bird populations, most notably 1. A bacteriophage is a type of
waterfowl virus that infects bacteria
2. Humans are generally 2. All bacteriophages are
considered resistant to type E composed of a nucleic acid
avian botulism molecule that is surrounded by
3. It is a paralytic disease brought a protein structure
on by the the Botulinum 3. Bacteriophages occasionally
neurotoxin (BoNt) of the remove a portion of their host
bacterium Clostridium cells' bacterial DNA during the
botulinum infection process and then
Which of the given above transfer this DNA into the
statements is/are correct? genome of new host cells, the
(a) 1 and 3 process is known as
(b) 1 only transcription
(c) 2 and 3

Telegram: https://t.me/insightsIAStips
30
Youtube: https://www.youtube.com/channel/UCpoccbCX9GEIwaiIe4HLjwA
Revision Through MCQs (RTM) Compilation (November 2019)

Which of the given above Which of the give above statements


statements is/are correct? is/are correct?
(a) 1 and 3 (a) 3 only
(b) 1 and 2 (b) 2 and 3 only
(c) 1 only (c) 1 and 3 only
(d) 1, 2 and 3 (d) 1, 2 and 3
189. Western Asia is located directly 192. With reference to GST Council &
south of Eastern Europe. The its 36th meeting, consider the
region is surrounded by seven following statements
major seas which include? 1. GST rate on plates and cups
1. Black Sea made of flowers, leaves and
2. Red Sea bark reduced from 12% to 5 %
3. Aegean Sea 2. There is no GST payable on
4. Caspian Sea tamarind
Select the correct answer using the 3. 18% GST rate payable on
codes below. Caffeinated Beverages
(a) 1 and 2 only 4. Woven/ Non-woven
(b) 1, 2 and 4 only Polyethylene Packaging bags
(c) 3 and 4 only attracts 12% of GST rate
(d) 1, 2, 3 and 4 Select the correct answer using the
190. Consider a triangle that is code below:
formed from the following; (a) 1, 2 and 3
1. Southernmost point of (b) 2 and 4
Mediterranean Sea (c) 1, 3 and 4
2. Northernmost point of Black (d) 3 and 4
Sea 193. Which of the following
3. Southernmost point of Caspian statements is/are correct?
Sea 1. State of Jammu and Kashmir
Which of these nations would fall joined GST earlier this year
partially or fully in the triangle? 2. Under the GST law, Delhi and
(1) Syria (2) Turkey (3) Iran (4) Pondicherry have State
Kuwait Legislatures, they are treated
Select the correct answer using the as States under GST
codes below. 3. GSTC is a constitutional body
(a) 1 only 4. Under GST law, Centre will
(b) 1, 2 and 3 have one-third weightage of the
(c) 2 and 3 only total votes cast and the States
(d) 1, 2, 3 and 4 together will have two-thirds
191. With reference to Indian weightage of the total votes
Parliament, consider the following cast. Passing of a resolution
statements requires the support of at least
1. Any MP who is not a member of three-fourth weightage of the
lok sabha is referred to as a total votes cast
private member Select the correct answer suing the
2. Admissibility of a private code below:
member’s bill is the sole (a) 1 and 3
prerogative of speaker (b) 2, 3 and 4
3. While government Bills can be (c) 1, 3 and 4
introduced and discussed on (d) 3 and 4 only
any day, private member’s Bills 194. Consider the following
can be introduced and statements related to One Stop
discussed only on Fridays Centre (OSC) scheme

Telegram: https://t.me/insightsIAStips
31
Youtube: https://www.youtube.com/channel/UCpoccbCX9GEIwaiIe4HLjwA
Revision Through MCQs (RTM) Compilation (November 2019)

1. It is a Centrally Sponsored 198. Development expenditure of the


Scheme central government includes;
2. It falls under the MHA 1. Expenditure on social and
3. OSC will support all women community services
excluding girls below 18 years 2. Defence expenditure
of age affected by violence, 3. Grants to states
irrespective of caste, class, 4. Expenditure on economic
religion, region, sexual services
orientation or marital status Select the correct answer using the
Which of the given above codes below;
statements is/are correct? (a) 1, 3 and 4 only
(a) 1 only (b) 1 and 3 only
(b) 1 and 2 (c) 1 only
(c) 1 and 3 (d) 1, 2, 3 and 4
(d) 1, 2 and 3 199. Which of these seas is NOT
195. What are the issues associated included in the China Seas?
with Reverse Osmosis Plants (a) The Yellow Sea
1. Deposition of brine along the (b) The East China Sea
shores (c) The South China Sea
2. Reduced membrane (d) The Cametos Sea
performance by fouling 200. The mercantilist theory of
3. It can significantly reduce good Economics growth is primarily
minerals that can aid in overall concerned with an increase in the;
heart and muscle health (a) Volume of money in circulation
Which of the given above through import of gold
statements is/are correct? (b) Size of population
(a) 1 and 3 (c) State intervention in the
(b) 1 only economy
(c) 1 and 2 (d) Surplus in the balance of trade
(d) 1, 2 and 3
196. Recently Living root bridges are 201. Consider the following
in news for sometimes related to statements
(a) They are handmade from the 1. Judicial review shall be
aerial roots of Ficus elastic applicable to the proclamation of
trees the president calling for
(b) They are handmade from the President’s rule in a state.
aerial roots of Ocimum 2. Relevant material is necessary
tenuiflorum for the satisfaction of the
(c) They are handmade from the president to apply president’s
aerial roots of Azadirachta rule. The satisfaction of the
indica President must be based on
(d) They are handmade from the relevant material. In case the
aerial roots of Mangifera indica court find out that the
197. Common Services Centers (CSCs) President’s action was based on
are a strategic cornerstone of extraneous grounds, it could be
which of the following flagship struck down or declared mala
Programmes of the Government of fide.
India? 3. The centre shall bear the burden
(a) MGNREGA of proving that sufficient and
(b) Digital India programme relevant materials exist for
(c) National Nutrition Mission justification of imposition of
(d) Sarva Shiksha Abhiyan (SSA) President’s Rule.

Telegram: https://t.me/insightsIAStips
32
Youtube: https://www.youtube.com/channel/UCpoccbCX9GEIwaiIe4HLjwA
Revision Through MCQs (RTM) Compilation (November 2019)

In which of the following case storage wetland principally used


Supreme Court delivered above for effectively harnessing water
mentioned verdict? resources in the states of Haryana,
(a) S. R. Bommai case of 1994 Rajasthan, Punjab, and western
(b) Kihoto Hollohan case of 1992 Uttar Pradesh of North India, that
(c) Waman Rao case of 1981 collects and stores water
(d) Indira Gandhi case of 1975 throughout the year, to be used for
202. Which of the following article the purpose of recharging the
enables the Supreme Court to groundwater in the nearby water
review its own judgments, subject wells, washing, bathing and
to the provisions of any law made drinking by humans and cattle”
by Parliament? The above given passage describes,
(a) 137 which of the following rain water
(b) 145 harvesting system?
(c) 142 (a) Suranga Bawadi
(d) 138 (b) Johad
203. Every person including the (c) Kere
disabled has his life and liberty (d) Diggi
guaranteed under 207. Which of the following rhino is
(a) Article 15 the smallest of the five extant
(b) Article 19 rhino species in the world?
(c) Article 20 (a) White Rhino
(d) Article 21 (b) Black Rhino
204. With reference to Exchange (c) Greater One-Horned
Traded Funds, consider the Rhinoceros
following statements (d) None of the above
1. Unlike regular mutual funds,
an ETF trades like a common 208. If you travel by road from Leh
stock on a stock exchange (Ladakh) to Yanam (Puducherry),
2. Traded prices of ETF don’t what is the minimum number of
change throughout the day like states within India through which
any other stock you can travel, excluding origin and
3. Trading value of an ETF is destination?
based on the net asset value of (a) 4
the underlying stocks that an (b) 5
ETF represents (c) 6
Which of the above given (d) 7
statements is/are correct? 209. P-Series Fuels are a blend of
(a) 1 and 3 only 1. Ethanol
(b) 3 only 2. Natural gas liquids
(c) 1 and 2 only 3. Methyl tetrahydrofuran
(d) 2 and 3 only (MeTHF)
205. Construction of Daudhan dam 4. Biodiesel
will result into submergence of Select the correct answer using the
10% of critical tiger habitat of codes below.
MP’s. It is referring to (a) 1 and 2 only
(a) Kanha Tiger Reserve (b) 3 only
(b) Pench Tiger Reserve (c) 2, 3 and 4 only
(c) Panna Tiger Reserve (d) 1, 2 and 3 only
(d) Betla Tiger Reserve 210. Consider the following
206. “It is a community-owned statements.
traditional harvested rainwater

Telegram: https://t.me/insightsIAStips
33
Youtube: https://www.youtube.com/channel/UCpoccbCX9GEIwaiIe4HLjwA
Revision Through MCQs (RTM) Compilation (November 2019)

1. Common Risk Mitigation Procedure and the Conduct of


Mechanism (CRMM), established its business
under the Sendai framework, 2. Rules of Procedure and
intends to enlarge the total size Conduct of Business in the
of the renewable energy market House of the People were
by making investments in amended by the Speaker from
renewables widely available to time to time on the
institutional investors. recommendations of the
2. The Cities Climate Finance subordinate legislation
Leadership Alliance’s mission committee of the House until
aims to catalyse investment in 1954
low-carbon and climate- Which of the above given
resilient infrastructure in cities statements is/are correct?
across the World. (a) 1 only
3. Climate Finance Leadership (b) 2 only
Initiative works to fulfill the (c) Both 1 and 2
private financing objectives (d) Neither 1 Nor 2
included in the 2015 Paris 213. “This Committee consists of 15
Climate Change Agreement. members including the Speaker
Select the correct answer using the who is the ex-officio Chairman of
codes below. the Committee. The members are
(a) 1 and 3 only nominated by the Speaker. The
(b) 2 and 3 only Committee considers matters of
(c) 2 only procedure and conduct of business
(d) 1, 2 and 3 in the House and recommends any
211. Consider the following amendments or additions to the
statements Rules of Procedure and Conduct of
1. Samvidhan Divas is celebrated Business in Lok Sabha that are
every year on November 26 to considered necessary”
mark the day on which the The above given passage describes
Constitution of India was which of the following
adopted parliamentary committee?
2. Former Prime Minister of India (a) Business Advisory Committee
Manmohan Singh announced (b) Committee on Papers Laid on
that 26 November will be the Table
celebrated every year as (c) Committee on Subordinate
"Constitution Day" Legislation
Which of the above given (d) Rules Committee
statements is/are correct? 214. With reference to Credit-linked
(a) 1 only Subsidy Services Awas Portal,
(b) 2 only which of the following are its
(c) Both 1 and 2 significance
(d) Neither 1 Nor 2 1. The scheme will expand
212. Consider the following institutional credit flow to the
statements with reference to Rules housing needs of urban poor
of Procedure and Conduct of 2. Interest subsidy will be credited
Business in parliament upfront to the loan account of
1. Article 118(1) of the beneficiaries through Primary
Constitution empowers each Lending Institutions
House of Parliament to make 3. Credit linked subsidy will be
rules for regulating its available only for loan amounts
upto Rs 6 lakhs

Telegram: https://t.me/insightsIAStips
34
Youtube: https://www.youtube.com/channel/UCpoccbCX9GEIwaiIe4HLjwA
Revision Through MCQs (RTM) Compilation (November 2019)

4. HUDCO is the Central Nodal 218. The largest number of Hindu


Agency (CNA) to channelize this temples under the ASI’s protection
subsidy to the lending is in
institutions (a) Karnataka
Which of the above given (b) Tamil Nadu
statements is/are correct? (c) Odisha
(a) 2 and 3 (d) Uttara Pradesh
(b) 2 only 219. “The 8.8 km long Rohtang Tunnel
(c) 1, 2 and 4 will cut through X. When complete,
(d) 1, 2, 3 and 4 it will become the world’s longest
215. With reference to Coalbed highway tunnel above 10,000 feet.
Methane (CBM), consider the The tunnel provides a temporary
following statements winter link to the outside world not
1. CBM is an conventional form of only to residents of Lahaul and
natural gas found in coal Spiti and the tunnel will be one of
deposits or coal seams the longest road tunnels in India
2. CBM can be used as feedstock and is expected to reduce the
for fertilisers distance between Manali and
3. Best prospective areas for CBM Keylong by about 46 km.”
development are in eastern With reference to above passage,
India Identify the X from the following
Which of the above given given options:
statements is/are correct? (a) Karakoram Range
(a) 1 only (b) Pir Panjal Range
(b) 2 and 3 (c) Aravali Range
(c) 2 only (d) None of the above
(d) 1, 2 and 3 220. Consider the following
statements.
1. Rajasthan is the first state to
216. Consider the following have India’s first wildlife
statements conservation reserve dedicated
1. largest consumers of coal in exclusively to the blackbuck
India are electricity generation 2. Blackbucks are native to the
2. Gondwana coal is of low Indian subcontinent that has
calorific value and high ash been classified as endangered
content (IUCN) in 2017.
3. India has the fourth-largest Which of the above statements
coal reserves in the world is/are correct?
Which of the above given (a) 1 only
statements is/are correct? (b) 2 only
(a) 1 only (c) Both 1 and 2
(b) 2 and 3 (d) Neither 1 Nor 2
(c) 1 and 2 221. With reference to National
(d) 1, 2 and 3 Remote Sensing Centre (NRSC),
217. What is common to the clans consider the following statements
known as Swarga-Aroi, Basumati- 1. NRSC has a data reception
Aroi, Ramsa-Aroi? station at Master Hassan
(a) Monpa 2. NRSC ground station acquire
(b) Apatani the earth observation data from
(c) Munda Indian as well as foreign remote
(d) None of the above sensing satellite

Telegram: https://t.me/insightsIAStips
35
Youtube: https://www.youtube.com/channel/UCpoccbCX9GEIwaiIe4HLjwA
Revision Through MCQs (RTM) Compilation (November 2019)

3. It monitor and analyze the 1. When normal maximum


Cartosat series satellite temperature of a station is less
transferred data than or equal to 400C
Which of the above given 2. When normal maximum
statements is/are correct? temperature of a station is
(a) 1 and 3 more than 400C
(b) 2 and 3 3. When actual maximum
(c) 3 only temperature remains 450C or
(d) 1 only more irrespective of normal
222. Consider the following maximum temperature, heat
statements about National Remote wave should be declared
Sensing Centre (NRSC) Select the correct answer using the
1. It is a part of Indian Space code given below.
Research Organization (ISRO) (a) 1 and 3
2. It is the focal point for (b) 1 and 2
distribution of remote sensing (c) 2 and 3
satellite data products in India (d) All of the above
and its neighboring countries 225. Recently pre historic rock
Select the correct answer using the paintings are found in the X village
code below: of Tamil Nadu, which resemble the
(a) 1 Only script found in Indus civilization
(b) 2 Only sites. Here X refers to
(c) Both 1 and 2 (a) Aluva
(d) Neither 1 nor 2 (b) Chengannur
223. In June-2019 IMD reported that (c) Kunnathunad
Churu (Rajastan) recorded the (d) Karikiyoor
highest temperature of 48.4°C due
to increasing heat wave in the 226. Oussudu Lake sometimes seen in
north-India. In the context of “Heat the news recently, located in
wave” consider the following (a) Andhra Pradesh
statements (b) Kerala
1. Heat waves form when a warm, (c) Karnataka
high pressure system stalls in a (d) Puducherry
region 227. Talley Valley Wild life sanctuary
2. Heat is able to persist if there located in
aren't rain and clouds to cool (a) Assam
things off (b) Arunachal Pradesh
3. Heat-trapping ability of cities is (c) Odisha
called urban heat island effect (d) Sikkim
4. According to IPCC 4th 228. “E-2020 initiative” has been in
Assessment Report, Global news for sometimes was launched
warming is causing more heat by
waves (a) WHO
Select the correct answer using the (b) UNFCCC
code given below. (c) UNDP
(a) 1 and 3 (d) WTO
(b) 1, 2 and 3 229. Which of the following is the first
(c) 2 and 3 Arab country to get FATF
(d) All of the above membership?
224. According to IMD, the criteria for (a) Qatar
consideration of “Heat Wave” is (b) Kuwait
(c) Iran

Telegram: https://t.me/insightsIAStips
36
Youtube: https://www.youtube.com/channel/UCpoccbCX9GEIwaiIe4HLjwA
Revision Through MCQs (RTM) Compilation (November 2019)

(d) Saudi Arabia (a) 1 only


230. Centrally-controlled Air Traffic (b) 3 only
Flow Management (C-AFTM) system (c) 1 and 3
has been in news for sometimes is (d) 2 and 3
primarily related to 234. The annual flagship Emissions
(a) Air pollution control Gap Report released by
management (a) German Watch
(b) Airspace and airport (b) UNEP
management (c) IPCC
(c) Weather control and (d) UNFCCC
management 235. With reference to SATAT
(d) None of the above initiative, consider the following
231. Consider the following statements
statements 1. It was launched with the main
1. Question Hour is the first hour objective of addressing
of a sitting session of India's environmental problems arising
Lok Sabha from stubble burning
2. Star question is a one to which 2. It falls under the ages of
member desires an oral answer MoEFCC
from minister 3. Under the initiative natural gas
3. Un starred question is one to plants are proposed to be set
which written answer is desired up mainly through independent
by the member from minster entrepreneurs
Which of the above given Which of the above given
statements is/are correct? statements is/are not correct?
(a) 1 only (a) 1 and 2
(b) 2 and 3 only (b) 2 only
(c) 1 and 2 only (c) 3 only
(d) 1, 2 and 3 (d) All of the above
232. Common But Differentiated 236. Consider the following
Responsibilities and Respective statements
Capability (CBDR-RC) and the 1. Bio-gas is produced naturally
annex classifications were codified through a process of aerobic
under decomposition from waste
(a) Montreal Protocol 2. Natural gas is a naturally
(b) Cancun Summit occurring hydrocarbon gas
(c) Kyoto Protocol mixture consisting primarily of
(d) Parris Summit methane
233. With reference to Contract for 3. Compressed Bio-Gas is exactly
Web, Consider the following similar to the commercially
statements available natural gas in its
1. Contract for web is an initiative composition and energy
of International potential
Telecommunication Union Which of the above given
under UN leadership statements is/are correct?
2. Contract is binding on all the (a) 1 and 3
agreed nations (b) 2 and 3
3. It is used for to create standard (c) 1 and 2
policy on web for the benefits of (d) All of the above
the world 237. Which of the following
Which of the above given statements is/are correct
statements is/are correct?

Telegram: https://t.me/insightsIAStips
37
Youtube: https://www.youtube.com/channel/UCpoccbCX9GEIwaiIe4HLjwA
Revision Through MCQs (RTM) Compilation (November 2019)

(a) HIM VIJAY Military Exercise (c) For training of unorganized


conducted to validate sector workers
operational capabilities of our (d) For training of state
combat formations government employees
(b) HIM VIJAY Military Exercise 242. Which of the following
was an all Arms exercise organization has launched
(c) Both A and B Accelerator Lab in India, to work on
(d) None of the above pollution, water issues?
238. Which of the following is/are the (a) UNICEF
discretionary powers of Governor? (b) UNESCO
1. Can appoint anybody as chief (c) UNFCCC
minister if there is no political (d) UNDP
party with a clear-cut majority 243. Consider the following
in the assembly. statements
2. Can refuse to sign to an 1. Supplementary grants are the
ordinary bill passed by the additional grant required to
state legislature meet the required expenditure
3. Can reserve a bill passed by the of the government
state legislature for president’s 2. Excess Grant is granted for an
assent exceptional purpose which
4. Can seek information from the forms no part of the current
chief minister with regard to service of any financial year
the administrative and 3. Token Grant is granted when
legislative matters of the state money has been spent on any
Select the correct answer using the service during a financial year
code below: in excess of the amount
(a) 1, 3 and 4 granted for that year
(b) 1, 2 and 3 Which of the above given
(c) 2 and 4 statements is/are correct?
(d) All of the above (a) 1 only
239. Automatic Exchange of (b) 1 and 2 only
Information (AEOI) Regime is an (c) 1, 2 and 3
international deal signed between (d) 2 and 3 only
(a) India and Japan 244. With reference to FASTags,
(b) India and USA Consider the following statements
(c) India and Russia 1. All lanes, except one on each
(d) None of the above side, at all NHAI toll plazas
240. India’s longest electrified rail were to be declared as
tunnel was recently inaugurated in dedicated FASTag lanes
(a) Ladakh 2. After its commencement, any
(b) Assam motorist entering the dedicated
(c) Andhra Pradesh FASTag lane without the tag
(d) Karnataka would be charged double the
241. Recently a bill has been toll fee
introduced in Lok Sabha proposes 3. One of the lanes may be kept
setting up of a Re-skilling fund, as hybrid lane accepting
which is primarily related to FASTag and other modes of
(a) For training of retrenched payment even after its
employees commencement
(b) For training of central Which of the above given
government employees statements is/are not correct?
(a) 3 only

Telegram: https://t.me/insightsIAStips
38
Youtube: https://www.youtube.com/channel/UCpoccbCX9GEIwaiIe4HLjwA
Revision Through MCQs (RTM) Compilation (November 2019)

(b) 2 only 249. 42nd constitutional amendment,


(c) 1 only which was passed during
(d) None of the above emergency, has moved which of
245. Which of the following is the first the following subjects from state to
nation to declare climate concurrent list?
emergency? 1. Population control
(a) Scotland 2. Forest
(b) United Kingdom 3. Education
(c) EU 4. Administration of justice
(d) France Select the correct answer using the
246. Recently 6th Asian code below:
Dendrochronology Conference (a) 1, 3 and 4
being held at the Birbal Sahni (b) 1 and 3 only
Institute of Palaeosciences in (c) 1, 2 and 4
Lucknow. This is the first time that (d) All of the above
the conference is being held in 250. Which of the following is/are the
India. In this context what do you primary requirement for a person
mean by Dendrochronology? to practice medicine in India with
(a) Dendrochronology is the study foreign medical qualification?
of tree rings (a) Clearing National exit exam
(b) Dendrochronology is the study (b) One year internship in a
of tree branch hospital recognised by the
(c) Dendrochronology is the study Medical Council of India
of tree stem (c) Recommendation letter by the
(d) Dendrochronology is the study local MP
of tree roots (d) All of the above
247. ‘YuWaah’- Generation Unlimited 251. “He was considered as Unofficial
initiative is launched by Ambassador of India; he was one
(a) Ministry of Skill Development of the founding members of the
and Entrepreneurship Indian National Congress; he was a
(b) Ministry of Youth first Indian to be a British MP and
(c) NITI his major works includes
(d) None of the above Admission of educated natives into
248. Consider the following the Indian Civil Service and The
statements regarding cold wave in wants and means of India”. He was
India (a) Lala Lajpat Rai
1. A cold wave is a rapid fall in (b) Gopal Krishna Gokhale
temperature within a 24 hour (c) Udham Singh
period (d) Dadabhai Naoroji
2. The core of a cold wave at the 252. With reference to Ghadar Party
surface is a strong high- ,Consider the following statements
pressure center that forms 1. After the outbreak of World
during winter in high latitudes. War I, Ghadar party members
3. In India, Core cold wave zone returned to Punjab to agitate
covers Kerala, Tamil Nadu and for rebellion for freedom
Karnataka Movement
Which of the above given 2. The founding president of
statements is/are correct? Ghadar Party was Sohan Singh
(a) 1 and 2 Bhakna and Udham Singh was
(b) 2 only the co-founder of this party
(c) 1 only 3. Komagata Maru tragedy
(d) 1, 2 and 3 associated with Ghadar Party

Telegram: https://t.me/insightsIAStips
39
Youtube: https://www.youtube.com/channel/UCpoccbCX9GEIwaiIe4HLjwA
Revision Through MCQs (RTM) Compilation (November 2019)

Which of the given above 3. Under the individual


statements is/are not correct? Membership Action Plans,
(a) 1 and 2 only NATO currently has two
(b) 2 only candidate countries ie Bosnia
(c) 1 and 3 only and India, are in the process of
(d) 1, 2 and 3 joining the alliance
253. Which of the following section of Which of the given above
the Representation of the People statements is/are Incorrect?
Act, 1951 governs the registration (a) 3 only
of political parties in India? (b) 1 and 2 only
(a) Section 26A (c) 1 and 3 only
(b) Section 27A (d) 1, 2 and 3
(c) Section 28A 257. With reference to difference
(d) Section 29A between Censure motion and No-
254. Chagos Islands, a small confidence motion, consider the
archipelago is located in following statements
(a) Indian Ocean 1. Both censure motion and no-
(b) Pacific Ocean confidence motion can be
(c) Atlantic Ocean moved in Lok Sabha or lower
(d) Arctic Ocean house in states
255. Consider the following 2. According to procedures, a no-
statements: confidence motion can be
1. Global Migration Report 2020 moved only in Lok Sabha
has been released by 3. No-confidence motion moved
International Organization for against the entire Council of
Migration (IOM) Ministers and not individual
2. IOM is part of the United ministers or private members,
Nations Secretariat and it while censure motion can be
provides services and advice moved against individual
concerning migration to ministers or members
governments and migrants, Which of the given above
including internally displaced statements is/are correct?
persons, refugees, and migrant (a) 1 and 3
workers (b) 2 only
Which of the given above (c) 1 and 2
statements is/are correct? (d) 1, 2 and 3
(a) 1 only 258. Which of the following is the
(b) 2 only world’s largest cotton producer in
(c) Both 1 and 2 2019-2020 season?
(d) Neither 1 nor 2 (a) China
256. With reference to North Atlantic (b) India
Treaty Organization(NATO), (c) Brazil
Consider the following statements (d) Pakistan
1. It is an intergovernmental
military alliance
2. NATO constitutes a system of
collective defence whereby its
independent member states
agree to mutual defence in
response to an attack by any
external party

Telegram: https://t.me/insightsIAStips
40
Youtube: https://www.youtube.com/channel/UCpoccbCX9GEIwaiIe4HLjwA
Revision Through MCQs (RTM) Compilation (November 2019)

259. Consider the following 260. “This island is in the Sundarban


statements Delta complex of the Bay of Bengal
1. Nominal GDP is GDP evaluated has been facing the devastating
at current market prices effects of climate change. With
2. Real GDP is a measure of rising sea levels, high tides and
growth without according floods have been swallowing
inflation portions of the island. In the last
Which of the given above two decades, the island's size has
statements is/are correct? reduced by several times its
(a) 1 only original land mass”
(b) 2 only The above given passage refers to
(c) Both 1 and 2 which of the following island?
(d) Neither 1 nor 2 (a) Ghoramara island
(b) Bhasan char island
(c) Baratang Island
(d) Chatham Island

Telegram: https://t.me/insightsIAStips
41
Youtube: https://www.youtube.com/channel/UCpoccbCX9GEIwaiIe4HLjwA
Revision Through MCQs (RTM) Compilation (November 2019)

RTM- REVISION THROUGH MCQS - 1th -Nov-2019

1. “This place lies on the right bank of a distributary of river Swarnamukhi about
eighty kilometers from Tirupati and Nellore. Recent Explorations conducted in and
around this site within a radius of 15 kilometers revealed important vestiges viz.,
fortified early historic settlement Sunbrahmanya temple, unique rock cut laterite
stepped well at Yakasiri, Vishnu temple”
The above passage describes which of the following archeological site in India?
(a) Keeladi
(b) Gottiprolu
(c) Pavurallakonda
(d) Piklihal
Ans: (b)
Explanation:
 The site of Gottiprolu (13°
56’ 48” N; 79° 59’ 14” E) lies
on the right bank of a
distributary of river
Swarnamukhi about
seventeen kilometers east of
Naidupet and eighty
kilometers from Tirupati
and Nellore. Detailed
topographical study and
drone images have helped
in identifying an early
historic settlement
surrounded by a fortification and the possibility of a moat encircling it.
The fortification is very much clear on the eastern and southern sides of
the mound while its other arm seems to have been leveled as a result of
modern settlements
 Explorations conducted in and around Gottiprolu within a radius of 15
kilometers revealed important vestiges viz., fortified early historic
settlement at Puduru, Sunbrahmanya temple at Mallam, unique rock cut
laterite stepped well at Yakasiri, Vishnu temple at Tirumuru. Further the
entire seacoast on the east is dotted with various forms of antiquarian
remains extending from Tamil Nadu and culturally linked with each
other.
 The two fortified townships within the gap of 15kms during early historic
times indicates the significant strategic location was preferred by the
early historic people for trade in the region keeping in view of proximity of
the sea, River and Lake (Pulicat).
Refer: https://www.insightsonindia.com/2019/11/01/gottiprolu/

2. With reference to significance of October 31 in Indian History, consider the


following statements
1. On this day, Indian Prime Minister Rajiv Gandhi was assassinated
2. This day was the Birth Anniversary of Sardar Vallabhbhai Patel
Which of the statements given above is/are correct?
(a) 1 only
(b) 2 only

Telegram: https://t.me/insightsIAStips
42
Youtube: https://www.youtube.com/channel/UCpoccbCX9GEIwaiIe4HLjwA
Revision Through MCQs (RTM) Compilation (November 2019)

(c) Both 1 and 2


(d) Neither 1 nor 2
Ans: (b)
Explanation:
 31 Oct, 1984 - Indian Prime Minister Indira Gandhi was gunned down
outside of her home by Sikh extremists.
 Rashtriya Ekta Diwas was observed on 31st October across the nation.It
marks the occasion of the birth anniversary of Sardar Vallabhbhai Patel
Refer: https://www.insightsonindia.com/2019/11/01/rashtriya-ekta-diwas/

3. With reference to Indian modern history, consider the following statements


1. In 1928, No Tax Campaign led by Vallabhbhai Patel in Gujarath
2. In1931, Vallabhbhai Patel elected as the President of Indian National
Congress
3. Patel participated in the famous Salt Satyagraha movement and led it from
front
Which of the statements given above is/are correct?
(a) 1 and 2
(b) 3 only
(c) 2 and 3
(d) 1, 2 and 3
Ans: (a)
Explanation:
 1930– Imprisoned for participating in the famous Salt Satyagraha
movement initiated by Mahatma Gandhi.
 As Gandhi embarked on the Dandi Salt March, Patel was arrested in the
village of Ras and was put on trial without witnesses, with no lawyer or
journalists allowed to attend. Patel's arrest and Gandhi's subsequent
arrest caused the Salt Satyagraha to greatly intensify in Gujarat –
districts across Gujarat launched an anti-tax rebellion until and unless
Patel and Gandhi were released
Refer: https://www.insightsonindia.com/2019/11/01/rashtriya-ekta-diwas/

4. With reference to Indian polity, consider the following statements


1. The present delimitation of constituencies has been done on the basis of
2001 census under the provisions of Delimitation Act, 2002
2. Delimitation Commission of India’s orders can be challenged in court of law
3. Delimitation commissions have been set up four times in the past
Which of the statements given above is/are correct?
(a) 1 and 2
(b) 3 only
(c) 1 and 3
(d) 1, 2 and 3
Ans: (c)
Explanation:
 The Delimitation commission or Boundary commission of India is a
commission established by the Government of India under the provisions
of the Delimitation Commission Act.

Telegram: https://t.me/insightsIAStips
43
Youtube: https://www.youtube.com/channel/UCpoccbCX9GEIwaiIe4HLjwA
Revision Through MCQs (RTM) Compilation (November 2019)

 The main task of the commission is redrawing the boundaries of the


various assembly and Lok Sabha constituencies based on a recent
census.
 The representation from each State is not changed during this exercise.
However, the number of SC and ST seats in a state are changed in
accordance with the census.
 The present delimitation of constituencies has been done on the
basis of 2001 census under the provisions of Delimitation Act, 2002.
 The Commission is a powerful and independent body whose orders
cannot be challenged in any court of law. The orders are laid before
the Lok Sabha and the respective State Legislative Assemblies. However,
modifications are not permitted.
 Delimitation commissions have been set up four times in the past —
1952, 1963, 1973 and 2002 — under Delimitation Commission Acts of
1952, 1962, 1972 and 2002.
 The union government had suspended delimitation in 1976 until after
the 2001 census so that states' family planning programs would not
affect their political representation in the Lok Sabha. This had led to wide
discrepancies in the size of constituencies, with the largest having over
three million electors, and the smallest less than 50,000.
Refer: https://www.insightsonindia.com/2019/11/01/jammu-and-kashmir-
bifurcation/

5. Recently PESA has been repeatedly mentioned in media, related to


(a) OECD
(b) UNDP
(c) UNICEF
(d) UNESCO
Ans: (a)
Explanation:
 It is an international assessment that measures 15-year-old students’
reading, mathematics, and science literacy every three years.
 First conducted in 2000, the major domain of study rotates between
reading, mathematics, and science in each cycle.
 PISA also includes measures of general or cross-curricular competencies,
such as collaborative problem solving.
 PISA is coordinated by the Organization for Economic Cooperation
and Development (OECD), an intergovernmental organization of
industrialized countries, and is conducted in the United States by NCES.
Refer: https://www.insightsonindia.com/2019/11/01/programme-for-
international-students-assessment-pisa/

6. Consider the following pairs


Disease Caused by
1. Lymphatic filariasis Parasite
2. Malaria Mosquito
3. Rabies Virus
4. Cholera Bacteria
5. Rubella Fungi

Telegram: https://t.me/insightsIAStips
44
Youtube: https://www.youtube.com/channel/UCpoccbCX9GEIwaiIe4HLjwA
Revision Through MCQs (RTM) Compilation (November 2019)

Select the correct answer using the code below


(a) 1, 2, 3 and 4
(b) 2, 3, 4 and 5
(c) 1, 3, 4 and 5
(d) 2, 4 and 5
Ans: (a)
Explanation:
 German measles (Rubella)- Rubella virus
Refer: https://www.insightsonindia.com/2019/11/01/lymphatic-filariasis/

7. National Health Profile, 2019 has been recently released by


(a) NITI
(b) NHFS
(c) CBHI
(d) NHRC
Ans: (c)
Explanation:
 Central Bureau of Health Intelligence (CBHI), established in 1961 by
the Act of Parliament on the recommendation of Mudaliar committee, is
the Health Intelligence Wing under Directorate General of Health Services
(Dte.GHS), Ministry of Health & Family Welfare (MoHFW).
 CBHI is headed by Director (SAG level) from Central Health Services
cadre with specialization in public health administration supported by
officials Indian Statistical Services (ISS), Subordinate Statistical Services
(SSS) and Central Secretariat Services (CSS).
Refer: https://www.insightsonindia.com/2019/11/01/national-health-profile-
2019/

8. Which of the following Indian states celebrate November 1 as their Formation


Day?
1. Karnataka
2. Kerala
3. Chhattisgarh
4. Punjab
5. Haryana
6. Madhya Pradesh
Select the correct answer using the code below
(a) 1, 2, 3 and 4
(b) 1, 3, 4 and 5
(c) 1, 4, 5 and 6
(d) 1, 2, 3, 4, 5 and 6
Ans: (d)
Explanation:
 November 1 is celebrated as
o Kannada Rajyotsava in karnataka
o ‘Kerala Piravi Dinam’ in the kerala
 Madhya Pradesh, the second largest state in the country by area and
sixth-largest in terms of population, was formed on Nov 1, 1956.It is the
second largest state in India in terms of area and has the sixth-largest
population among Indian states

Telegram: https://t.me/insightsIAStips
45
Youtube: https://www.youtube.com/channel/UCpoccbCX9GEIwaiIe4HLjwA
Revision Through MCQs (RTM) Compilation (November 2019)

 Rajasthan is a state in the north western region of India. The state covers
10.4 percent of the total geographical area of India. It is the largest
Indian state by area and the seventh largest by population. Rajasthan
was reorganised on November 1, 1956
 On November 1, Punjab celebrates its Formation Day. The Punjabi Suba
movement was one of the key events in the formation of Punjab in the
erstwhile East Punjab state of India in the 1950s. The movement, led by
the Akali Dal, resulted in the formation of the Punjabi-majority Punjab
state, the Haryanvi-Hindi-majority Haryana state and the Union Territory
of Chandigarh.
 The state of Chhattisgarh was carved out of the state of Madhya Pradesh
on Nov 1, 2000. It is the tenth largest state of India

9. ENSURE Portal is primarily related to


(a) Public service
(b) Subsidy payment
(c) Data management
(d) Education
Ans: (b)
Explanation:
 ENSURE Portal has been launched by the Union Minister of Agriculture
and Farmer’s Welfare Mr.Radha Mohan Singh.
 The ENSURE 2.0 Online Portal is developed by the NABARD. ENSURE
Direct Transfer (DBT) will operate under the Department of Animal
Husbandry, Dairying & Fisheries. The objective of developing ENSURE
Portal is to make subsidy transfer process quicker& Faster.
Refer: https://www.insightsonindia.com/2018/12/12/online-portal-ensure/

10. Who was the author of The Philosophy of Rabindranath Tagore?


(a) Sarvepalli Radhakrishnan
(b) Subhas Chandra Bose
(c) Ras Bihari Bose
(d) Jawahar Lal Nehru
Ans: (a)
Explanation:
 Sarvepalli Radhakrishnan was India’s first Vice President and second
President.
 His book, ‘The Philosophy of Rabindranath Tagore’ attracted global
attention to Indian philosophy.
 His philosophy was grounded in Advaita Vedanta. He defended Hinduism
against “uninformed Western criticism” and played a major role in the
formation of contemporary Hindu identity.
 He earned the reputation of being the bridge-builder between India and
the West.
 He was one of the founders of Helpage India, a renowned NGO for elderly
underprivileged in India.
 He was awarded the Bharat Ratna, India’s highest civilian award, in
1954.
 He was awarded several other distinguished awards as well such as a
knighthood in 1931 and honorary membership of the British Royal Order
of Merit in 1963.

Telegram: https://t.me/insightsIAStips
46
Youtube: https://www.youtube.com/channel/UCpoccbCX9GEIwaiIe4HLjwA
Revision Through MCQs (RTM) Compilation (November 2019)

Refer: https://www.insightsonindia.com/2019/09/05/insights-daily-current-
affairs-pib-05-september-2019/

RTM- REVISION THROUGH MCQS - 2nd -Nov-2019

11. With respect to Electoral Bond’s, consider the following statements


1. Electoral bond is a bearer instrument in the manner of a promissory note
2. Bonds shall not carry the donor's name, although the payee will have to fulfil
KYC
3. Every party that is registered under section 29A of the Representation of the
Peoples Act, 1951 are eligible to receive money through electoral bonds
Which of the given above statements is/are correct?
(a) 1 and 2
(b) 2 and 3
(c) 1 and 3
(d) All of the above
Ans: (a)
Explanation:
 Electoral Bond- the banking instrument resembling promissory notes will
not carry any interest.
 It will be a bearer instrument.
 It will not carry the name of the payee.
 It can be bought for any value, in multiples of Rs 1,000, Rs 10,000, Rs 1
lakh, Rs 10 lakh or Rs 1 crore
 The electoral bonds can be purchased for specified denominations and
the payee then can bestow it upon a registered political party as donation
which can then be cashed in via the party's verified account within 15
days.
 The bonds shall not carry the donor's name, although the payee will have
to fulfil KYC (Know Your Customer) norms at the bank. Further, no
report is required to be submitted by receiving parties in case of
donations received via electoral bonds. In short, neither the donors nor
the political parties are obliged to reveal the sources of donations.
 Every party that is registered under section 29A of the
Representation of the Peoples Act, 1951 and has secured at least
one per cent of the votes polled in the most recent Lok Sabha or
State election will be allotted a verified account by the Election
Commission of India. Electoral bond transactions can be made only via
this account.
Refer: https://www.insightsonindia.com/2019/11/02/insights-daily-current-
affairs-pib-02-november-2019/

12. Consider the following statements


1. Tiware dam is located in Maharashtra
2. Polavaram is a multi-purpose irrigation project on the river Krishna
Which of the statements given above is/are Not correct?
(a) 1 only
(b) 2 only
(c) Both 1 and 2
(d) Neither 1 nor 2

Telegram: https://t.me/insightsIAStips
47
Youtube: https://www.youtube.com/channel/UCpoccbCX9GEIwaiIe4HLjwA
Revision Through MCQs (RTM) Compilation (November 2019)

Ans: (b)
Explanation:
 On 2 July 2019, the Tiware dam in Ratnagiri district of Maharastra
state of India failed following heavy rains. The failure resulted in flooding
of the villages situated downstream. At least 19 people died, most of them
from the lower castes, and four went missing.
 Polavaram dam across the Godavari River is under construction located
in West Godavari District and East Godavari District in Andhra Pradesh
state and its reservoir spreads in parts of Chhattisgarh and Orissa States
also.
Refer: https://www.insightsonindia.com/2019/11/02/polavaram-project-2/

13. With reference to Graded Response Action Plan (GRAP), consider the following
statements
1. It works only as an emergency and short term measure
2. GRAP does not include action by various state governments to be taken
throughout the year to tackle air pollution
Which of the statements given above is/are correct?
(a) 1 only
(b) 2 only
(c) Both 1 and 2
(d) Neither 1 nor 2
Ans: (c)
Explanation:
 Approved by the Supreme Court in 2016, the plan was formulated after
several meetings that the Environment Pollution (Prevention and Control)
Authority (EPCA) held with state government representatives and experts.
The result was a plan that institutionalised measures to be taken when
air quality deteriorates.
 GRAP works only as an emergency measure. As such, the plan does not
include action by various state governments to be taken throughout the
year to tackle industrial, vehicular and combustion emissions. When the
air quality shifts from poor to very poor, the measures listed under both
sections have to be followed since the plan is incremental in nature.
Refer: https://www.insightsonindia.com/2019/11/02/public-health-emergency-
declared-in-delhi/

14. With reference to Armed Forces (Special Powers) Act (AFSPA), consider the
following statements
1. AFSPA was first enacted as an ordinance in the backdrop of Quit India
Movement
2. AFSPA was first enacted to deal with the Naga insurgency in the Assam
region.
3. Governor of a State and the Central Government are empowered to declare
any part or full of any state as a disturbed area
Which of the given above statements is/are correct?
(a) 1 and 2
(b) 2 and 3
(c) 1 and 3
(d) All of the above
Ans: (d)

Telegram: https://t.me/insightsIAStips
48
Youtube: https://www.youtube.com/channel/UCpoccbCX9GEIwaiIe4HLjwA
Revision Through MCQs (RTM) Compilation (November 2019)

Explanation: The salient features of the AFSPA act are:


 Governor of a State and the Central Government are empowered to
declare any part or full of any state as a disturbed area if according
to their opinion that it has become necessary to disrupt terrorist
activity or any such activity that might impinge on the sovereignty
of India or cause insult to the national flag, anthem or India’s
Constitution.
 Section (3) of AFSPA provides that, if the governor of a state issues an
official notification in The Gazette of India then the Central government
has the authority to deploy armed forces for assisting the civilian
authorities. Once a region is declared ‘disturbed’ then it has to maintain
status quo for a minimum of three months, as per The Disturbed Areas
Act of 1976.
 Section (4) of AFSPA gives special powers to army officers in disturbed
areas to shoot (even if it kills) any individual who violates law / or is
suspected to violate law (this includes assembly of five or more people,
carrying of weapons) etc. The only condition is that the officer has to give
a warning before opening fire.
 Security forces can arrest anybody even without a warrant, and carry out
searches without consent.
 Once a person is taken into custody, he/she has to be handed over to the
nearest police station as soon as possible.
 Prosecution of the officer on duty for alleged violation of human rights
requires the prior permission of the Central Government.
 AFSPA was first enacted to deal with the Naga insurgency in the
Assam region.
o In 1951, the Naga National Council (NNC) reported that it
conducted a “free and fair plebiscite” in which about 99 percent of
Nagas voted for a ‘Free Sovereign Naga Nation’. There was a
boycott of the first general election of 1952 which later extended to
a boycott of government schools and officials.
 The AFSPA – like many other controversial laws – is of a colonial
origin. The AFSPA was first enacted as an ordinance in the backdrop
of Quit India Movement launched by Mahatma Gandhi in 1942.
Refer: https://www.insightsonindia.com/2019/11/02/afspa-3/

15. Recently Chrysaor, Framroot and Zero-day vulnerability has been repeatedly
mentioned in media with apprehension. These are related to
(a) Climate change
(b) Kerala mass killing
(c) Cyber security
(d) Agri pesticide
Ans: (c)
Explanation:
 Android version of Pegasus spyware is called Chrysaor Malware
 The android version of Pegasus installs as an application on phone and
uses root technique called framaroot
 A zero-day (also known as 0-day) vulnerability is a computer-software
vulnerability that is unknown to, or unaddressed by, those who should
be interested in mitigating the vulnerability (including the vendor of the
target software).

Telegram: https://t.me/insightsIAStips
49
Youtube: https://www.youtube.com/channel/UCpoccbCX9GEIwaiIe4HLjwA
Revision Through MCQs (RTM) Compilation (November 2019)

Refer: https://www.insightsonindia.com/2019/11/02/whatsapp-hacking-and-
issues-related/

16. Consider the following statements


1. CERT-In is nodal government agency that deals with cyber security threats
like hacking and phishing in India
2. Defence cyber agency focuses on non-civilian cyber issues
Which of the statements given above is/are correct?
(a) 1 only
(b) 2 only
(c) Both 1 and 2
(d) Neither 1 nor 2
Ans: (c)
Explanation:
 Defence Cyber Agency: It will work in close coordination with National
Cyber Security Advisor. It will focus on non-civilian cyber issues,
including safeguarding critical infrastructure. It will be have over 1,000
personnel and will be distributed to various formations of Army, Navy
and IAF.
Refer: https://www.insightsonindia.com/2019/11/02/whatsapp-hacking-and-
issues-related/

17. Serious Fraud Investigation Office (SFIO) falls under which ministry
(a) MoCA
(b) MoF
(c) MHA
(d) MoD
Ans: (a)
Explanation:
Refer: Facts for prelims: https://www.insightsonindia.com/2019/11/02/insights-
daily-current-affairs-pib-02-november-2019/

18. With reference to diffused light, consider the following statements


1. Light is diffused due to its dual nature of wave and particle
2. Light diffuses when it gets scattered
3. In atmosphere light gets diffused by gas molecules
Which of the above statements is/are correct?
(a) 1 and 2 Only
(b) 1 Only
(c) 2 and 3 Only
(d) 1,2 and 3
Ans: (d)
Explanation:
 First statement is right as it is its particle nature of light that scatters it.
(https://sciencing.com/diffused-light-5470956.html)
 Second statement is correct. Light is diffused after being scattered by
particles in atmosphere (or in any medium)
 Rayleigh scattering of sunlight in the atmosphere causes diffuse sky
radiation, which is the reason for the blue color of the sky and the yellow
tone of the sun itself. This involves gas molecules.

Telegram: https://t.me/insightsIAStips
50
Youtube: https://www.youtube.com/channel/UCpoccbCX9GEIwaiIe4HLjwA
Revision Through MCQs (RTM) Compilation (November 2019)

19. Consider the following events


1. First Famine Commission
2. Arms Act to restrict only Indians to carry arms
3. Introduced uniform salt tax throughout British India
4. War with Afghans
Under whose rule the above given events happened?
(a) Lord Ripon
(b) Lord Lytton
(c) Lord Curzon
(d) None of the above shared all four events simultaneously
Ans: (b)
Explanation:
 Under Lytton, the first Famine Commission (1878-80) under Sir
Richard Strachey was appointed and it made many commendable
recommendations. They include provision of funds for famine relief and
construction work in the annual budget. The Famine Code came into
existence in 1883.
 In 1878, the Arms Act was passed by Lord Lytton. This Act prevented
the Indians to keep arms without appropriate license. Its violation would
be a criminal offence. The Europeans and the Anglo- Indians were
exempted from the operation of these legislations
 Lord Lytton introduced uniform salt tax throughout British India
 when Lord Lytton was appointed the Viceroy of India, he was instructed
by the
home government to follow a forward policy. The Russian attempt to send
a mission to Afghanistan was the main cause of the Second Afghan War
from 1878 to 1880.

20. Recently Gandhipedia has been in news for sometimes is launched by


(a) Wikipedia
(b) Ministry of culture
(c) UNESCO
(d) National Council for Science Museums
Ans: (d)
Explanation:
 Gandhipedia is being developed by the National Council for Science
Museums (NCSM) to sensitise the youth and the society at large about
Gandhian values
 The portal will function on artificial intelligence (AI) and machine
running. It will be a repository of knowledge on Mahatama Gandhi. The
portal will contain original photographs, visuals, speeches and the 100
collected works of Gandhiji. Books by Gandhiji will also be available on
the portal apart from books that have been written on him
 The Gandhipedia will be free for access to everybody. The project is at
present being designed by the National Council for Science and
Museums. It will be jointly implemented by IIT-Kharagpur and IIT-
Gandhinagar

Telegram: https://t.me/insightsIAStips
51
Youtube: https://www.youtube.com/channel/UCpoccbCX9GEIwaiIe4HLjwA
Revision Through MCQs (RTM) Compilation (November 2019)

RTM- REVISION THROUGH MCQS - 4th -Nov-2019

21. With reference to North Eastern India, Consider the following statements
1. Currently Inner Line Permit is operational in Arunachal Pradesh, Mizoram
and Tripura
2. Assam has the largest and smallest river islands in the world
3. Population of all the Northeastern states is close to the population of Odisha
Which of the above statements is/are correct?
(a) 1 and 2 Only
(b) 1 Only
(c) 2 and 3 Only
(d) 1,2 and 3
Ans: (c)
Explanation:
 Currently, the Inner Line Permit is operational in Arunachal Pradesh,
Mizoram and Nagaland.
 An unknown fact about Assam is that it is home to some of the biggest
and smallest river Islands. Where Majuli Island is the world’s biggest
River Island and the Umananda Island is believed to be the world’s
smallest inhabited island and it is nestled right in the middle of river
Brahmaputra.
 The total population of all the seven sister states and Sikkim combined
together are roughly equal to the population of the eastern state of
Odisha, which happens to be the 11th most populated state in the
country. The state of Arunachal Pradesh is the least dense state in India
while being the biggest out of all 8 states of Northeast India, in terms of
landmass.
Refer: https://www.insightsonindia.com/2019/11/04/all-visitors-to-meghalaya-
must-first-register/

22. With reference to East Asia Summit(EAS), consider the following statements
1. EAS is an initiative of SAARC
2. It is a forum held annually by leaders of 18 countries from all over the world
3. EAS acts as an alternative to the APEC in which India doesn’t enjoy the
membership
Which of the above statements is/are correct?
(a) 1 and 2 Only
(b) 3 Only
(c) 2 and 3 Only
(d) 1,2 and 3
Ans: (b)
Explanation:
 EAS is an initiative of ASEAN and is based on the premise of the
centrality of ASEAN.
 It is a forum held annually by leaders of 18 countries in the East Asian,
Southeast Asian and South Asian regions.
 For India, EAS acts as an alternative to the APEC in which India
doesn’t enjoy the membership.
 India’s membership to the EAS is a recognition of its fast growing
economic and political clout.

Telegram: https://t.me/insightsIAStips
52
Youtube: https://www.youtube.com/channel/UCpoccbCX9GEIwaiIe4HLjwA
Revision Through MCQs (RTM) Compilation (November 2019)

Refer: https://www.insightsonindia.com/2019/11/04/east-asia-summit/

23. Recently SCOJTEX- 2019, has been in news sometimes related to


(a) SCO
(b) ASEAN
(c) SAARC
(d) RCEP
Ans: (a)
Explanation:
 Shanghai Cooperation Organization Joint Exercise on Urban Earthquake
Search & Rescue.
 The latest edition is being held in India.
Key facts:
 Hosted by NDRF.
 Aim to rehearse the disaster response mechanism, share knowledge,
experience, technology & also for mutual coordination, etc.
 The participants of all 08 member countries namely China, India,
Kazakhastan, Kyrgyzstan, Pakistan, Russia, Tajikistan and Uzbekistan
shall be participating in this exercise.
 Main focus shall be to test the region’s preparedness and resilience
towards effective activation of Inter- governmental interaction for
immediate response.
Refer: https://www.insightsonindia.com/2019/11/04/scojtex-2019/

24. Which of the following pairs are correctly matched?


1. Global Microscope on Financial Inclusion report- Economist Intelligence Unit
2. Global Innovation Index-World Economic Forum
3. Southeast Asia Energy Outlook- ASEAN
Find the correct answer using the code below
(a) 1 and 2 Only

Telegram: https://t.me/insightsIAStips
53
Youtube: https://www.youtube.com/channel/UCpoccbCX9GEIwaiIe4HLjwA
Revision Through MCQs (RTM) Compilation (November 2019)

(b) 1 Only
(c) 3 Only
(d) 2 and 3
Ans: (b)
Explanation:
 Global Innovation Index- Cornell University INSEAD and the World
Intellectual Property Organization (WIPO)
 Southeast Asia Energy Outlook- International Energy Agency
Refer: https://www.insightsonindia.com/2019/11/04/global-microscope-on-
financial-inclusion-report/

25. Consider the following statements


1. ‘Red Atlas Action Plan Map’ atlas was prepared by NDRF
2. Coastal Flood Warning System(CFLOWS) app for Chennai is the first
integrated coastal flood warning system
Which of the statements given above is/are correct?
(a) 1 only
(b) 2 only
(c) Both 1 and 2
(d) Neither 1 nor 2
Ans: (b)
Explanation:
 Red Atlas Action Plan Map: It is a first of its kind ready reckoner map,
prepared by Union Ministry of Earth Sciences to aid state government
of Tamil Nadu in effective flood mitigation in Chennai which witnessed
the worst deluge in 2015.
 The atlas is aimed at flood mitigation, preparedness, operations and
management aspects.
 Coastal Flood Warning System app for Chennai (CFLOWS- CHENNAI):
Launched by NIOT, it is India’s first integrated coastal flood warning
system.
 It is an integrated GIS-based decision support system to provide forecast
on potential inundation 10 days in advance.
Refer: https://www.insightsonindia.com/2019/11/04/red-atlas-action-plan-map-
atlas-and-cflows-chennai/

26. Consider the following


National Waterways State Served
1. NW-2 Meghalaya
2. NW-4 Kerala
3. NW6 Assam
Which of the above is/are correctly matched?
(a) 1 and 3
(b) 2 only
(c) 3 only
(d) 2 and 3
Ans: (a)
Explanation:
 National Waterways Act came into effect in 2016. It proposed 106
additional National Waterways and merges 5 existing Acts which were
declared the 5 National Waterways.

Telegram: https://t.me/insightsIAStips
54
Youtube: https://www.youtube.com/channel/UCpoccbCX9GEIwaiIe4HLjwA
Revision Through MCQs (RTM) Compilation (November 2019)

 In 1986, the Government of India created Inland Waterways Authority of


India (IWAI) for regulation and development of Inland Waterways for
navigation and shipping.
 Out of the 111 National Waterways declared under the National
Waterways Act, 2016, 13 are operational for shipping and navigation and
cargo/passenger vessels are moving on them.

Refer: Facts for prelims: https://www.insightsonindia.com/2019/11/04/insights-


daily-current-affairs-pib-04-november-2019/

27. With Reference to National Institute of Ocean Technology (NIOT), consider the
following statements
1. It is an autonomous society
2. It falls under Department of Ocean Development
Which of the statements given above is/are correct?
(a) 1 only
(b) 2 only
(c) Both 1 and 2
(d) Neither 1 nor 2
Ans: (b)
Explanation:

Telegram: https://t.me/insightsIAStips
55
Youtube: https://www.youtube.com/channel/UCpoccbCX9GEIwaiIe4HLjwA
Revision Through MCQs (RTM) Compilation (November 2019)

 Department of Ocean Development was created in July 1981 as a


nodal and independent department under the direct charge of PM for
organising, co-coordinating and promoting ocean development activities.
 Considering the need for capacity building and specialised skilled human
resources as emphasised in the Ocean Policy Statement, Department of
Ocean Development has established national institutes, viz. National
Institute of Ocean Technology (NIOT) at Chennai, National Centre for
Antarctic and Ocean Research (NCAOR) at Goa, Indian National Centre
for Ocean and Information Services (INCOIS) at Hyderabad, Project
Directorate, Integrated Coastal and Marine Area Management (PD-
ICMAM) at Chennai and Centre for Marine Living Resources and Ecology
(CMLRE) at Kochi.
Refer: Facts for prelims:
https://www.insightsonindia.com/2019/11/04/insights-daily-current-
affairs-pib-04-november-2019/

28. Recently Dustlik 2019 has been in news sometimes related to


(a) It is the first-ever Joint Exercise between India and Uzbekistan
(b) It is the first-ever Joint Exercise between India and Kazakhstan
(c) It is the first-ever Joint Exercise between India and Turkmenistan
(d) It is the first-ever Joint Exercise between India and Afghanistan
Ans: (a)
Explanation:
Refer: Facts for prelims:
https://www.insightsonindia.com/2019/11/04/insights-daily-current-affairs-
pib-04-november-2019/

29. In ancient India, ‘Kutagarashala’ referred to


(a) A place where children of high Brahman family were given education.
(b) A place where the head of a tribe resided.
(c) A place where the debate between philosophers took place.
(d) None of the statements (a), (b) and (c) are correct.
Ans: (c)
Explanation:
 Kutagarashala – literally, a hut with a pointed roof – or in groves where
travelling mendicants halted. Debates between philosophers took place
here. If a philosopher succeeded in convincing one of his rivals, the
followers of the latter also became his disciples.

30. With reference to Chola Education and Literature, consider the following
statements:
1. Temples and mathas served as the only educational centres
2. Not only Vedas and Epics, but subjects like mathematics and medicine were
also taught
3. Endowment of lands was made to run educational centres
Select the correct answer using the codes below.
(a) 1 and 3
(b) 2 and 3 only
(c) 3 only
(d) 1 only
Ans: (b)

Telegram: https://t.me/insightsIAStips
56
Youtube: https://www.youtube.com/channel/UCpoccbCX9GEIwaiIe4HLjwA
Revision Through MCQs (RTM) Compilation (November 2019)

Explanation:
 During this period, besides the temples and mathas as educational
centres, several educational institutions also flourished.
 The development of Tamil literature reached its peak during the Chola
period. And a lot of it came from the contribution of education and
literature to the enrichment of this culture.
 Mathematics and medicine studies showed that Kings encouraged logical
and intellectual pursuits and were not centred entirely on religious
studies.

RTM- REVISION THROUGH MCQS - 5th -Nov-2019

31. With reference to Welfare policies in India, consider the following statements
1. Skills Build platform is launched by Directorate General of Supplies under the
aegis of Ministry of Skill Development & Entrepreneurship (MSDE)
2. Program of Skills Build Platform facilitates co-operation and collaboration
with private player
3. Pradhan Mantri Kaushal Vikas Yojana (PMKVY) is a flagship scheme of MSDE
Which of the given above statements is/are correct?
(a) 1 and 3
(b) 2 only
(c) 2 and 3
(d) 1,2 and 3
Ans: (c)
Explanation:
 Ministry of Skill Development & Entrepreneurship launches Skills Build
platform in Collaboration with IBM.
 Skills build Platform Launched by Directorate General of Training
(not Directorate General of supplies), under the aegis of Ministry of
Skill Development & Entrepreneurship (MSDE).
 Pradhan Mantri Kaushal Vikas Yojana (PMKVY) is a flagship scheme of
the Ministry of Skill Development and Entrepreneurship (MSDE). It
was launched with an objective of enabling a large number of youth in
India to take up skill training that is relevant to specific industry, which
would help them in securing a better livelihood.
Refer: https://www.insightsonindia.com/2019/11/05/skills-build-platform/

32. With reference to Regional Comprehensive Economic Partnership (RCEP),


consider the following statements
1. India’s trade deficit with the RCEP nations is $105 billion
2. RCEP is a preferential trade agreement originally devised to consist of 16
countries across the Asia-Pacific region
3. RCEP’s administrative core is ASEAN
Which of the given above statements is/are correct?
(a) 1 and 2
(b) 2 and 3
(c) 3 only
(d) 1 and 3
Ans: (d)
Explanation:

Telegram: https://t.me/insightsIAStips
57
Youtube: https://www.youtube.com/channel/UCpoccbCX9GEIwaiIe4HLjwA
Revision Through MCQs (RTM) Compilation (November 2019)

 Regional Comprehensive Economic Partnership is a free trade


agreement originally devised to consist of 16 countries across the Asia-
Pacific region.
 Preferential trade agreements (PTA) are economic agreements that give
preferential access to certain products from the participating countries by
reducing, but not eliminating, tariffs. Under the GATT-WTO, these
agreements are valid only if they lead to free trade between the involved
parties.
What’s the Difference?
 PTAs serve to reduce tariffs but not completely eliminate them – free
trade agreements (FTAs) often eliminate tariffs completely
 PTAs are the starting point of economic integration between the two
countries – FTAs are the final goal
 “PTAs can also involve agreements where one party is itself a PTA, such
as in the case of various agreements signed between the [European
Council (EC)] and several other European and Mediterranean countries.
There can even be PTAs in which all parties are PTAs, such as the EC-
MERCOSUR agreement under discussion”
 Generally, PTAs, unlike FTAs, do not cover all trade
Refer: Mind map on preferential trade agreements
https://www.insightsonindia.com/wp-content/uploads/2016/08/Preferential-
Trade-Agreements.jpg
https://www.insightsonindia.com/2019/11/05/regional-comprehensive-economic-
partnership-rcep-6/

33. Recently, India negotiating a agreement named Agreement on Reciprocal


Logistics Support (ARLS) with which of the following nation
(a) USA
(b) RUSSIA
(c) JAPAN
(d) CHINA
Ans: (b)
Explanation:
 India and Russia are finalising a defence agreement that will simplify
interoperability and enable military platforms to receive support and
supplies across bases in both nations- Agreement on Reciprocal Logistics
Support (ARLS).
 Agreement on Reciprocal Logistics Support (ARLS): It is an
arrangement that will allow access to India and Russia, to each other’s
military facilities for supplies and fuel, expanding the logistics support
and operational turnaround of the Indian military.
Refer: https://www.insightsonindia.com/2019/11/05/agreement-on-reciprocal-
logistics-support-arls/

34. With reference to NBFC, consider the following statements


1. NBFC is a company registered under the Companies Act,1956 or 2013
2. NBFC conducts financial activity as principal business ie when a company’s
financial assets constitute more than 50 per cent of the total assets and
income from financial assets constitute more than 50 per cent of the gross
income

Telegram: https://t.me/insightsIAStips
58
Youtube: https://www.youtube.com/channel/UCpoccbCX9GEIwaiIe4HLjwA
Revision Through MCQs (RTM) Compilation (November 2019)

3. Deposit insurance facility of Deposit Insurance and Credit Guarantee


Corporation is not available to depositors of NBFCs, unlike in case of banks
Which of the above given statements is/are correct?
(a) 1 and 3
(b) 3 Only
(c) 2 and 3
(d) 1, 2 and 3
Ans: (d)
Explanation:
 A Non – Banking Financial Corporation is a company incorporated
under the Companies Act 2013 or 1956 which is engaged in the
business of Loans and Advances, Acquisition of stocks, equities, debt etc
issued by the government or any local authority. The main objective of
this type of a company is to accept deposits under any scheme or
manner.
 According to section 45-I (c) of the RBI Act, a Non – Banking Company
carrying on the business of a financial institution will be an NBFC. It is
governed by the Ministry of Corporate Affairs as well as the Reserve
Bank of India.
 What does conducting financial activity as “principal business”
mean?
o Financial activity as principal business is when a company’s
financial assets constitute more than 50 per cent of the total assets
and income from financial assets constitute more than 50 per cent
of the gross income. A company which fulfils both these criteria will
be registered as NBFC by RBI.
o The term 'principal business' is not defined by the Reserve
Bank of India Act.
 NBFCs are doing functions similar to banks. What is difference
between banks & NBFCs?
o NBFCs lend and make investments and hence their activities are
akin to that of banks; however there are a few differences as given
below:
o NBFC cannot accept demand deposits;
o NBFCs do not form part of the payment and settlement system
and cannot issue cheques drawn on itself;
o Deposit insurance facility of Deposit Insurance and Credit
Guarantee Corporation is not available to depositors of NBFCs,
unlike in case of banks.
Refer: https://www.insightsonindia.com/2019/11/05/nbfc-liquidity-norms/

35. What do you mean by the term called “LCR”?


(a) It refers to the proportion of highly liquid assets held by companies to ensure
their ongoing ability to meet short-term obligations.
(b) It mean liquid assets that can be readily sold or immediately converted into
cash at little or no loss of value, or used as collateral to obtain funds in a
range of stress scenarios
(c) It allows banks borrow money through repurchase agreements
(d) None of the above
Ans: (a)
Explanation:

Telegram: https://t.me/insightsIAStips
59
Youtube: https://www.youtube.com/channel/UCpoccbCX9GEIwaiIe4HLjwA
Revision Through MCQs (RTM) Compilation (November 2019)

 Liquidity coverage ratio (LCR): It refers to the proportion of highly


liquid assets held by companies to ensure their ongoing ability to meet
short-term obligations.
 High Quality Liquid Assets (HQLAs):It mean liquid assets that can be
readily sold or immediately converted into cash at little or no loss of
value, or used as collateral to obtain funds in a range of stress scenarios
 Liquidity adjustment facility (LAF) is a tool used in monetary policy,
primarily by the Reserve Bank of India (RBI) that allows banks to borrow
money through repurchase agreements (repos) or for banks to make
loans to the RBI through reverse repo agreements.
Refer: https://www.insightsonindia.com/2019/11/05/nbfc-liquidity-norms/

36. With reference to NAVIC, consider the following statements


1. It is an independent global navigation satellite system
2. It is designed to provide accurate position information service to users in
India as well as the region extending up to 5500 sqkm from its boundary
3. The space segment consists of the IRNSS with an operational name of NAVIC
constellation of eight satellites
Which of the above statements given is/are Not correct?
(a) 1 and 3
(b) 3 only
(c) 1 and 2
(d) 1, 2 and 3
Ans: (d)
Explanation: Here Directive word is Not Correct!
 IRNSS is an independent regional (not global) navigation satellite
system being developed by India. It is designed to provide accurate
position information service to users in India as well as the region
extending up to 1500 km from its boundary, which is its primary service
area
 IRNSS will provide two types of services, namely, Standard Positioning
Service (SPS) which is provided to all the users and Restricted Service
(RS), which is an encrypted service provided only to the authorised users.
The IRNSS System is expected to provide a position accuracy of better
than 20 m in the primary service area.
 Some applications of IRNSS are:
o Terrestrial, Aerial and Marine Navigation
o Disaster Management
o Vehicle tracking and fleet management
o Integration with mobile phones
o Precise Timing
o Mapping and Geodetic data capture
o Terrestrial navigation aid for hikers and travellers
o Visual and voice navigation for drivers
 The IRNSS space segment consists of 7 satellites (3 GEO and 4 GSO)
 Three of these will be geostationary over the Indian Ocean, i.e., they will
appear to be stationary in the sky over the region, and four will be
geosynchronous – appearing at the same point in the sky at the same
time every day.
Refer: https://www.insightsonindia.com/2019/11/05/navic-2/

Telegram: https://t.me/insightsIAStips
60
Youtube: https://www.youtube.com/channel/UCpoccbCX9GEIwaiIe4HLjwA
Revision Through MCQs (RTM) Compilation (November 2019)

37. Recently, ICEDASH and ATITHI has been in news sometimes related to
(a) New IT Initiatives to regulate custom clearance in airport only
(b) New IT Initiatives to regulate custom clearance in port only
(c) New IT Initiatives to regulate custom clearance in airport and port
(d) New IT Initiatives to regulate custom clearance in airport, border bus station
and port
Ans: (c)
Explanation:
 ICEDASH is an Ease of Doing Business (EoDB) monitoring dashboard of
the Indian Customs helping public see the daily Customs clearance
times of import cargo at various ports and airports. With ICEDASH,
Indian Customs has taken a lead globally to provide an effective tool that
helps the businesses compare clearance times across ports and plan
their logistics accordingly. This dashboard has been developed by CBIC
in collaboration with NIC(not C-DAC). ICEDASH can be accessed
through the CBIC website.
 With ATITHI, CBIC has introduced an easy to use mobile app for
international travelers to file the Customs declaration in advance.
Passengers can use this app to file declaration of dutiable items and
currency with the Indian Customs even before boarding the flight to India.
Refer: Facts for prelims: https://www.insightsonindia.com/2019/11/05/insights-
daily-current-affairs-pib-05-november-2019/

38. Polypedates bengalensis is a newly discovered frog species from


(a) Western Ghats
(b) West Bengal
(c) Western Himalaya
(d) West Meghalaya
Ans: (b)
Explanation:
Refer: Facts for prelims: https://www.insightsonindia.com/2019/11/05/insights-
daily-current-affairs-pib-05-november-2019/

39. Consider the following statements


1. White Biotechnology is applied to industrial process
2. Red Biotechnology is concerned with development of Drugs
3. Green Biotechnology is related Agriculture
Which of the above given statements is/are correct?
(a) 1 and 2
(b) 3 only
(c) 2 only
(d) 1, 2 and 3
Ans: (d)
Explanation: Color Code of Biotechnology
 The life science sector of biotechnology is occasionally described as a
rainbow with each sector having its own color code.
 Biotechnology is symbolized majorly by four colours.
o Red biotechnology is for the health and medical sector
including diseases & diagnostics

Telegram: https://t.me/insightsIAStips
61
Youtube: https://www.youtube.com/channel/UCpoccbCX9GEIwaiIe4HLjwA
Revision Through MCQs (RTM) Compilation (November 2019)

o Green biotechnology refers to the agriculture or food, plant


and environment
o white or grey biotechnology indicates the industrial sector
o Blue biotechnology is for aquaculture and marine

40. With reference to ISRO, consider the following statements


1. Indian National Committee for Space Research (INCOSPAR) was established
under Jawaharlal Nehru government in 1962 under the Department of
Atomic Energy
2. first Indian-made satellite was the Rohini-75
3. first Indian-built launch vehicle was ASLV
Which of the above given statements is/are correct?
(a) 1 and 2
(b) 2 and 3
(c) 2 only
(d) 1 and 3
Ans: (a)
Explanation: ISRO evolution, milestones and rise as a space power
 India decided to go to space when Indian National Committee for Space
Research (INCOSPAR) was set up by the Government of India in 1962. With
the visionary Dr Vikram Sarabhai at its helm, INCOSPAR set up the Thumba
Equatorial Rocket Launching Station (TERLS) in Thiruvananthapuram for
upper atmospheric research.
 First Indian-made satellite was the Rohini-75 (RH-75). It was launched
from TERLS in 1967.
 ISRO built its first satellite in 1975 and named it Aryabhatta. This was
launched by the Soviet Union.
 The first Indian-built launch vehicle was SLV-3 and it was used to launch
the Rohini satellite in 1980.
 ISRO launched the first INSAT in 1988. It was a communication satellite.
 ISRO also launched the first IRS (remote-sensing satellite) in 1988.
 ISRO has developed three types of launch vehicles (or rockets) namely, the
PSLV (Polar Satellite Launch Vehicle), the GSLV (Geosynchronous Satellite
Launch Vehicle) and Geosynchronous Satellite Launch Vehicle Mark III
(GSLV Mark III or LVM).
 ISRO launched its first lunar mission Chandrayaan I in 2008.
 It also launched the Mars Orbiter Mission or the Mangalyaan in 2014. With
this, India became the first country to achieve success in putting a satellite
on the Mars orbit in its maiden attempt and the fourth space agency and the
first Asian agency to do so.
 In 2017, ISRO created another world record by launching 104 satellites in a
single rocket. It launched its heaviest rocket yet, the Geosynchronous
Satellite Launch Vehicle-Mark III and placed the GSAT 19 in orbit. There are
future plans for a human spaceflight (Gaganyaan), interplanetary probes
and a solar mission as well.
Refer: https://www.isro.gov.in/about-isro/isros-timeline-1960s-to-today

Telegram: https://t.me/insightsIAStips
62
Youtube: https://www.youtube.com/channel/UCpoccbCX9GEIwaiIe4HLjwA
Revision Through MCQs (RTM) Compilation (November 2019)

RTM- REVISION THROUGH MCQS - 6th -Nov-2019

41. With reference to Indian History, consider the following statements


1. First Battle of Panipat laid the foundation of the Mughal Empire in India
2. Third Battle of Panipat fought between Maratha forces and invading armies
of Iran general Abdali
Which of the given above statements is/are correct?
(a) 1 only
(b) 2 only
(c) Both 1 and 2
(d) Neither 1 nor 2
Ans: (a)
Explanation:
 First Battle of Panipat, in 1526, laid the foundation of the Mughal
Empire in India after its first ruler, Babur, ended the Delhi Sultanate,
which at the time was led by the Lodi dynasty.
 Third Battle of Panipat: Fought between Maratha forces and invading
armies of Afghan (not Iran) general Ahmed Shah Abdali of Durrani
Empire in 1761.
Refer: https://www.insightsonindia.com/2019/11/06/third-battle-of-panipat/

42. With reference to Alzheimer’s disease, consider the following statements


1. Alzheimer’s disease is a non-communicable disease
2. There is no cure/treatment available for Alzheimer’s disease
Which of the given above statements is/are correct?
(a) 1 only
(b) 2 only
(c) Both 1 and 2
(d) Neither 1 nor 2
Ans: (c)
Explanation:
 Non communicable diseases (NCDs) include a range of chronic
conditions, including cancer, diabetes, cardiovascular disease,
hypertension, as well as Alzheimer's and other dementias. They are
commonly thought of as "diseases of affluence".
 There is no cure for Alzheimer’s, because its exact causes are not
known. Most drugs being developed try to slow down or stop the
progression of the disease.
Refer: https://www.insightsonindia.com/2019/11/06/china-proposes-to-treat-
alzheimers-with-new-drug/

43. Recently JCPOA has been in news sometimes commonly referred to


(a) USA Nuclear Deal
(b) Iran Nuclear Deal
(c) Iraq Nuclear Deal
(d) Both A and C
Ans: (b)
Explanation:
 The Joint Comprehensive Plan of Action, known commonly as the Iran
nuclear deal or Iran deal, is an agreement on the Iranian nuclear

Telegram: https://t.me/insightsIAStips
63
Youtube: https://www.youtube.com/channel/UCpoccbCX9GEIwaiIe4HLjwA
Revision Through MCQs (RTM) Compilation (November 2019)

program reached in Vienna on July 14, 2015, between Iran and the P5+1
together with the European Union
Refer: https://www.insightsonindia.com/2019/11/06/iran-nuclear-deal/

44. With reference to Paris Agreement, consider the following statements


1. Global stocktake established under the Paris Agreement, it takes place every
five years
2. Nationally determined contributions should be reported every five years and
are to be registered by the UNFCCC Secretariat
3. India pledged to reduce the emissions intensity of its GDP by 33 to 35 per
cent by 2030 from 2005 level as NDC
Which of the above given statements is/are correct?
(a) 1 and 3
(b) 3 only
(c) 2 and 3
(d) All of the above
Ans: (d)
Explanation:
 Global stocktake established under Article 14 of the Paris Agreement is
a process for taking stock of collective progress toward achieving the
purpose of the Agreement and its long-term goals. It takes place every
five years.
 Contributions each individual country should make to achieve the
worldwide goal are determined by all countries individually and are called
nationally determined contributions (NDCs). Article 3 requires them to
be "ambitious", "represent a progression over time" and set "with the view
to achieving the purpose of this Agreement". The contributions should
be reported every five years and are to be registered by the UNFCCC
Secretariat.
Salient features of India's INDC
 To put forward and further propagate a healthy and sustainable way of
living based on traditions and values of conservation and moderation.
 To adopt a climate-friendly and a cleaner path than the one followed
hitherto by others at corresponding level of economic development.
 To reduce the emissions intensity of its GDP by 33 to 35 per cent by
2030 from 2005 level.
 To achieve about 40 per cent cumulative electric power installed capacity
from non-fossil fuel based energy resources by 2030, with the help of
transfer of technology and low cost international finance, including from
Green Climate Fund.
 To create an additional carbon sink of 2.5 to 3 billion tonnes of CO2
equivalent through additional forest and tree cover by 2030.
 To better adapt to climate change by enhancing investments in
development programmes in sectors vulnerable to climate change,
particularly agriculture, water resources, Himalayan region, coastal
regions, health and disaster management.
 To mobilize domestic and new and additional funds from developed
countries to implement the above mitigation and adaptation actions in
view of the resource required and the resource gap.

Telegram: https://t.me/insightsIAStips
64
Youtube: https://www.youtube.com/channel/UCpoccbCX9GEIwaiIe4HLjwA
Revision Through MCQs (RTM) Compilation (November 2019)

 To build capacities, create domestic framework and international


architecture for quick diffusion of cutting edge climate technology in
India and for joint collaborative R&D for such future technologies.
Refer: https://www.insightsonindia.com/2019/11/06/us-exiting-the-paris-
agreement/
http://vikaspedia.in/energy/environment/climate-change/india2019s-intended-
nationally-determined-contribution

45. With reference to Indian Economy, consider the following statements


1. Trade deficit represents an outflow of domestic currency to foreign markets
2. In the long run, Trade deficit may lead to fewer jobs
3. Trade deficit signifies demand in the domestic economy is not being met by
the domestic producers
Which of the above given statements is/are correct?
(a) 1 and 2
(b) 1 and 3
(c) 2 and 3
(d) All of the above
Ans: (d)
Explanation:
 Trade deficit is an economic measure of international trade in which a
country's imports exceed its exports. A trade deficit represents an
outflow of domestic currency to foreign markets. It is also referred to
as a negative balance of trade (BOT).
 Trade Deficit = Total Value of Imports – Total Value of Exports
 A trade deficit means broadly can mean two things:
o The demand in the domestic economy is not being met by the
domestic producers.
o Many a time a deficit signifies the lack of competitiveness of the
domestic industry
 In the long run, however, a trade deficit may lead to fewer jobs. If the
country is importing more goods from foreign companies, prices will
decline, and domestic companies may be unable to produce items that
compete with lower costs. Manufacturing companies are usually hit the
hardest when a country imports more than it exports. This impact results
in fewer jobs or lower incomes for employees, due to the competition from
imports. Fewer jobs mean fewer products are produced in the economy
which, in turn, leads to even more imports and a greater deficit.
Refer: https://www.insightsonindia.com/2019/11/06/what-is-trade-deficit/

46. Recently Wasteland Atlas has been released by


(a) Ministry of Home Affairs
(b) Ministry of Rural Development
(c) Ministry of Environment, Forest and Climate Change
(d) National Institution for Transforming India
Ans: (b)
Explanation:
Refer: https://www.insightsonindia.com/2019/11/06/wasteland-atlas/

Telegram: https://t.me/insightsIAStips
65
Youtube: https://www.youtube.com/channel/UCpoccbCX9GEIwaiIe4HLjwA
Revision Through MCQs (RTM) Compilation (November 2019)

47. “It lies about 125 m below sea level, and is one of the hottest and most
inhospitable places on Earth— temperatures average 34.5 Celsius but have been
recorded above 50 Celsius. Numerous sulfur springs, volcanoes, geysers, acidic
pools, vast salt pans, and colorful mineral-laden lakes dot the area, which formed
above the divergence of three tectonic plates. Volcanic activity heats spring water,
bringing sulfur and iron to the surface, leaving behind yellow, green, and orange
deposits. For centuries, locals have been trekking in with camel caravans to mine
the salt by hand, and in recent years, a few have been guiding tourists into the
alien-looking landscape”
The above given passage describes which of the following geographic location?
(a) Ethiopia's Danakil Depression
(b) Eastern part of Sahar Desert
(c) Somalia’s Guban Desert
(d) Libyan Desert
Ans: (a)
Explantion:

Refer: Facts for prelims:


https://www.insightsonindia.com/2019/11/06/insights-daily-current-affairs-
pib-06-november-2019/

48. With reference to Super Conductivity, consider the following statements


1. It is achieved when electric resistance in a medium becomes zero
2. A superconductor can carry a current indefinitely without losing any energy
3. 100 times more magnetic field is generated in a superconductor as
compared to a normal conductor.
Which of the above given statements is/are correct?
(a) 1 and 2
(b) 2 and 3
(c) 1 and 3
(d) All of the above
Ans: (a)
Explanation:
 Superconductors are materials that conduct electricity with no
resistance.
 Two fundamental properties of a superconductor:

Telegram: https://t.me/insightsIAStips
66
Youtube: https://www.youtube.com/channel/UCpoccbCX9GEIwaiIe4HLjwA
Revision Through MCQs (RTM) Compilation (November 2019)

o Zero resistance to electrical current.


o Diamagnetism
 They also have several other very important properties, such as the
fact that no magnetic field can exist within a superconductor
 Diamagnetism is a property opposite to normal magnetism that we are
used to. A diamagnetic substance repels an external magnetic field, in
sharp contrast to normal magnetism, or ferromagnetism, under which a
substance is attracted by an external magnetic field.
Refer: https://www.insightsonindia.com/2019/06/22/superconductivity/

49. Which of the following established the network of Population Research Centres
across India
(a) NITI Aayog
(b) MoH&FW
(c) MHRD
(d) MHA
Ans: (b)
Explanation:
 Ministry of Health and Family Welfare (MoHFW) established a
network of Population Research Centres (PRCs) with the mandate to
provide, inter alia, critical research based inputs related to the Health
and Family Welfare Programs and Policies at the national and state
levels.
 Their main role is to undertake research projects relating to family
planning, demographic research and biological studies & qualitative
aspect of population control, with a view to gainfully utilize the feedback
from these research studies for plan formulation, strategies and
modifications of on- going schemes.
Refer: https://www.insightsonindia.com/2019/06/03/population-research-
centres-prcs/

50. With reference to welfare policies in India, consider the following statements
1. Bhavantar Bhugtan Yojana is a scheme of the Government of Odisha
2. Pradhan Mantri Kisan Samman Nidhi is a Central Sector scheme
3. Rythu Bandhu scheme of Andhra Pradesh provides ₹4,000 per acre for every
season to all the farmers of the state
Which of the given above statements is/are not correct?
(a) 2 only
(b) 1 and 3
(c) 2 and 3
(d) All of the above
Ans: (b)
Explanation: Here Directive word is Not Correct!!
 Bhavantar Bhugtan Yojana in Madhya Pradesh was sought to provide
relief to farmers by providing the differential between MSPs and market
prices.
 PM- KISAN Scheme. Pradhan Mantri Kisan Samman Nidhi (PM-KISAN)
is a Central Sector scheme with 100% funding from Government of
India.

Telegram: https://t.me/insightsIAStips
67
Youtube: https://www.youtube.com/channel/UCpoccbCX9GEIwaiIe4HLjwA
Revision Through MCQs (RTM) Compilation (November 2019)

 Rythu Bandhu scheme of the Telangana government provides ₹4,000


per acre for every season to all the farmers of the state. Similar initiatives
have also be framed in Jharkhand and Odisha.
Refer: https://www.insightsonindia.com/2019/06/03/pm-kisan-scheme-2/

RTM- REVISION THROUGH MCQS - 7th -Nov-2019

51. With reference to Indian Constitution, consider the following statements


1. President can disqualify MLAs for holding an ‘office of profit’
2. Office of profit seeks to enforce the principle of separation of power between
the legislature and the executive
3. State legislatures have power to enact laws for exempting certain offices from
the purview of office of profit
Which of the given above statements is/are correct?
(a) 2 and 3
(b) 2 only
(c) 3 only
(d) 1, 2 and 3
Ans: (d)
Explanation:
 In 2018, Following the recommendation of the Election Commission
(EC), the President disqualified 20 MLAs of the Delhi Legislative
Assembly last month for holding an ‘office of profit’. The legislators in
question were appointed as parliamentary secretaries to various
ministries in the Delhi government.
 MPs and MLAs, as members of the legislature, hold the government
accountable for its work. The essence of disqualification under the office
of profit law is if legislators holds an ‘office of profit’ under the
government, they might be susceptible to government influence, and
may not discharge their constitutional mandate fairly. The intent is that
there should be no conflict between the duties and interests of an
elected member. Hence, the office of profit law simply seeks to
enforce a basic feature of the Constitution- the principle of
separation of power between the legislature and the executive.
 What does Constitution says about holding an Office of Profit?
o Under the provisions of Article 102 (1) and Article 191 (1) of the
Constitution, an MP or an MLA (or an MLC) is barred from
holding any office of profit under the central or state government.
The articles clarify that “a person shall not be deemed to hold an
office of profit under the government of India or the government of
any state by reason only that he is a minister”. The Constitution
specifies that the number of ministers including the Chief
Minister has to be within 15% of the total number of
members of the assembly (10% in the case of Delhi, which is
a union territory with legislature).
o Provisions of Articles 102 and 191 also protect a legislator
occupying a government position if the office in question has been
made immune to disqualification by law. In the recent past,
several state legislatures have enacted laws exempting

Telegram: https://t.me/insightsIAStips
68
Youtube: https://www.youtube.com/channel/UCpoccbCX9GEIwaiIe4HLjwA
Revision Through MCQs (RTM) Compilation (November 2019)

certain offices from the purview of office of profit. Parliament


has also enacted the Parliament (Prevention of Disqualification)
Act, 1959, which has been amended several times to expand the
exempted list.
Refer: https://www.insightsonindia.com/2019/11/07/office-of-profit-2/

52. With reference to Bay of Bengal Initiative for Multi-Sectoral Technical and
Economic Cooperation (BIMSTEC), consider the following statements
1. BIST-EC was founded after Asian Financial crisis
2. 5 member nations of BIMSTEC are also member of SAARC
3. BIMSTEC Permanent Secretariat is at Katmandu
4. In 2004, Nepal and Bhutan become full members
Which of the above given statements is/are correct?
(a) 1, 2 and 4
(b) 2 and 4
(c) 2, 3 and 4
(d) 1, 2, 3 and 4
Ans: (b)
Explanation:
 On 6 June 1997, a new sub-regional grouping was formed in Bangkok
under the name BIST-EC (Bangladesh, India, Sri Lanka, and Thailand
Economic Cooperation). Following the inclusion of Myanmar on 22
December 1997 during a special Ministerial Meeting in Bangkok, the
Group was renamed ‘BIMST-EC’ (Bangladesh, India, Myanmar, Sri Lanka
and Thailand Economic Cooperation). In 1998, Nepal became an
observer. In February 2004, Nepal and Bhutan become full members.
 Permanent Secretariat located in Dhaka, Bangladesh.
 On 31 July 2004, in the first Summit the grouping was renamed as
BIMSTEC or the Bay of Bengal Initiative for Multi-Sectoral Technical and
Economic Cooperation.
 The Asian financial crisis was a period of financial crisis that gripped
much of East Asia and Southeast Asia beginning in July 1997 and
raised fears of a worldwide economic meltdown due to financial
contagion.
 BIMSTEC V/S SAARC

Telegram: https://t.me/insightsIAStips
69
Youtube: https://www.youtube.com/channel/UCpoccbCX9GEIwaiIe4HLjwA
Revision Through MCQs (RTM) Compilation (November 2019)

 India, Bangladesh, Bhutan, Nepal, Sri Lanka are common members of


both SAARC and BIMSTEC
Refer: https://www.insightsonindia.com/2019/11/07/bimstec-2/

53. What do you mean by Core investment companies?


(a) CICs are non-banking financial companies
(b) CICs are special purpose vehicles with asset size of less than ₹100 crore
(c) CICs are non-banking financial companies with asset size of less than ₹500
crore
(d) CICs are non-banking financial companies with asset size of ₹100 crore and
above
Ans: (d)
Explanation:
 CICs are non-banking financial companies with asset size of ₹100 crore
and above which carry on the business of acquisition of shares and
securities, subject to certain conditions.
 CICs, which are allowed to accept public funds, hold not less than 90% of
their net assets in the form of investment in equity shares, preference
shares, bonds, debentures, debt or loans in group companies.
 Exemption: CICs having asset size of below Rs 100 crore are exempted
from registration and regulation from the RBI, except if they wish to
make overseas investments in the financial sector.
Refer: https://www.insightsonindia.com/2019/11/07/core-investment-companies-
cics/

54. Which of the following fund is not included under SEBI regulated Alternative
investment fund?
(a) Angel Funds
(b) Mutual Funds

Telegram: https://t.me/insightsIAStips
70
Youtube: https://www.youtube.com/channel/UCpoccbCX9GEIwaiIe4HLjwA
Revision Through MCQs (RTM) Compilation (November 2019)

(c) SME Funds


(d) Social Venture Funds
Ans: (b)
Explanation:
 Alternative Investment Fund or AIF means any fund
established or incorporated in India which is a privately pooled
investment vehicle which collects funds from sophisticated investors,
whether Indian or foreign, for investing itin accordance with a defined
investment policy for the benefit of its investors
 AIF does not include funds covered under the SEBI
(Mutual Funds) Regulations, 1996, SEBI (Collective Investment
Schemes) Regulations, 1999 or any other regulations of the Board to
regulate fund management activities.
 Category I AIF:
o Venture capital funds (Including Angel Funds)
o SME Funds
o Social Venture Funds
o Infrastructure funds
Refer: https://www.insightsonindia.com/2019/11/07/alternative-investment-
funds-aifs/

55. “Sarvajna and Tiruvalluvar are popular Kannada and Tamil poets, respectively.
Statues of Sarvajna in Tamil Nadu and Tiruvalluvar in Karnataka has been
unveiled in August 2009 as a symbolic effort to bolster ties between the two
Indian states, whose relationship has been strained by issues related to sharing
of Kaveri water and Hogenakkal water supply power project”
With reference to Sarvajna and Tiruvalluvar mentioned in the above passage,
consider the following statements
1. Sarvajna and Tiruvalluvar were contemporaries
2. Sarvajna famous for his pithy three-lined poems called tripadi
3. Manimekalai is a Tamil epic composed by Tiruvalluvar
Which of the given above statements is/are correct?
(a) 1 and 3
(b) 2 only
(c) 3 only
(d) 1, 2 and 3
Ans: (b)
Explanation:
 Sarvajna was a sixteenth-century poet in the Kannada language.
 He is famous for his pithy three-lined poems which are called tripadis,
"with three padas, three-liners", a form of Vachanas.
 He is also referred to as Sarvagna in modern translation.
 In all, about 2000 three-liners are attributed to Sarvajna.
 Popular because of their alliterative structure and simplicity, they deal
mainly with social, ethical and religious issues. A number of riddles are
also attributed to Sarvajna.
 Sattanar or Chithalai Sathanar was the Tamil poet who composed the
epic Manimekalai.
 Thiruvalluvar is a celebrated Tamil poet and philosopher whose
contribution to Tamil literature is the Thirukkural, a work on ethics. He

Telegram: https://t.me/insightsIAStips
71
Youtube: https://www.youtube.com/channel/UCpoccbCX9GEIwaiIe4HLjwA
Revision Through MCQs (RTM) Compilation (November 2019)

is thought to have lived sometime between the 4th century BC and the
1st century BC.
Refer: Facts for prelims: https://www.insightsonindia.com/2019/11/07/insights-
daily-current-affairs-pib-07-november-2019/

56. Recently MHRD has launched Shaala Darpan portal for


(a) Navodaya Vidyalaya Samiti (NVS)
(b) Central Board of Secondary Education(CBSE)
(c) Vidya Bharati of Rashtriya Swayamsevak Sangh (RSS)
(d) Morarji Desai Residential Schools
Ans: (a)
Explanation:
 Value addition: RTE not applicable to minority schools: SC
o Minority-run schools cannot be forced to implement the Right to
Education Act, 2009, that mandates 25% reservation for
economically disadvantaged children in all schools, the Supreme
Court ruled.
Refer: Facts for prelims: https://www.insightsonindia.com/2019/11/07/insights-
daily-current-affairs-pib-07-november-2019/

57. Which of the following is the World’s southernmost city?


(a) Puerto Williams, Chile
(b) Puerto Williams, Argentina
(c) Puerto Williams, Falkland island
(d) Puerto Williams, Antarctica
Ans: (a)
Explanation:
 Puerto Williams in Chile has become world’s southernmost city after its
status upgraded from hamlet to city. It is the main settlement, port and
naval base on Navarino Island in Chile.
 It took over the title from Ushuaia in Argentina which was world’s
southernmost city till recently.
 Puerto Williams faces the Beagle Channel, which is a strait in Tierra del
Fuego Archipelago (shared by Chile and Argentina) on South America’s
extreme southern tip.
Refer: Facts for Prelims: https://www.insightsonindia.com/2019/06/03/insights-
daily-current-affairs-pib-03-june-2019/

58. Cotigao Wildlife sanctuary is located in


(a) Goa
(b) Karnataka
(c) Maharashtra
(d) Assam
Ans: (a)
Explanation:
 Scientists have recently identified a new species of wasp from the genus
Kudakrumia in Goa. The new Wasp species was collected in the Western
Ghats forests from Cotigao Wildlife sanctuary.
 Cotigao Wildlife Sanctuary is located in Canacona Taluka, South Goa
district, of Goa, India, established in 1968.

Telegram: https://t.me/insightsIAStips
72
Youtube: https://www.youtube.com/channel/UCpoccbCX9GEIwaiIe4HLjwA
Revision Through MCQs (RTM) Compilation (November 2019)

Refer: Facts for Prelims: https://www.insightsonindia.com/2019/06/03/insights-


daily-current-affairs-pib-03-june-2019/

59. “It is the largest active volcano in Europe and one of the world’s most frequently
erupting volcanoes. It is also the volcano with the longest record of continuous
eruption. The mountain’s largest feature is the Valle del Bove (Valley of the Ox), a
large horseshoe-shaped caldera on the eastern slope. In June 2013, it was added
to the list of UNESCO World Heritage Sites. Due to its history of recent activity
and nearby population, it has been designated a Decade Volcano by the United
Nations”
The above given passage refers which of the following volcano
(a) Mount Etna
(b) Mount Fuji
(c) Mayon Volcano
(d) Mount Stromboli
Ans: (a)
Explanation:
 Mount Etna is the largest active volcano in Europe and one of the
world’s most frequently erupting volcanoes. It is also the volcano with the
longest record of continuous eruption. Located near the east coast of the
island of Sicily in Italy, Mount Etna is 10,900 feet tall.

Refer: Facts for Prelims: https://www.insightsonindia.com/2019/06/04/insights-


daily-current-affairs-pib-04-june-2019/

60. Recently ‘UdChalo’ has been in news sometimes is related to


(a) Travel portal for the personal travel of the military and paramilitary forces
(b) Skill development training for disables
(c) Happy curriculum program for children
(d) New portal for STEM program
Ans: (a)
Explanation:

Telegram: https://t.me/insightsIAStips
73
Youtube: https://www.youtube.com/channel/UCpoccbCX9GEIwaiIe4HLjwA
Revision Through MCQs (RTM) Compilation (November 2019)

 ‘UdChalo’ is a travel portal that caters for the personal travel of the
military and paramilitary forces personnel by aggregating defence fares
and getting exclusive discounts.
 It aims to empower the disabled military veterans. The initiative is
unique and has given a new lease of life of these soldiers who are now
confined to wheelchairs.
Refer: Facts for Prelims: https://www.insightsonindia.com/2019/06/04/insights-
daily-current-affairs-pib-04-june-2019/

RTM- REVISION THROUGH MCQS - 8th -Nov-2019

61. Which of the following are not necessarily the consequences of the
proclamation of the President’s rule in a State?
1. Dissolution of the State Legislative Assembly
2. Removal of the Council of Ministers in the State
3. Dissolution of the local bodies
Select the correct answer using the code given below:
(a) 1 and 2 only
(b) 1 and 3 only
(c) 2 and 3 only
(d) 1, 2 and 3
Ans: (b)
Explanation:
 When the President’s Rule is imposed in a state, the President dismisses
the state council of ministers headed by the chief minister. The state
governor, on behalf of the President, carries on the state administration
with the help of the chief secretary of the state or the advisors appointed
by the President. This is the reason why a proclamation under Article
356 is popularly known as the imposition of ‘President’s Rule’ in a state.
 Further, the President either suspends or dissolves the state
legislative assembly.
 The Parliament passes the state legislative bills and the state budget.
 The President's Rule does not lead to dissolution of local bodies.

Telegram: https://t.me/insightsIAStips
74
Youtube: https://www.youtube.com/channel/UCpoccbCX9GEIwaiIe4HLjwA
Revision Through MCQs (RTM) Compilation (November 2019)

Refer: https://www.insightsonindia.com/2019/11/08/what-next-in-maharashtra/

62. Which one of the following suggested that the Governor should be an eminent
person from outside the State and should be a detached figure without intense
political links or should not have taken part in politics in the recent past?
(a) First Administrative Reforms Commission (1966)
(b) Rajamannar Committee(1969)
(c) Sarkaria Commission (1983)
(d) National Commission to Review the Working of the Constitution(2000)
Ans: (c)
Explanation: On the issue of appointment of the Governors, Sarkaria
Commission (1983) made some important recommendations as given in the
following:
 The Governor should be eminent in some walk of life and from outside
the state. He should be a detached figure without intense political links
or should not have taken part in politics in recent past. Besides, he
should not be a member of the ruling party.
 He should be appointed after effective consultations with the state Chief
Minister and Vice President and Speaker of the Lok Sabha should be
consulted by the PM before his selection.
 As far as possible, the governor should enjoy the term of five years.
 He should be removed before his tenure only on the grounds as
mentioned in the constitution or if aspersions are cast on his morality,
dignity, constitutional propriety, etc.

Telegram: https://t.me/insightsIAStips
75
Youtube: https://www.youtube.com/channel/UCpoccbCX9GEIwaiIe4HLjwA
Revision Through MCQs (RTM) Compilation (November 2019)

 In the process of removal, state government may be informed and


consulted.
Refer: https://www.insightsonindia.com/2019/11/08/what-next-in-maharashtra/

63. With reference to Government Policies on Energy, Consider the following


statements
1. National Policy on Biofuels-2018 envisages target of 20% blending of ethanol
in petrol and 5% blending of bio-diesel in diesel by 2030.
2. Government has reduced GST on ethanol for blending in fuel from 18% to
5%
3. Repurpose Used Cooking Oil (RUCO) launched by MNRE aims for an
ecosystem that will enable the collection and conversion of used cooking oil
to biodiesel.
Which of the following above statements is true?
(a) 1 and 2
(b) 2 and 3
(c) 1 and 3
(d) 1, 2 and 3
Ans: (a)
Explanation:
 Goal of the National Policy on Biofuels-2018 is to enable
availability of biofuels in the market thereby increasing its blending
percentage.
 Currently the ethanol blending percentage in petrol is around 2.0%
and biodiesel blending percentage in diesel is less than 0.1%.
 An indicative target of 20% blending of ethanol in petrol and 5%
blending of biodiesel in diesel is proposed by 2030. This goal is to be
achieved by
o Reinforcing ongoing ethanol/biodiesel supplies through increasing
domestic production
o Setting up Second Generation (2G) bio refineries
o Development of new feedstock for biofuels
o Development of new technologies for conversion to biofuels.
o Creating suitable environment for biofuels and its integration with
the main fuels
 Goods and Services Tax Council has lowered the applicable tax rates
across alternative fuels.
o The tax rate on lithium ion batteries has been lowered to 18 per
cent from 28 per cent in a bid to boost prospects of electric
vehicles. Tax on fuel cell vehicles has been lowered to 12 per cent
from 28 per cent.
o The GST rate on ethanol targeted for blending with petrol has
also been lowered from 18 per cent to 5 per cent, the tax on
bio fuel pellets has been fixed at five per cent.
 Repurpose Used Cooking Oil (RUCO) launched by Food Safety and
Standards Authority of India (FSSAI) aims for an ecosystem that will
enable the collection and conversion of used cooking oil to biodiesel.

Refer: https://www.insightsonindia.com/2019/11/08/why-govt-is-encouraging-
ethanol-production/

Telegram: https://t.me/insightsIAStips
76
Youtube: https://www.youtube.com/channel/UCpoccbCX9GEIwaiIe4HLjwA
Revision Through MCQs (RTM) Compilation (November 2019)

64. Recently India Justice Report (IJR)-first-ever ranking of Indian states on justice
delivery has been released. Which is prepared by
(a) Bar council of India
(b) Association of all India lawyers
(c) Tata trusts with Daksh
(d) Supreme courts Centre for Research and Planning (CRP)
Ans: (c)
Explanation:
 India Justice Report (IJR) has been released. It has been prepared by
Tata Trusts in collaboration with Centre for social Justice, Common
Cause, Commonwealth Human Rights Initiative, Daksh, TISS-Prayas and
Vidhi Centre for Legal Policy.
Refer: https://www.insightsonindia.com/2019/11/08/india-justice-report-ijr/

65. With reference to Feni River, consider the following statements


1. It is a transnational river
2. It originates in Tripura
3. Recently there is a river sharing dispute between Tripura state and west
Bengal
Which of the given above statements is/are correct?
(a) 1 and 3
(b) 2 and 3
(c) 1 and 2
(d) 1, 2 and 3
Ans: (c)
Explanation:
 The Feni River forms part of the India-Bangladesh border.
 It originates in the South Tripura district, passes through Sabroom town
on the Indian side, and meets the Bay of Bengal after it flows into
Bangladesh.
 The dispute:
o There has been no water-sharing agreement between the countries
on the Feni previously.
o The dispute over the sharing of the river water has been long-
standing. It was taken up between India and Pakistan (before the
independence of Bangladesh) in 1958 during a Secretary-level
meeting in New Delhi.
Refer: https://www.insightsonindia.com/2019/11/08/feni-river-and-its-
significance/

66. Which of the following is/are not a Credit Rating Agency in India
1. Brickwork Ratings
2. SMERA
3. Moody
Find the appropriate answer using the code below:
(a) 3 only
(b) 1 and 3
(c) 2 and 3
(d) All of the above
Ans: (a)
Explanation:

Telegram: https://t.me/insightsIAStips
77
Youtube: https://www.youtube.com/channel/UCpoccbCX9GEIwaiIe4HLjwA
Revision Through MCQs (RTM) Compilation (November 2019)

 As of now, there are six credit rating agencies registered under SEBI
namely, CRISIL, ICRA, CARE, SMERA, Fitch India and Brickwork
Ratings. Ratings provided by these agencies determine the nature and
integrals of the loan.
Refer: https://www.insightsonindia.com/2019/11/08/moodys-ratings/

67. What is common to places known as Barna, Tawa, Matatila and Tilaiya?
(a) Recently discovered uranium deposits
(b) Sites of Sangam age in Tamilnadu
(c) Hots spots of Naxalism in central India
(d) Water reservoirs
Ans: (d)
Explanation:
 At present, around 18 water reservoirs have water level below 50 per
cent of normal storage capacity. Of these, six reservoirs (Sholayar, Lower
Bhawani, Vaigai, Mettur Stanley, Aliyar, Parambikulam) are in Tamil
Nadu, three (Isapur, Yeldari, Pench (Totaladoh) in Maharashtra, two
(Vanivilas Sagar, Tungabhadra) in Karnataka, two (Tawa, Barna) in
Madhya Pradesh, one each in Gujarat (Sardar Sarovar), Uttar Pradesh
(Matatila), Odisha (Balimela), Jharkhand (Tilaiya) and Andhra
Pradesh and Telangana (Nagarjuna Sagar).
Refer: Question framed on this related topic:
https://www.insightsonindia.com/2019/11/08/punjab-preservation-of-subsoil-
water-act-2009/

68. “The name of this community means people living in the woods, these are an
ethnic group living in Thailand, Laos, India and Vietnam. They settled mostly
along waterways. Traditionally they live in small houses that are built on stilts.
The houses are arranged around a central meeting building around a circle. These
tribals are mainly believe in Animism, but some (especially in Thailand), are
adherents of Theravada Buddhism, which is observed along with Animism, which
includes worship of ancestors, the spirits of the rice and fire spirits. Sacred
objects to this community include relics and fragments of ancient weapons and
household objects”
The passage describes which of the following tribal community?
(a) Brus Tribe
(b) Kuki Tribe
(c) Angami Tribe
(d) Garo Tribe
Ans: (a)
Explanation:
 The Brus, also referred to as the Reangs, are spread across the
northeastern states of Tripura, Assam, Manipur, and Mizoram.
 In Tripura, they are recognised as a Particularly Vulnerable Tribal
Group. In Mizoram, they have been targeted by groups that do not
consider them indigenous to the state.
Refer: https://www.insightsonindia.com/2019/11/08/brus-of-mizo/

Telegram: https://t.me/insightsIAStips
78
Youtube: https://www.youtube.com/channel/UCpoccbCX9GEIwaiIe4HLjwA
Revision Through MCQs (RTM) Compilation (November 2019)

69. 2019 edition of SDG Gender Index has been developed by


(a) UNDP
(b) WEF
(c) Equal Measure 2030
(d) Amnesty international
Ans: (c)
Explanation:
 The 2019 edition of SDG Gender Index has been released.
 India ranked 95 among 129 in the index.
 About the SDG Gender Index:
o Developed by Equal Measures 2030, a joint effort of regional and
global organisations including African Women’s Development and
Communication Network, Asian-Pacific Resource and Research
Centre for Women, Bill and Melinda Gates Foundation etc.
 It accounts for 14 out of 17 SDGs (sustainable development goals) that
cover aspects such as poverty, health, education, literacy, political
representation and equality at the workplace.
 A score of 100 reflects the achievement of gender equality in relation to
the targets set for each indicator. It means, for example, that 100% of
girls complete secondary education, or that there is around 50-50 parity
for women and men in Parliament. A score of 50 signifies that a country
is about halfway to meeting a goal.
Refer: https://www.insightsonindia.com/2019/06/04/sdg-gender-index/

70. Arrange the following from South to North direction


1. Jerusalem
2. Baghdad
3. Sana
4. Muscat
Select the correct answer using the code below:
(a) 3-4-2-1
(b) 3-4-1-2
(c) 2-1-4-3
(d) 1-2-4-3
Ans: (b)
Explanation:

Telegram: https://t.me/insightsIAStips
79
Youtube: https://www.youtube.com/channel/UCpoccbCX9GEIwaiIe4HLjwA
Revision Through MCQs (RTM) Compilation (November 2019)

RTM- REVISION THROUGH MCQS - 9th -Nov-2019

71. Which of the following statements is/are true with respect to Steel Scrap
Recycling Policy
1. Policy that aims to reduce imports, conserve resources and save energy
2. Policy is based on 6Rs principles
3. It contains the provision of Extended Producer Responsibility
Select the correct answer using the code below
(a) 1 only
(b) 1 and 2
(c) 2 and 3
(d) 1,2 and 3
Ans: (d)
Explanation:
 In a bid to ensure quality scrap for the steel industry, the government on
Friday came out with a Steel Scrap Recycling Policy that aims to
reduce imports, conserve resources and save energy. The country's
steel scrap imports were valued at Rs 24,500 crore in 2017-18, while the
deficit was to the tune of 7 MT.
 The policy is based on "6Rs principles of Reduce, Reuse, Recycle,
Recover, Redesign and Remanufacture through scientific handling,
processing and disposal of all types of recyclable scraps including non-
ferrous scraps, through authorized centers / facility".
 The policy said that Ministry of Road Transport and Highways (MoRTH)
and the Department of Heavy Industries are working towards ‘Extended
Producer Responsibility’ by requiring the vehicle manufacturers to
incentivise scrapping of unfit vehicles in exchange for price discounts for
purchase of new vehicles.
Refer: https://www.insightsonindia.com/2019/11/09/steel-scrap-recycling-
policy/

Telegram: https://t.me/insightsIAStips
80
Youtube: https://www.youtube.com/channel/UCpoccbCX9GEIwaiIe4HLjwA
Revision Through MCQs (RTM) Compilation (November 2019)

72. Consider the following statements


1. Kalapani, a disputed territory between India and Nepal is located in Uttar
Pradesh
2. According to Indian mythology, Great sage Vyasa meditated at kalapani,
giving the region its name
3. Kalapani is controlled by India's Indo-Tibetan Border Police
Which of the given above statements is/are correct?
(a) 1 only
(b) 2 and 3
(c) 3 only
(d) 1 and 3
Ans: (b)
Explanation:
 Kalapani is a territory disputed between India and Nepal, but under
Indian administration as part of Pithoragarh district in the
Uttarakhand state.
 It is marked by the Kalapani River, one of the headwaters of the Kali
River in the Himalayas at an altitude of 3600 meters. The valley of the
Kalapani forms the Indian route to Kailash–Manasarovar, an ancient
pilgrimage site.
 It is said that the Great Sage Vyasa meditated at this place, giving the
region its name - Vyas Valley.
 A pool by the temple of the Goddess Kali is considered to by some to be
the source of the Kali River.
 Although claimed by Nepal as part of Darchula District, Kalapani is
controlled by India's Indo-Tibetan Border Police since the 1962
border war with China.

Refer: https://www.insightsonindia.com/2019/11/09/kalapani-territory/

73. Which of the following is not a member of Quad countries


(a) Japan
(b) India

Telegram: https://t.me/insightsIAStips
81
Youtube: https://www.youtube.com/channel/UCpoccbCX9GEIwaiIe4HLjwA
Revision Through MCQs (RTM) Compilation (November 2019)

(c) Australia
(d) South Korea
Ans: (d)
Explanation:
 The quadrilateral formation includes Japan, India, United States and
Australia.
 All four nations find a common ground of being the democratic nations
and common interests of unhindered maritime trade and security.

Refer: https://www.insightsonindia.com/2019/11/09/quad-countries-2/

74. “It is the only spacecraft to have visited all four gas giant planets — Jupiter,
Saturn, Uranus and Neptune — and discovered 16 moons, as well as phenomena
like Neptune’s mysteriously transient Great Dark Spot, the cracks in Europa’s ice
shell, and ring features at every planet”
The given above passage describes which of the following Space Mission?
(a) Voyager 1 Mission
(b) Voyager 2 Mission
(c) New Horizon Mission
(d) Cassini-Huygens Mission
Ans: (b)
Explanation:
 Voyager 2 is the only spacecraft to have visited all four gas giant
planets — Jupiter, Saturn, Uranus and Neptune — and discovered 16
moons, as well as phenomena like Neptune’s mysteriously transient
Great Dark Spot, the cracks in Europa’s ice shell, and ring features at
every planet.
 The twin Voyager 1 and 2 spacecraft are exploring where nothing from
Earth has flown before. Continuing on their more-than-40-year journey
since their 1977 launches, they each are much farther away from Earth
and the sun than Pluto. In August 2012, Voyager 1 made the historic
entry into interstellar space, the region between stars, filled with material
ejected by the death of nearby stars millions of years ago. Voyager 2
entered interstellar space on November 5, 2018 and scientists hope to
learn more about this region. Both spacecraft are still sending scientific
information about their surroundings through the Deep Space Network,
or DSN.

Telegram: https://t.me/insightsIAStips
82
Youtube: https://www.youtube.com/channel/UCpoccbCX9GEIwaiIe4HLjwA
Revision Through MCQs (RTM) Compilation (November 2019)

 The primary mission was the exploration of Jupiter and Saturn. After
making a string of discoveries there — such as active volcanoes on
Jupiter's moon Io and intricacies of Saturn's rings — the mission was
extended. Voyager 2 went on to explore Uranus and Neptune, and is still
the only spacecraft to have visited those outer planets. The adventurers'
current mission, the Voyager Interstellar Mission (VIM), will explore the
outermost edge of the Sun's domain. And beyond.
Refer: https://www.insightsonindia.com/2019/11/09/nasas-voyager-2-spacecraft-
2/

75. With reference to Fiber Optic Network, consider the following statements
1. Fiber is preferred over electrical cabling when high bandwidth, long distance,
or immunity to electromagnetic interference are required
2. Optical fiber has large advantages over existing copper wire in long-distance
due to much lower attenuation and interference
3. Fiber optics transmit data in the form of light particles
Which of the given above statements is/are correct?
(a) 1 only
(b) 2 and 3 only
(c) 1 and 2 only
(d) 1, 2 and 3
Ans: (d)
Explanation:
 Optical fiber is used by many telecommunications companies to transmit
telephone signals, Internet communication and cable television signals. It
is also used in a multitude of other industries, including medical,
defense/government, for data storage, and industrial/commercial. In
addition to serving the purposes of telecommunications, it is used as
light guides, for imaging tools, lasers, hydrophones for seismic waves,
SONAR, and as sensors to measure pressure and temperature.
 Due to much lower attenuation and interference, optical fiber has
large advantages over existing copper wire in long-distance, high-
demand applications. However, infrastructure development within cities
was relatively difficult and time-consuming, and fiber-optic systems were
complex and expensive to install and operate. Due to these difficulties,
fiber-optic communication systems have primarily been installed in long-
distance applications, where they can be used to their full transmission
capacity, offsetting the increased cost. The prices of fiber-optic
communications have dropped considerably since 2000.
 Fiber optics transmit data in the form of light particles -- or photons
-- that pulse through a fiber optic cable. The glass fiber core and the
cladding each have a different refractive index that bends incoming light
at a certain angle. When light signals are sent through the fiber optic
cable, they reflect off the core and cladding in a series of zig-zag bounces,
adhering to a process called total internal reflection. The light signals do
not travel at the speed of light because of the denser glass layers, instead
traveling about 30% slower than the speed of light. To renew, or boost,
the signal throughout its journey, fiber optics transmission sometimes
requires repeaters at distant intervals to regenerate the optical signal by
converting it to an electrical signal, processing that electrical signal and
retransmitting the optical signal.

Telegram: https://t.me/insightsIAStips
83
Youtube: https://www.youtube.com/channel/UCpoccbCX9GEIwaiIe4HLjwA
Revision Through MCQs (RTM) Compilation (November 2019)

 Fiber is preferred over electrical cabling when high bandwidth, long


distance, or immunity to electromagnetic interference are required.
This type of communication can transmit voice, video, and telemetry
through local area networks, computer networks, or across long
distances
Refer: https://www.insightsonindia.com/2019/11/09/kerala-fibre-optic-network-
project/

76. No Money For Terror Conference is organised by


(a) FATF
(b) FIU
(c) OECD
(d) Transparency International
Ans: (b)
Explanation:
 India will host the next edition of the “No Money For Terror” conference
to be held in 2020. The announcement was made at the “No Money For
Terror” conference in Melbourne, Australia.
 The “No Money For Terror” conference is organised by Financial
Intelligence Units (FIUs) of over 100 countries, jointly called The
Egmont Group.
Refer: https://www.insightsonindia.com/2019/11/09/no-money-for-terror-
conference/

77. Tiger TRIUMP, It is a first tri-services amphibious exercise between India and
(a) UK
(b) USA
(c) Singapore
(d) Japan
Ans: (b)
Explanation:
 Tiger TRIUMP:
o It is a first tri- services amphibious exercise between India and
the US.
o Conducted in Visakhapatnam and Kakinada, Andhra Pradesh.
o The exercise will help both the countries to exchange knowledge
and to establish professional and personal relationship.
Refer: Facts for Prelims: https://www.insightsonindia.com/2019/11/09/insights-
daily-current-affairs-pib-09-november-2019/

78. Which of the following pairs is/are correctly matched?


1. Cyclone Bul Bul- Bay of Bengal
2. Cyclone Pabuk - South China Sea and Andaman Sea
3. Cyclone Vayu - Arabian Sea
Select the correct answer using the code below
(a) 1 only
(b) 1 and 3
(c) 2 and 3
(d) 1, 2 and 3
Ans: (d)
Explanation:

Telegram: https://t.me/insightsIAStips
84
Youtube: https://www.youtube.com/channel/UCpoccbCX9GEIwaiIe4HLjwA
Revision Through MCQs (RTM) Compilation (November 2019)

 The season has seen Cyclone Pabuk (South China Sea-Andaman Sea),
Cyclone Fani (Bay of Bengal), Cyclone Vayu (Arabian Sea), Cyclone
Hikka (Arabian Sea), Cyclone Kyarr (Arabian Sea), Cyclone Bul Bul (Bay
of Bengal) and Cyclone Maha (Arabian Sea)
Refer: Facts for Prelims: https://www.insightsonindia.com/2019/11/09/insights-
daily-current-affairs-pib-09-november-2019/

79. With reference to Indian Polity, consider the following statements


1. Cabinet committees are extra-constitutional in nature
2. Presently, there are 8 Cabinet Committee under the Transaction of Business
Rules
3. Newly constituted, Cabinet Committee on Employment & Skill Development
will be headed by Prime minister
Which of the given above statements is/are correct?
(a) 1 only
(b) 2 only
(c) 1 and 2 only
(d) 1, 2 and 3
Ans: (d)
Explanation:
 Prime Minister Narendra Modi-led central government has reconstituted
eight key cabinet committees, including appointments committee, the
cabinet committee on economic affairs, and security, among others.
 The eight committees which have been reconstituted include - Cabinet
Committee on Accommodation, Cabinet Committee on Parliamentary
Affairs, Cabinet Committee on Political Affairs, Cabinet Committee on
Security, Cabinet Committee on Investment and Growth and Cabinet
Committee on Employment and Skill Development.
 In the newly-constituted eight cabinet committees, Modi is part of six
committees, barring Cabinet Committee on Accommodation and
Cabinet Committee on Parliamentary Affairs, while Home Minister
Amit Shah is part of all cabinet committees
 The Cabinet Committee are organizations which are instrumental in
reducing the workload of the Cabinet. These committees are extra-
constitutional in nature and are nowhere mentioned in the
Constitution.
Refer: https://www.insightsonindia.com/2019/06/06/cabinet-committees/

80. Recently New START treaty has been in news sometimes is related to
(a) Nuclear arms reduction treaty between the USA and Russia
(b) Nuclear arms reduction treaty between the Turkey and Syria
(c) Nuclear arms reduction treaty between the Iran and Isreal
(d) Nuclear arms reduction treaty between the Saudi Arabia and Yeman
Ans: (a)
Explanation:
 It is a nuclear arms reduction treaty between the United States and the
Russian Federation with the formal name of Measures for the Further
Reduction and Limitation of Strategic Offensive Arms.
 It was signed on 8 April 2010 in Prague, and, after ratification entered
into force on 5 February 2011.

Telegram: https://t.me/insightsIAStips
85
Youtube: https://www.youtube.com/channel/UCpoccbCX9GEIwaiIe4HLjwA
Revision Through MCQs (RTM) Compilation (November 2019)

 New START replaced the Treaty of Moscow (SORT), which was due to
expire in December 2012. Its name is a follow-up to the START I treaty,
which expired in December 2009, the proposed START II treaty, which
never entered into force, and the START III treaty, for which negotiations
were never concluded.
Refer: https://www.insightsonindia.com/2019/06/07/new-start-strategic-arms-
reduction-treaty/

RTM- REVISION THROUGH MCQS -10th -Nov-2019

81. With reference to Thunder and Lightning, consider the following statements
1. Lightning is an electrical energy while thunder is a sound energy
2. Lightning is seen first before one can hear the sound of thunder
3. Lightning is more dangerous and destructive than thunder
Which of the given above statements is/are correct?
(a) 1 and 2
(b) 2 only
(c) 1 and 3 only
(d) 1, 2 and 3
Ans: (d)
Explanation:
 A thunderstorm occurs when there is a raid upward movement of warm,
moist air. As it moves upward, it loses heat and cools then compresses
and form cumulonimbus clouds where air currents form water droplets
and ice particles which collide with each other and build up static energy
which causes thunder and lighting.
 Lightning is that sudden flash of electricity in the sky which can either
be straight or forked. It is very hot, reaching a temperature of up to
54,000 degrees Fahrenheit. It can travel at a speed of up to 140,000
miles per hour.
 Difference Between Thunder and Lightning
o Lightning is an electrical energy while thunder is a sound energy.
o Both occur at the same time during a thunderstorm, but since
light travels faster than sound, lightning is seen first before one
can hear the sound of thunder.
o Lightning is fast and very hot while thunder can usher in heavy
rain and strong winds, but lightning is more dangerous and
destructive than thunder.
o Lightning is formed when water and ice particles collide with
warm, moist air and build up static energy while thunder is
formed by the fast expansion of gases in the electrical charge of
lightning.
Refer: https://www.insightsonindia.com/2019/11/11/how-lightning-strikes-2/

82. Consider the following statements with reference to Ayodhya Dispute


1. Three-domed mosque(Babari) built by Mir Baqi of Mughal kingdom in 1528
is in the Jaunpuri style
2. In this case, Supreme Court invoked doctrine of adverse possession

Telegram: https://t.me/insightsIAStips
86
Youtube: https://www.youtube.com/channel/UCpoccbCX9GEIwaiIe4HLjwA
Revision Through MCQs (RTM) Compilation (November 2019)

3. Ayodya case is the second longest herd case in the history of supreme court
after the landmark Keshvanand Bharti case in 1973
Which of the given above statements is/are correct?
(a) 1 and 2
(b) 1 and 3
(c) 3 only
(d) 1, 2 and 3
Ans: (d)
Explanation:
 The arguments in the matter were the second longest after the
landmark Keshvanand Bharti case in 1973 during which the
proceedings continued for 68 days
 First Mughal Emperor Babar is believed to have constructed Babri
Masjid
 The three-domed mosque built by Mir Baqi commander of Mughal
emperor Babur in 1528 is in the Jaunpuri style
 Doctrine of Adverse Possession
o Under the “doctrine of adverse possession”, under which a person
who is not the original owner becomes the owner because of the
fact that he has been in possession of the property for a minimum
of 12-years, within which the real owner did not seek legal
recourse to oust him.
o Adverse possession is possession of a property – which has to be
continuous, uninterrupted and peaceful.
Refer: https://www.insightsonindia.com/2019/11/11/ayodhya-verdict/

83. Recently HS-code has been in news sometimes was developed by


(a) World Customs Organization
(b) World Trade Organization
(c) Reserve Bank of India
(d) Ministry of Finance
Ans: (a)
Explanation:
 The Harmonised System, or simply ‘HS’, is a six-digit identification code.
Of the six digits, the first two denote the HS Chapter, the next two give
the HS heading, and the last two give the HS subheading.
 Developed by the World Customs Organization (WCO).
 Called the “universal economic language” for goods.
 Recently Ministry of Commerce and Industry has allocated a separate
Harmonised System (HS) code for Khadi.
Refer: https://www.insightsonindia.com/2019/11/11/hs-code/

84. With reference to Fall army worm, consider the following statements
1. It is also known as Spodoptera frugiperda or fruit destroyer
2. It is native of the tropical and sub-tropical of Asia
3. It was reported in India for the first-time in Maharashtra
4. Pest can attack at least 80 types of crops including bajra, jawar, ragi, paddy,
wheat and vegetables
Which of the given above statements is/are correct?
(a) 1 and 4
(b) 1, 2 and 4

Telegram: https://t.me/insightsIAStips
87
Youtube: https://www.youtube.com/channel/UCpoccbCX9GEIwaiIe4HLjwA
Revision Through MCQs (RTM) Compilation (November 2019)

(c) 1, 3 and 4
(d) All of the above
Ans: (a)
Explanation:
 The fall armyworm, also known as Spodoptera frugiperda or fruit
destroyer, loves to eat corn but also plagues many other crops vital to
human food security, such as rice and jowar
 It is a native of the tropical and sub-tropical regions of the Americas.
 First detected in the African continent in 2016. Since then, it has spread
to other countries such as China, Thailand, Malaysia and Sri Lanka.
 The pest can attack at least 80 types of crops including bajra, jawar,
ragi, paddy, wheat and vegetables.
 In India: It was reported in India for the first-time in Karnataka.
Within a span of only six months, almost 50 per cent of the country,
including Mizoram, Maharashtra, Karnataka, Tamil Nadu, Andhra
Pradesh, Chhattisgarh, Madhya Pradesh, Gujarat and West Bengal, has
reported FAW infestations.
Refer: https://www.insightsonindia.com/2019/11/11/fall-armyworm-faw-3/

85. Which of the following is often dubbed as “sea monsters” and largest aquatic
carnivorous reptiles that have ever lived on this earth?
(a) Pliosaur
(b) Plesiosaur
(c) Dinosaur
(d) Caiman
Ans: (a)
Explanation:
 While the Plesiosaurs evolved long necks and small heads, their Pliosaur
cousins went the opposite direction. Their necks were short and their
heads were large, measuring up to 10 feet long. But much like
plesiosaurs, pliosaurs were massive, growing up to 50 feet long and
weighing almost 100,000 pounds
Refer: Facts for Prelims: https://www.insightsonindia.com/2019/11/11/insights-
daily-current-affairs-pib-11-november-2019/

86. With reference to Indian Constitution, consider the following statements


1. President can disqualify MLAs for holding an ‘office of profit’
2. Office of profit seeks to enforce the principle of separation of power between
the legislature and the executive
3. State legislatures have power to enact laws for exempting certain offices from
the purview of office of profit
Which of the given above statements is/are correct?
(a) 2 and 3
(b) 2 only
(c) 3 only
(d) 1, 2 and 3
Ans: (d)
Explanation:
 In 2018, Following the recommendation of the Election Commission
(EC), the President disqualified 20 MLAs of the Delhi Legislative
Assembly last month for holding an ‘office of profit’. The legislators in

Telegram: https://t.me/insightsIAStips
88
Youtube: https://www.youtube.com/channel/UCpoccbCX9GEIwaiIe4HLjwA
Revision Through MCQs (RTM) Compilation (November 2019)

question were appointed as parliamentary secretaries to various


ministries in the Delhi government.
 MPs and MLAs, as members of the legislature, hold the government
accountable for its work. The essence of disqualification under the office
of profit law is if legislators holds an ‘office of profit’ under the
government, they might be susceptible to government influence, and
may not discharge their constitutional mandate fairly. The intent is that
there should be no conflict between the duties and interests of an
elected member. Hence, the office of profit law simply seeks to
enforce a basic feature of the Constitution- the principle of
separation of power between the legislature and the executive.
 What does Constitution says about holding an Office of Profit?
o Under the provisions of Article 102 (1) and Article 191 (1) of the
Constitution, an MP or an MLA (or an MLC) is barred from
holding any office of profit under the central or state government.
The articles clarify that “a person shall not be deemed to hold an
office of profit under the government of India or the government of
any state by reason only that he is a minister”. The Constitution
specifies that the number of ministers including the Chief
Minister has to be within 15% of the total number of
members of the assembly (10% in the case of Delhi, which is
a union territory with legislature).
o Provisions of Articles 102 and 191 also protect a legislator
occupying a government position if the office in question has been
made immune to disqualification by law. In the recent past,
several state legislatures have enacted laws exempting
certain offices from the purview of office of profit. Parliament
has also enacted the Parliament (Prevention of Disqualification)
Act, 1959, which has been amended several times to expand the
exempted list.
Refer: https://www.insightsonindia.com/2019/11/07/office-of-profit-2/

87. With reference to Bay of Bengal Initiative for Multi-Sectoral Technical and
Economic Cooperation (BIMSTEC), consider the following statements
1. BIST-EC was founded after Asian Financial crisis
2. 5 member nations of BIMSTEC are also member of SAARC
3. BIMSTEC Permanent Secretariat is at Katmandu
4. In 2004, Nepal and Bhutan become full members
Which of the above given statements is/are correct?
(a) 1, 2 and 4
(b) 2 and 4
(c) 2, 3 and 4
(d) 1, 2, 3 and 4
Ans: (b)
Explanation:
 On 6 June 1997, a new sub-regional grouping was formed in Bangkok
under the name BIST-EC (Bangladesh, India, Sri Lanka, and Thailand
Economic Cooperation). Following the inclusion of Myanmar on 22
December 1997 during a special Ministerial Meeting in Bangkok, the
Group was renamed ‘BIMST-EC’ (Bangladesh, India, Myanmar, Sri Lanka

Telegram: https://t.me/insightsIAStips
89
Youtube: https://www.youtube.com/channel/UCpoccbCX9GEIwaiIe4HLjwA
Revision Through MCQs (RTM) Compilation (November 2019)

and Thailand Economic Cooperation). In 1998, Nepal became an


observer. In February 2004, Nepal and Bhutan become full members.
 Permanent Secretariat located in Dhaka, Bangladesh.
 On 31 July 2004, in the first Summit the grouping was renamed as
BIMSTEC or the Bay of Bengal Initiative for Multi-Sectoral Technical and
Economic Cooperation.
 The Asian financial crisis was a period of financial crisis that gripped
much of East Asia and Southeast Asia beginning in July 1997 and
raised fears of a worldwide economic meltdown due to financial
contagion.
 BIMSTEC V/S SAARC


 India, Bangladesh, Bhutan, Nepal, Sri Lanka are common members of
both SAARC and BIMSTEC
Refer: https://www.insightsonindia.com/2019/11/07/bimstec-2/

88. What do you mean by Core investment companies?


(a) CICs are non-banking financial companies
(b) CICs are special purpose vehicles with asset size of less than ₹100 crore
(c) CICs are non-banking financial companies with asset size of less than ₹500
crore
(d) CICs are non-banking financial companies with asset size of ₹100 crore and
above
Ans: (d)
Explanation:
 CICs are non-banking financial companies with asset size of ₹100 crore
and above which carry on the business of acquisition of shares and
securities, subject to certain conditions.

Telegram: https://t.me/insightsIAStips
90
Youtube: https://www.youtube.com/channel/UCpoccbCX9GEIwaiIe4HLjwA
Revision Through MCQs (RTM) Compilation (November 2019)

 CICs, which are allowed to accept public funds, hold not less than 90% of
their net assets in the form of investment in equity shares, preference
shares, bonds, debentures, debt or loans in group companies.
 Exemption: CICs having asset size of below Rs 100 crore are exempted
from registration and regulation from the RBI, except if they wish to
make overseas investments in the financial sector.
Refer: https://www.insightsonindia.com/2019/11/07/core-investment-companies-
cics/

89. Which of the following fund is not included under SEBI regulated Alternative
investment fund?
(a) Angel Funds
(b) Mutual Funds
(c) SME Funds
(d) Social Venture Funds
Ans: (b)
Explanation:
 Alternative Investment Fund or AIF means any fund
established or incorporated in India which is a privately pooled
investment vehicle which collects funds from sophisticated investors,
whether Indian or foreign, for investing itin accordance with a defined
investment policy for the benefit of its investors
 AIF does not include funds covered under the SEBI
(Mutual Funds) Regulations, 1996, SEBI (Collective Investment
Schemes) Regulations, 1999 or any other regulations of the Board to
regulate fund management activities.
 Category I AIF:
o Venture capital funds (Including Angel Funds)
o SME Funds
o Social Venture Funds
o Infrastructure funds
Refer: https://www.insightsonindia.com/2019/11/07/alternative-investment-
funds-aifs/

90. “Sarvajna and Tiruvalluvar are popular Kannada and Tamil poets, respectively.
Statues of Sarvajna in Tamil Nadu and Tiruvalluvar in Karnataka has been
unveiled in August 2009 as a symbolic effort to bolster ties between the two
Indian states, whose relationship has been strained by issues related to sharing
of Kaveri water and Hogenakkal water supply power project”
With reference to Sarvajna and Tiruvalluvar mentioned in the above passage,
consider the following statements
1. Sarvajna and Tiruvalluvar were contemporaries
2. Sarvajna famous for his pithy three-lined poems called tripadi
3. Manimekalai is a Tamil epic composed by Tiruvalluvar
Which of the given above statements is/are correct?
(a) 1 and 3
(b) 2 only
(c) 3 only
(d) 1, 2 and 3
Ans: (b)
Explanation:

Telegram: https://t.me/insightsIAStips
91
Youtube: https://www.youtube.com/channel/UCpoccbCX9GEIwaiIe4HLjwA
Revision Through MCQs (RTM) Compilation (November 2019)

 Sarvajna was a sixteenth-century poet in the Kannada language.


 He is famous for his pithy three-lined poems which are called tripadis,
"with three padas, three-liners", a form of Vachanas.
 He is also referred to as Sarvagna in modern translation.
 In all, about 2000 three-liners are attributed to Sarvajna.
 Popular because of their alliterative structure and simplicity, they deal
mainly with social, ethical and religious issues. A number of riddles are
also attributed to Sarvajna.
 Sattanar or Chithalai Sathanar was the Tamil poet who composed the
epic Manimekalai.
 Thiruvalluvar is a celebrated Tamil poet and philosopher whose
contribution to Tamil literature is the Thirukkural, a work on ethics. He
is thought to have lived sometime between the 4th century BC and the
1st century BC.
Refer: Facts for prelims: https://www.insightsonindia.com/2019/11/07/insights-
daily-current-affairs-pib-07-november-2019/

RTM- REVISION THROUGH MCQS - 12th -Nov-2019

91. With reference to Swachh – Nirmal Tat Abhiyaan, consider the following
statements
1. It is a pan India beaches cleaning drive
2. The collected waste under this initiative will be processed as per extant
Waste Management Rules, 2016
Which of the given above statements is/are correct?
(a) 1 only
(b) 2 only
(c) Both 1 and 2
(d) Neither 1 nor 2
Ans: (b)
Explanation:
 To strive to make our beaches clean and create awareness amongst
citizens about the importance of coastal ecosystems, the Ministry of
Environment, Forest and Climate Change (MoEF&CC) are undertaking a
mass cleanliness-cum-awareness drive in 50 identified beaches(not
pan India) under the "Swachh – Nirmal Tat Abhiyaan", from 11th -17th
November, 2019. The identified beaches are in 10 coastal States/Union
Territories (UTs) namely Gujarat, Daman & Diu, Maharashtra, Goa,
Karnataka, Kerala, Tamil Nadu, Puducherry, Andhra Pradesh, and
Odisha. The beaches have been identified after the consultation with the
States/UTs.
 For beach cleaning activities which will be a duration of two hours on
daily basis, a minimum of one Kilometre stretch of the beach shall be
identified. Beach sand cleaning machines shall also be deployed at about
identified 15 beaches. Thereafter collected waste will be processed as
per extant Waste Management Rules, 2016.
 Environment Education Division of the Ministry and Society of
Integrated Coastal Management (SICOM) under the aegis of this
Ministry will be responsible for the overall coordination for the drive in

Telegram: https://t.me/insightsIAStips
92
Youtube: https://www.youtube.com/channel/UCpoccbCX9GEIwaiIe4HLjwA
Revision Through MCQs (RTM) Compilation (November 2019)

50 beaches. Respective State Governments and Central Ministries will


also be actively participating in the beach cleaning drives.
Refer: https://www.insightsonindia.com/2019/11/12/swachh-nirmal-tat-
abhiyaan/

92. With reference to Seed Treaty, consider the following statements


1. Access and benefit sharing is the one of the provision of the treaty
2. Treaty recognizes Farmers’ rights
3. In conformity with seed treaty, India enacted Protection of Plant Varieties and
Farmers’ Rights (PPV&FR) Act, 2001
Which of the given above statements is/are correct?
(a) 1 and 3
(b) 2 only
(c) 1 and 2
(d) All of the above
Ans: (d)
Explanation:
 The Protection of Plant Varieties and Farmers’ Rights (PPV&FR) Act,
2001:
o Enacted by India in 2001 adopting sui generis system.
o It is in conformity with International Union for the Protection of
New Varieties of Plants (UPOV), 1978.
o The legislation recognizes the contributions of both commercial
plant breeders and farmers in plant breeding activity and also
provides to implement TRIPs in a way that supports the specific
socio-economic interests of all the stakeholders including private,
public sectors and research institutions, as well as resource-
constrained farmers.
Refer: https://www.insightsonindia.com/2019/11/12/international-treaty-on-
plant-genetic-resources-for-food-and-agriculture/

93. 2019 Brown to Green Report has been published by


(a) German Watch
(b) New Climate Institute
(c) Climate Action Network International
(d) Climate Transparency
Ans: (d)
Explanation:
 The Climate Change Performance Index (CCPI) is an annual
publication by Germanwatch, the NewClimate Institute, and Climate
Action Network International.
Refer: https://www.insightsonindia.com/2019/11/12/brown-to-green-report-
2019/

94. Consider the following statements


1. Largest natural fresh water lake of India is Dal Lake
2. Chilaka Lake is the largest inland salt water lake of India
3. Largest brackish water lake of India is Sambar Lake
4. Pulicat Lagoon is considered to be the second largest brackish water body in
India
Which of the given above statements is/are not correct?

Telegram: https://t.me/insightsIAStips
93
Youtube: https://www.youtube.com/channel/UCpoccbCX9GEIwaiIe4HLjwA
Revision Through MCQs (RTM) Compilation (November 2019)

(a) 2, 3 and 4
(b) 1, 2 and 3
(c) 2 and 4
(d) All of the above
Ans: (b)
Explanation: Here Directive word is Not Correct!!
 Largest natural fresh water lake of India is Wular lake (260 Sq Km).
o Wular is a natural lake that is a major part of the Jhelum River
basin
 Rajasthan’s Sambhar Lake is the largest inland salt water lake of
India, with the surface area of approximately 200 sq. kilometres.
o Hundreds of birds were found dead at Sambhar Lake in
Rajasthan, India’s biggest saltwater lake, which has been a
favoured destination for migratory birds from Northern Asia and
Siberia.
 Largest brackish water lake of India is Chilka lake (1165 Sq Km).
o Chilka Lake is a brackish water lagoon. It is the largest lake of
India measuring approximately 1165 sq. kilometres. Chilka lake is
thus the largest salt water lake of India.
 Pulicat Lagoon is the second largest brackish water lagoon in India, after
Chilika Lake
Refer: https://www.insightsonindia.com/2019/11/12/dal-lake-area-to-be-eco-
sensitive-zone/

Telegram: https://t.me/insightsIAStips
94
Youtube: https://www.youtube.com/channel/UCpoccbCX9GEIwaiIe4HLjwA
Revision Through MCQs (RTM) Compilation (November 2019)

95. Recently MHRD has launched Shaala Darpan portal for


(a) Navodaya Vidyalaya Samiti (NVS)
(b) Central Board of Secondary Education(CBSE)
(c) Vidya Bharati of Rashtriya Swayamsevak Sangh (RSS)
(d) Morarji Desai Residential Schools
Ans: (a)
Explanation:
 Value addition: RTE not applicable to minority schools: SC
o Minority-run schools cannot be forced to implement the Right to
Education Act, 2009, that mandates 25% reservation for
economically disadvantaged children in all schools, the Supreme
Court ruled.
Refer: Facts for prelims: https://www.insightsonindia.com/2019/11/07/insights-
daily-current-affairs-pib-07-november-2019/

96. Which of the following is the World’s southernmost city?


(a) Puerto Williams, Chile
(b) Puerto Williams, Argentina
(c) Puerto Williams, Falkland island
(d) Puerto Williams, Antarctica
Ans: (a)

Telegram: https://t.me/insightsIAStips
95
Youtube: https://www.youtube.com/channel/UCpoccbCX9GEIwaiIe4HLjwA
Revision Through MCQs (RTM) Compilation (November 2019)

Explanation:
 Puerto Williams in Chile has become world’s southernmost city after its
status upgraded from hamlet to city. It is the main settlement, port and
naval base on Navarino Island in Chile.
 It took over the title from Ushuaia in Argentina which was world’s
southernmost city till recently.
 Puerto Williams faces the Beagle Channel, which is a strait in Tierra del
Fuego Archipelago (shared by Chile and Argentina) on South America’s
extreme southern tip.
Refer: Facts for Prelims: https://www.insightsonindia.com/2019/06/03/insights-
daily-current-affairs-pib-03-june-2019/

97. Cotigao Wildlife sanctuary is located in


(a) Goa
(b) Karnataka
(c) Maharashtra
(d) Assam
Ans: (a)
Explanation:
 Scientists have recently identified a new species of wasp from the genus
Kudakrumia in Goa. The new Wasp species was collected in the Western
Ghats forests from Cotigao Wildlife sanctuary.
 Cotigao Wildlife Sanctuary is located in Canacona Taluka, South Goa
district, of Goa, India, established in 1968.
Refer: Facts for Prelims: https://www.insightsonindia.com/2019/06/03/insights-
daily-current-affairs-pib-03-june-2019/

98. Consider the following about subsidies


1. Subsidies distort price signals and thus demand for goods and services
2. Subsidies are faulty as they do not follow principle of equity
3. Subsidies should be phased out by the government of India
Select the correct answer using the codes below:
(a) 1 and 2 only
(b) 2 only
(c) 1 and 3 only
(d) 1, 2 and 3
Ans: (a)
Explanation:
 Subsidies distort price signals.
 For example, the over use of ground water is a case in point about
subsidies creating more demand and wasteful with drawl of ground
water.
 Subsidies should be targeted to the poor, but in India schemes are
merely universal For example, PM KISAN scheme is regardless of land
size.
 Subsidies cannot be phased out as poor people will not be able to make
ends meet, and it would also violate the general principle of equity as a
fundamental right.

Telegram: https://t.me/insightsIAStips
96
Youtube: https://www.youtube.com/channel/UCpoccbCX9GEIwaiIe4HLjwA
Revision Through MCQs (RTM) Compilation (November 2019)

99. What do you mean by Inward looking trade strategy?


(a) To follow a policy of import substitution
(b) To not be part of the trade system by staying independent and disconnected
from world markets in all aspects.
(c) To enable a greater interaction amongst centre and various parts of India
(d) None of the above
Ans: (a)
Explanation:
 Inward looking trade strategy is also known as import substitution. Its
main aim is to produce goods domestically which are imported to our
nation. Here, the government protects the domestically produced goods
from foreign competition. This policy protects imports in two forms,
tariffs and quota.

100. Consider the following about Total Fertility Rates in India


1. since not many women use methods of reversible contraception, they have
little control over when they start having children, but only seem to have
control over when they stop having children
2. the average number of children born to a women over her lifetime has
dropped below replacement levels for all religious communities barring
Hindus and Muslims
Which of the above statements is/are correct?
(a) 1 only
(b) 2 only
(c) 1 and 2 only
(d) Neither 1 nor 2
Ans: (c)
Explanation:
 Since not many women use methods of reversible contraception, they have
little control over when they start having children, but only seem to have
control over when they stop having children. This could affect other
milestones early on in a woman’s life; for example, women may not get the
same access to employment that men do.
 Replacement-level fertility rate, which is the average number of children
a woman has to have to keep population at a constant size, is 2.1 and
India is closing in with the overall the number of children per woman at
2.3


Telegram: https://t.me/insightsIAStips
97
Youtube: https://www.youtube.com/channel/UCpoccbCX9GEIwaiIe4HLjwA
Revision Through MCQs (RTM) Compilation (November 2019)

RTM- REVISION THROUGH MCQS - 13th -Nov-2019

101. With reference to President Rule, consider the following statements


1. Proclamation imposing President’s Rule in states has been originally
mentioned in the constitution of India
2. Upon the imposition of this rule, Vidhan Sabha is either dissolved or
prorogued
Which of the given above statements is/are correct?
(a) 1 only
(b) 2 only
(c) Both 1 and 2
(d) Neither 1 nor 2
Ans: (c)
Explanation:
 Article 356 of the Constitution of India gives President of India the power
to suspend state government and impose President’s rule of any state in
the country if “if he is satisfied that a situation has arisen in which the
government of the state cannot be carried on in accordance with the
provisions of the Constitution”.
 Upon the imposition of this rule, there would be no Council of Ministers.
The Vidhan Sabha is either dissolved or prorogued.
 The state will fall under the direct control of the Union government, and
the Governor will continue to be head the proceedings, representing the
President of India – who is the Head of the State.
 The imposition of the President’s rule requires the sanction of both the
houses of Parliament.
 If approved, it can go on for a period of six months. However, the
imposition cannot be extended for more than three years, and needs to
be brought before the two houses every six months for approval.
Refer: https://www.insightsonindia.com/2019/11/13/maharashtra-placed-under-
presidents-rule/

102. With reference to discretionary powers of the President of India, consider the
following statements
1. Veto power
2. Right to be informed of all important deliberation of council of ministers
3. Appointment of Prime Minister in case of a hung parliament
Which of the given above statements is/are correct?
(a) 2 only
(b) 1 and 3 only
(c) 2 and 3 only
(d) 1, 2 and 3
Ans: (d)
Explanation: 3 types of discretionary powers are vested in the President of
India:
 The President has veto powers by which he can withhold or refuse to give
assent to Bills (other than Money Bill) passed by the Parliament.
 Constitutionally, the President has a right to be informed of all important
matters and deliberations of the Council of Ministers. The Prime Minister
is obliged to furnish all the information that the President may call for. The

Telegram: https://t.me/insightsIAStips
98
Youtube: https://www.youtube.com/channel/UCpoccbCX9GEIwaiIe4HLjwA
Revision Through MCQs (RTM) Compilation (November 2019)

President often writes to the Prime Minister and expresses his views on
matters confronting the country.
 In case of a Hung Parliament situation when after an election, no leader
has a clear majority in the Lok Sabha, then the President has to use his
own discretion in judging who really may have the support of the majority
or who can actually form and run the government.
Refer: https://www.insightsonindia.com/2019/11/13/maharashtra-placed-under-
presidents-rule/

103. The state level minister of parliament can hold the portfolio during pleasure of
the
(a) Governor of the state
(b) The Prime Minister
(c) The President of India
(d) None of above
Ans: (c)
Explanation:
 Article 75(1): The Prime Minister shall be appointed by the President and
the other Ministers shall be appointed by the President on the advice of
the Prime Minister.
 Article 75(2): The Ministers shall hold office during the pleasure of
the President.
Refer: https://www.insightsonindia.com/2019/11/13/maharashtra-placed-under-
presidents-rule/

104. Which of the following statement is true?


(a) Commutation means exchange of one thing for another
(b) Remissions means awarding a lesser punishment on some special grounds
(c) Respite means the reduction of the amount of sentence without changing its
character
(d) All of the above statements are true
Ans: (a)
Explanation: Clemency powers of the President under article 72:
 It says that the President shall have the power to grant pardons,
reprieves, respites or remissions of punishment or to suspend, remit or
commute the sentence of any person convicted of any offence.
 Pardon–A pardon completely absolves the offender from all sentences
and punishment and disqualifications and places him in the same
position as if he had never committed the offence.
 Commutation– Commutation means exchange of one thing for another.
In simple words to replace the punishment with less severe punishment.
For example for Rigorous imprisonment-simple imprisonment.
 Reprieve– Reprieve means temporary suspension of death sentence. For
example- pending a proceeding for pardon or commutation.
 Respite– Respite means awarding a lesser punishment on some special
grounds. For example- the Pregnancy of women offender.
 Remissions– Remission means the reduction of the amount of sentence
without changing its character, for example, a sentence of 1 year may be
remitted to 6 months
Refer: https://www.insightsonindia.com/2019/11/13/pardoning-powers-of-
president-2/

Telegram: https://t.me/insightsIAStips
99
Youtube: https://www.youtube.com/channel/UCpoccbCX9GEIwaiIe4HLjwA
Revision Through MCQs (RTM) Compilation (November 2019)

105. Which of the following state recently draft a law to protect journalist?
(a) Bihar
(b) Kerala
(c) Odisha
(d) None of the above
Ans: (d)
Explanation:
 A draft bill has been prepared by a committee to safeguard media
persons in Chhattisgarh from harassment, intimidation and violence.
 The committee was set up in March to draft a law to foster an
atmosphere wherein journalists could perform their work fearlessly.
Refer: https://www.insightsonindia.com/2019/11/13/chhattisgarh-moots-law-to-
protect-journalists/

106. Recently Project Zero has been in news is introduced by


(a) Amazon
(b) Flipkart
(c) Wallmart
(d) Microsoft
Ans: (a)
Explanation:
 Introduced by Amazon.
 To block selling of counterfeit goods on its platform.
 The aim is to ensure that customers receive authentic goods when
shopping on Amazon.
Refer: Facts for Prelims: https://www.insightsonindia.com/2019/11/13/insights-
daily-current-affairs-pib-13-november-2019/

107. Recently Raman 1.0 has been in news sometimes is related to


(a) Upcoming movie on Sir C V Raman
(b) Physics laboratory set up by IISC
(c) New metrics to quantify Scattering of light
(d) None of the above
Ans: (d)
Explanation:
 Raman 1.0: It is hand-held battery operated equipment that performs
rapid detection, in less than a minute of economically driven adulteration
in fats, ghee and edible oils.
Refer: Facts for Prelims: https://www.insightsonindia.com/2019/06/10/insights-
daily-current-affairs-pib-10-june-2019/

108. Recently IAF signed a deal to procure SPICE 2000 guided bombs from
(a) Israel
(b) USA
(c) Russia
(d) South Korea
Ans: (a)
Explanation:
 Indian Air Force (IAF) has signed a deal with Israel’s Rafael Advanced
Defense Systems, an Israeli defence firm to procure a batch of SPICE
2000 guided bombs.

Telegram: https://t.me/insightsIAStips
100
Youtube: https://www.youtube.com/channel/UCpoccbCX9GEIwaiIe4HLjwA
Revision Through MCQs (RTM) Compilation (November 2019)

 The SPICE stands for Smart, Precise Impact and Cost-Effective. It is


guidance and maneuvering kit manufactured by Israel’s Rafael Advanced
Defense Systems.
 SPICE can overcome errors in locating target, GPS jamming and also
reduces collateral damage.
Refer: Facts for Prelims: https://www.insightsonindia.com/2019/06/10/insights-
daily-current-affairs-pib-10-june-2019/

109. What do you understand by the term “Buoyancy in tax”


(a) Efficiency and responsiveness of revenue mobilization in response to growth
in the Gross domestic product or National income.
(b) Effect on a decrease in tax rates on tax collection
(c) Streamlining of various taxes under one Tax such as GST
(d) None of the above
Ans: (a)
Explanation:
 Tax buoyancy is an indicator to measure efficiency and responsiveness of
revenue mobilization in response to growth in the Gross domestic
product or National income.
 A tax is said to be buoyant if the tax revenues increase more than
proportionately in response to a rise in national income or output.

110. In India, which of the following is having highest share in the disbursement of
credit to agriculture and allied activities?
(a) Cooperative Banks
(b) Regional Rural Banks
(c) Commercial Banks
(d) Microfinance institutions
Ans: (c)
Explanation:

Telegram: https://t.me/insightsIAStips
101
Youtube: https://www.youtube.com/channel/UCpoccbCX9GEIwaiIe4HLjwA
Revision Through MCQs (RTM) Compilation (November 2019)

RTM- REVISION THROUGH MCQS - 14th -Nov-2019

111. With reference Right to Information Act in India, consider the following
statements
1. Under the act, Chief Justice of India is a public authority
2. Non-Resident Indians (NRIs) cannot file RTI applications to seek governance-
related information
3. Right to Information is a part of fundamental rights under Article 19(1) of
the Constitution
Which of the given above statements is/are correct?
(a) 1 and 2
(b) 2 and 3
(c) 1 and 3
(d) All of the above
Ans: (c)
Explanation:
 Recently, The Supreme Court has held that office of the Chief Justice of
India is a public authority under the transparency law, the Right to
Information Act.
o The landmark judgment was pronounced by five-judge
constitution bench headed by Chief Justice Ranjan Gogoi.
 Non-Resident Indians (NRIs) can file RTI applications to seek
governance-related information, the government has said, correcting
its earlier stand.
o In response to a query in Lok Sabha, Minister of State for
Personnel, Public Grievances and Pensions Jitendra Singh had on
August 8, 2018 said NRIs were not eligible to file applications
under the Right to Information (RTI) Act.
o The ministry corrected its stand after an activist, after RTI activist
wrote to it pointing out that the transparency act allows every
Indian to seek information.
 RTI is a fundamental Right, which derives its practical applicability and
further real time credibility through the RTI Act.
 Right to Information is a part of fundamental rights under Article
19(1) of the Constitution.
Refer: https://www.insightsonindia.com/2019/11/14/chief-justice-of-indias-
office-under-rti-act/

112. With reference to Green climate fund, consider the following statements
1. The fund was set up under world Bank
2. GCF was formally established during the COP-16 in Cancun
3. Fund’s investments can be in the form of grants, loans, equity or guarantees
Which of the given above statements is/are correct?
(a) 1 and 3
(b) 2 only
(c) 2 and 3
(d) 1, 2 and 3
Ans: (c)
Explanation:
 GCF was set up in 2010 under the UNFCCC’s financial mechanism to
channel funding from developed countries to developing countries to

Telegram: https://t.me/insightsIAStips
102
Youtube: https://www.youtube.com/channel/UCpoccbCX9GEIwaiIe4HLjwA
Revision Through MCQs (RTM) Compilation (November 2019)

allow them to mitigate climate change and also adapt to disruptions


arising from a changing climate.
 It aims to deliver equal amounts of funding to mitigation and adaptation,
while being guided by the Convention’s principles and provisions.
 During COP-16 in Cancun, the matter of governing the GCF was
entrusted to the newly founded Green Climate Fund Board, and the
World Bank was chosen as the temporary trustee.
 The Fund’s investments can be in the form of grants, loans, equity or
guarantees.
Refer: https://www.insightsonindia.com/2019/11/14/green-climate-fund/

113. Vaippar river is a river in the Indian state of


(a) Karnataka
(b) Tamil Nadu
(c) Kerala
(d) Andhra Pradesh
Ans: (b)
Explanation:
 The Kerala State government is saying that it is taking all precautions to
prevent the implementation of the Pamba-Achankovil-Vaippar river link
project.
 Kerala is not allowing Pamba-Achankovil to be connected with the
Vaippar river in Tamil Nadu under the inter-linking of rivers project.
Kerala asserts that there is no excess water in rivers in the state.
 Vaippar is a river in the Indian state of Tamil Nadu. It originates from
the Varushanadu hills bordering the state of Kerala, runs from Sivagiri
and flows through Virudhunagar and Tuticorin districts before entering
the Gulf of Mannar.
Refer: https://www.insightsonindia.com/2019/11/14/pamba-achankovil-vaippar-
river-link-project/

114. With reference to BHIM, consider the following statements


1. It was developed by RBI
2. BHIM named after B R Ambedkar
3. BHIM allow users to send or receive money to non-UPI based accounts
Which of the given above statements is/are correct?
(a) 1 and 3
(b) 2 only
(c) 2 and 3
(d) 1, 2 and 3
Ans: (c)
Explanation:
 Bharat Interface for Money (BHIM) is a UPI based payment interface.
o Developed by National Payments Corporation of India (NPCI).
o Allows real time fund transfer.
o Named after B. R. Ambedkar and Launched in December, 2016.
 BHIM allow users to send or receive money to or from UPI payment
addresses, or to non-UPI based accounts (by scanning a QR code with
account number and IFSC code or MMID (Mobile Money Identifier) Code).
Refer: https://www.insightsonindia.com/2019/11/14/bhim-upi-goes-
international/

Telegram: https://t.me/insightsIAStips
103
Youtube: https://www.youtube.com/channel/UCpoccbCX9GEIwaiIe4HLjwA
Revision Through MCQs (RTM) Compilation (November 2019)

115. Principle of Indicative Planning was adopted for the first- time in the
(a) 6th five year plan
(b) 7th five year plan
(c) 8th five year plan
(d) 9th five year plan
Ans: (c)
Explanation:
 Indicative planning was adopted since 8 five year plan (1992-97).
 Indicative Planning – Here the planning authority sets the broad goals
for the economy. The implementation of the plan requires the co-
ordination of the various units in the economy
 It was characterized by an economy where the private sector is given a
substantial role. State would turn its role into a facilitator from that of a
controller and regulator.
 India’s Eighth Plan was unique in the sense that it attempted to manage
the transition from a centrally planned economy to a market-oriented
economy without tearing the socio-cultural framework of the country, or
to be more specific, our social commitments to the under- previliged
sections. The Eighth Plan mentioned that planning would have to be
reoriented so as to make it indicative.

116. Which of the following is the main objectives of the 12th Five Year Plan?
(a) Inclusive growth and poverty reduction
(b) Inclusive and sustainable growth
(c) Faster, sustainable and more inclusive growth
(d) Sustainable and inclusive growth to reduce poverty and unemployment
Ans: (c)
Explanation:
 The aim of the 12th Five Year plan is to achieve “faster, sustainable
and more inclusive growth”.
 Broad Objectives of 12th Five Year Plan
o To reduce poverty
o To improve regional equality across states and within states
o To improve living conditions for SCs, STs, OBCs, Minorities
o To generate attractive employment opportunities for Indian youth
o To eliminate gender gaps

117. With reference to Disputes raised by India in WTO forum, consider the
following statements
1. Certain measures on imports of iron and steel
2. Countervailing duty by United States on Indian steel products
3. United States Sub-Federal Renewable energy programme
Which of the given above statements is /are correct?
(a) 1 and 2 only
(b) 1 and 3 only
(c) 2 and 3 only
(d) 1, 2 and 3
Ans: (d)

Telegram: https://t.me/insightsIAStips
104
Youtube: https://www.youtube.com/channel/UCpoccbCX9GEIwaiIe4HLjwA
Revision Through MCQs (RTM) Compilation (November 2019)

Explanation: India has 7 disputes at WTO which are at different stages of


settlement. India is defending its interest in these disputes with the help of
experienced Law Firms.
 DS430 - Import of poultry and poultry products from United States,
Complainant: India
 DS436 - Countervailing duty by United States on Indian steel
products, Complainant: India
 DS456 - National Solar Mission dispute with United States, Complainant:
United States
 DS510- United States Sub-Federal Renewable energy programme,
Complainant: India
 DS518 - India-certain Measures on imports of iron and steel products
from Japan, Complainant: Japan
 DS541- Export Subsidies measures of India, Complainant: United States,
 DS-547 - United States-Certain measures of Unites States on steel
and aluminium products, Complainant: India.
Refer: https://pib.gov.in/PressReleaseIframePage.aspx?PRID=1558952

118. With reference to UNNATI Initiative, consider the following statements


1. It is an initiative of ISRO
2. It is open only for developing countries
3. It provides opportunities to strengthen in assembling, integrating and testing
of Nanosatellite
Which of the given above statements is/are correct?
(a) 1 and 2 only
(b) 1 and 3 only
(c) 2 and 3 only
(d) 1, 2 and 3
Ans: (d)
Explanation: UNispace Nanosatellite Assembly & Training by ISRO (UNNATI)
 During the 54th session of Scientific & Technical Subcommittee of
COPUOS at Vienna in February 2017, UNOOSA requested Member
States to propose new initiatives for the benefit of member countries.
 In June 2018, the international community gathered in Vienna to
celebrate the 50th anniversary of the first United Nations Conference on
the Exploration and Peaceful uses of Outer Space (UNISPACE+50).
 During this meet, India announced a capacity building training
programme UNNATI (UNispace Nanosatellite Assembly &Training by
ISRO) on Nanosatellites development through a combination of
theoretical coursework and hands-on training on Assembly, Integration
and Testing (AIT).
 It is an ISRO initiative to commemorate the 50th anniversary of the
first United Nations Conference on the Exploration and Peaceful Uses of
Outer Space (UNISPACE+50).
 The Programme provides opportunities to the participants from
developing countries to strengthen their capabilities in assembling,
integrating and testing of nanosatellites.

Telegram: https://t.me/insightsIAStips
105
Youtube: https://www.youtube.com/channel/UCpoccbCX9GEIwaiIe4HLjwA
Revision Through MCQs (RTM) Compilation (November 2019)

 UNNATI is a capacity building programme on nanosatellite development.


Refer: https://www.insightsonindia.com/2019/01/18/unnati-programme/

119. With reference to Assam Accord of 1985, consider the following statements
1. The accord provided for expulsion of all foreigners, who entered Assam after
1965
2. Clause 6 of the Accord provides for the protection and promotion of
Assamese culture
Which of the given above statements is/are correct?
(a) 1 only
(b) 2 only
(c) Both 1 and 2
(d) Neither 1 nor 2
Ans: (b)
Explanation: Key provisions of the Accord:
 The Assam Accord (1985) was a Memorandum of Settlement (MoS) signed
between representatives of the Government of India and the leaders of
the Assam Movement in New Delhi on 15 August 1985.
 The accord brought an end to the Assam Agitation and paved the way for
the leaders of the agitation to form a political party and form a
government in the state of Assam soon after.
 As per the Accord, those Bangladeshis who came between 1966 and 1971
will be barred from voting for ten years. The Accord also mentions that the
international borders will be sealed and all persons who crossed over
from Bangladesh after 1971 are to be deported.
 What does Clause 6 state?
o Clause 6 of the Assam Accord “envisaged that appropriate
constitutional, legislative and administrative safeguards shall be
provided to protect, preserve and promote the cultural, social,
linguistic identity and heritage of the Assamese people.”
Refer: https://www.insightsonindia.com/2019/01/03/assam-accord/

120. Which of the following are in line with the larger ideology of ‘Pacifism’?
(a) Opposition to war or violence as a means of settling disputes
(b) Sticking to principle rather than pragmatism
(c) Aversion to modern nation-states military institutions and their intrusion in
other nations
(d) Both (a) and (c)
Ans: (d)
Explanation:
 Pacifism preaches opposition to war or violence as a means of
settling disputes.
 It covers a spectrum of views ranging from a preference for diplomacy in
resolving international disputes to absolute opposition to the use of
violence, or even force, in any circumstance. Pacifism may be based on
principle or pragmatism.
 Principled pacifism springs from the belief that war, deliberate lethal
force, violence or any form of coercion is morally wrong. Pragmatic
pacifism does not adhere to such an absolute principle but holds that

Telegram: https://t.me/insightsIAStips
106
Youtube: https://www.youtube.com/channel/UCpoccbCX9GEIwaiIe4HLjwA
Revision Through MCQs (RTM) Compilation (November 2019)

there are better ways of resolving a dispute than war, or considers the
benefits of a war to be outweighed by the costs.
 ‘Dove’ or ‘dovish’ are informal terms used to describe people who seek to
avoid war. The terms allude to the placid nature of the dove. Some people
termed dovish would not view their position as pacifist, for they would
consider war to be justifiable in some circumstances. The opposite of a
dove is a ‘hawk’ or a warmonger.
 Some pacifists, while opposing war, are not opposed to all use of coercion,
physical force against people or destruction of property. Anti-militarists,
for example, are specifically opposed to the modern nation-states’
military institutions rather than to ‘violence’ in general. Other pacifists
follow principles of non-violence, believing that only non-violent action is
acceptable

RTM- REVISION THROUGH MCQS - 15th -Nov-2019

121. With reference to Tribunal’s in India, Consider the following statements


1. Article 323-A of the constitution provides for the establishment of
administrative tribunals by a parliament law
2. Administrative tribunals are constitutional body
3. Constitutional Status to the Tribunals were given by the 42nd (amendment)
act, 1976
Which of the given above statements is/are correct?
(a) 1 and 2
(b) 1 and 3
(c) 1 only
(d) 2 and 3
Ans: (b)
Explanation:
 Administrative Tribunals Act owes its origin to Article 323-A of the
Constitution which empowers Central Government to set-up by an Act of
Parliament, Administrative Tribunals for adjudication of disputes and
complaints with respect to recruitment and conditions of service of
persons appointed to the public service and posts in connection with the
affairs of the Union and the States.
 In pursuance of the provisions contained in the Administrative
Tribunals Act, 1985, the Administrative Tribunals set-up under it
exercise original jurisdiction in respect of service matters of employees
covered by it, therefore it is a Statutory body; and not a constitutional
one
 Constitutional Status to the Tribunals were given by the 42nd
(Amendment) Act, 1976 which inserted Articles 323A and 323B by
which Parliament has been authorized to constitute administrative and
other Tribunals to decide and adjudicate the matters specific therein
Refer: https://www.insightsonindia.com/2019/11/15/sc-strikes-down-finance-
act-rules-for-appointments-to-judicial-tribunals/

Telegram: https://t.me/insightsIAStips
107
Youtube: https://www.youtube.com/channel/UCpoccbCX9GEIwaiIe4HLjwA
Revision Through MCQs (RTM) Compilation (November 2019)

122. Money Bill is defined by article _ of the Indian Constitution


(a) Article 108
(b) Article 110
(c) Article 111
(d) Article 112
Ans: (b)
Explanation:
 A money bill is defined by Article 110 of the Constitution, as a draft
law that contains only provisions that deal with all or any of the matters
listed therein.
 These comprise a set of seven features, broadly including items such as
the imposition or regulation of a tax; the regulation of the borrowing of
money by the Government of India; the withdrawal of money from the
Consolidated Fund of India; and so forth.
 Article 110 further clarifies that in cases where a dispute arises over
whether a bill is a money bill or not, the Lok Sabha Speaker’s decision on
the issue shall be considered final.
Refer: https://www.insightsonindia.com/2019/11/15/sc-strikes-down-finance-
act-rules-for-appointments-to-judicial-tribunals/

123. Recently ‘Nadu-Nedu’ programme has been in news sometimes is launched by


(a) Telangana
(b) Andra Pradesh
(c) Karnataka
(d) Tamil Nadu
Ans: (b)
Explanation:
 Nadu-Nedu’ programme has been launched in Andhra Pradesh.
 The programme seeks to transform government schools into vibrant and
competitive institutions.
Refer: https://www.insightsonindia.com/2019/11/15/nadu-nedu-programme/

124. The government cannot impose restrictions on the practice of freedom of


religion on which of the following grounds?
(a) Public order
(b) Public health
(c) Morality
(d) National Integrity
Ans: (d)
Explanation:
 In India, everyone is free to choose a religion and practice that religion.
Freedom of religion also includes the freedom of conscience. This means
that a person may choose any religion or may choose not to follow any
religion.
 Freedom of religion includes the freedom to profess, follow and propagate
any religion. Freedom of religion is subject to certain limitations.
 The government can impose restrictions on the practice of freedom
of religion in order to protect public order, morality and health.
This means that the freedom of religion is not an unlimited right.
 The government can interfere in religious matters for rooting out certain
social evils. For example in the past, the government has taken steps

Telegram: https://t.me/insightsIAStips
108
Youtube: https://www.youtube.com/channel/UCpoccbCX9GEIwaiIe4HLjwA
Revision Through MCQs (RTM) Compilation (November 2019)

banning practices like sati, bigamy or human sacrifice. Such restrictions


cannot be opposed in the name of interference in right to freedom of
religion
Refer: https://www.insightsonindia.com/2019/11/15/sabarimala-case/

125. Consider the following statements about Surang Bawadi


1. It is an important Karez system was built by Bahmani kings
2. Suranga Bawadi used for rapid mobilization of soldiers
Which of the given above statements is /are correct?
(a) 1 only
(b) 2 only
(c) Both 1 and 2
(d) Neither 1 nor 2
Ans: (d)
Explanation:
 Suranga Bawadi, an integral part of the ancient Karez system of
supplying water through subterranean tunnels built during Adil Shahi
era in Vijayapura, is now set to get funding for restoration.
 A New York-based non-governmental organisation has included it in the
World Monument Watch list for 2020 along with 24 other monuments
from across the world.
 The karez technology basically taps into the ground water sources (or
natural springs) and transports it through an underground tunnel to the
settlement, ending in surface canal and/or pools in the village for various
uses like drinking, washing, ablution, watering livestock, and also further
used for irrigating fields, orchards and gardens.
Refer: https://www.insightsonindia.com/2019/11/15/unesco-world-heritage-
week/

126. With reference to Coalition for Disaster Resilient Infrastructure, consider the
following statements
1. It is an international coalition of countries, private sector and academic
institutions
2. The headquarters of CDRI located in New Delhi
3. It was launched by the Indian Prime Minister at the 2019 UN Climate Action
Summit
Which of the given above statements is/are correct?
(a) 1 and 3
(b) 3 only
(c) 1 and 2
(d) 1, 2 and 3
Ans: (d)
Explanation:
 The Coalition for Disaster Resilient Infrastructure (CDRI) is an
international coalition of countries, United Nations (UN) agencies,
multilateral development banks, the private sector, and academic
institutions that aims to promote disaster-resilient infrastructure.
 Its objective is to promote research and knowledge sharing in the fields of
infrastructure risk management, standards, financing, and recovery
mechanisms.

Telegram: https://t.me/insightsIAStips
109
Youtube: https://www.youtube.com/channel/UCpoccbCX9GEIwaiIe4HLjwA
Revision Through MCQs (RTM) Compilation (November 2019)

 It was launched by the Indian Prime Minister Narendra Modi at the 2019
UN Climate Action Summit
 Headquarters New Delhi, India (Interim Secretariat)
Refer: https://www.insightsonindia.com/2019/11/15/coalition-for-disaster-
resilient-infrastructure-cdri/

127. With reference to the Outer Space Treaty, its obligations and provisions,
consider the following statements.
1. It prohibits the launching of ballistic missiles, which could be armed with
WMD warheads through space
2. China is not a party to the treaty
3. The treaty explicitly forbids any government to claim a celestial resource
such as the Moon or a planet
4. It allows state parties to hold bilateral or multilateral military exercises in
space or on celestial bodies
5. As per the treaty, if a State launches a space object, and retains jurisdiction
and control over that object, it is also liable for damages caused by their
space object
Which of the given above statements is/are correct?
(a) 3 and 5 only
(b) 1, 2, 3, 4 and 5
(c) 2, 3 and 5 only
(d) 1, 4 and 5 only
Ans: (a)
Explanation:
 Space exploration is governed by a complex series of international
treaties and agreements which have been in place for years. The first and
probably most important of them is the Outer Space Treaty.
 The treaty was initially signed by the United States of America, the
United Kingdom and the Soviet Union on January 27, 1967 and it came
into effect from October 10, 1967.
 The treaty was initially called ‘Treaty on Principles Governing the
Activities of States in the Exploration and Use of Outer Space, including
the Moon and other Celestial bodies.
 The treaty forbids countries from deploying “nuclear weapons or any
other kinds of weapons of mass destruction” in outer space. The term
“weapons of mass destruction” is not defined, but it is commonly
understood to include nuclear, chemical, and biological weapons. The
treaty, however, does not prohibit the launching of ballistic missiles,
which could be armed with WMD warheads, through space.
 China is a member.
 The treaty’s key arms control provisions are in Article IV. States-parties
commit not to:
o Place in orbit around the Earth or other celestial bodies any
nuclear weapons or objects carrying WMD.
o Install WMD on celestial bodies or station WMD in outer space in
any other manner.
o Establish military bases or installations, test “any type of
weapons,” or conduct military exercises on the moon and other
celestial bodies.

Telegram: https://t.me/insightsIAStips
110
Youtube: https://www.youtube.com/channel/UCpoccbCX9GEIwaiIe4HLjwA
Revision Through MCQs (RTM) Compilation (November 2019)

o The treaty explicitly forbids any government to claim a celestial


resource such as the Moon or a planet.
 Article II of the Treaty states that "outer space, including the Moon and
other celestial bodies, is not subject to national appropriation by claim of
sovereignty, by means of use or occupation, or by any other means".

Refer: Cooperation in the peaceful uses of outer space:


https://www.insightsonindia.com/2018/06/07/insights-daily-current-affairs-07-
june-2018/

128. It is a compulsory termination of Indian citizenship by the Central


government, if
1. The citizen has shown disloyalty to the Constitution of India
2. The citizen has been ordinarily resident out of India for four years
continuously
Which of the given above statement is/are correct?
(a) 1 only
(b) 2 only
(c) Both 1 and 2
(d) Neither 1 Nor 2
Ans: (a)
Explanation: Some other cases are:
 the citizen has obtained the citizenship by fraud:
 the citizen has shown disloyalty to the Constitution of India (but not to
the laws, because most law breakers are only imprisoned or punished):
 the citizen has unlawfully traded or communicated with the enemy
during a war;
 The citizen has, within five years after registration or naturalisation,
been imprisoned in any country for two years; and the citizen has been
ordinarily resident out of India for seven years continuously.
Refer: https://www.insightsonindia.com/2019/09/28/citizenship-amendment-
bill/

129. The major purpose of the IndIGO consortium, with support from the USA, is to
1. Set up the LIGO-India detector, which would help enhance the network of
gravitational wave detectors worldwide

Telegram: https://t.me/insightsIAStips
111
Youtube: https://www.youtube.com/channel/UCpoccbCX9GEIwaiIe4HLjwA
Revision Through MCQs (RTM) Compilation (November 2019)

2. Make findings about cosmic radiation widely available so that it strengthens


research synergy in theoretical physics
Which of the given above statements is/are correct?
(a) 1 only
(b) 2 only
(c) Both 1 and 2
(d) Neither 1 Nor 2
Ans: (a)
Explanation:
 INDIGO, or IndIGO (Indian Initiative in Gravitational-wave
Observations) is a consortium of Indian gravitational-wave physicists.
 This is an initiative to set up advanced experimental facilities for a multi-
institutional observatory project in gravitational-wave astronomy.
 Since 2009, the IndIGO Consortium has been planning a roadmap for
gravitational-wave astronomy and a phased strategy towards Indian
participation in realizing a gravitational-wave observatory in the Asia-
Pacific region.
 IndIGO is the Indian partner (along with the LIGO Laboratory in U.S.) in
planning the LIGO-India project. The network includes the two LIGO
detectors in the US (in Hanford and Livingston), the Virgo and GEO600
detectors in Europe, and the proposed KAGRA detector in Japan. By
simultaneous detection of the same event on these multiple detectors, a
precise location in the sky can be pinpointed for the source of the
detected waves.
 For example, the first detected gravitational waves by LIGO could only
pinpoint the location of the black hole merger source to a broad area of
the southern hemisphere sky. Using triangulation, this location
information could be improved if the signal was detected on more than
two detectors.
 Another important goal of IndIGO is to train scientists for successfully
operating the LIGO-India detector, when commissioned. Previous studies
have shown that a detector operational in India would improve source
localization significantly, by an order of magnitude or more, depending on
the region of the sky.
Refer: http://www.mea.gov.in/Portal/ForeignRelation/India_US_brief.pdf

130. The Constitution confers which of the following rights and privileges on the
citizens of India but denies the same to aliens?
1. Right to education
2. Right to freedom of speech and expression
3. Right against exploitation
Select the correct answer using the codes below.
(a) 2 and 3 only
(b) 1 and 3 only
(c) 2 only
(d) 1, 2 and 3
Ans: (c)
Explanation: These rights are not available to the foreigners (friendly
aliens):
 Right against discrimination on grounds of religion, race, caste, sex or
place of birth (Article 15).

Telegram: https://t.me/insightsIAStips
112
Youtube: https://www.youtube.com/channel/UCpoccbCX9GEIwaiIe4HLjwA
Revision Through MCQs (RTM) Compilation (November 2019)

 Right to equality of opportunity in the matter of public employment


(Article 16).
 Right to freedom of speech and expression, assembly, association,
movement, residence and profession (Article 19).
 Cultural and educational rights (Articles 29 and 30).
 Right to vote in elections to the Lok Sabha and state legislative
assembly.
 Right to contest for the membership of the Parliament and the state
legislature.
 Eligibility to hold certain public offices, that is, President of India,
Vice-President of India, judges of the Supreme Court and the high courts,
governor of states, attorney general of India and advocate general of
states.
Refer: https://www.insightsonindia.com/2016/12/07/air-spotlight-summary-
fundamental-rights-duties/

RTM- REVISION THROUGH MCQS - 16th -Nov-2019

131. With reference to Indian Judiciary, consider the following statements


1. Articles 125 and 219 give the power of contempt of court to the higher
judiciary
2. Contempt of Courts Act, 1971, clearly states that fair criticism of any case
which has been heard and decided is not contempt
Which of the given above statements is/are correct?
(a) 1 Only
(b) 2 Only
(c) Both 1 and 2
(d) Neither 1 Nor 2
Ans: (b)
Explanation:
 Articles 129 and 215 give the power of contempt of court to the higher
judiciary, and this power limits the freedom granted by Article 19(1)(a).
 Contempt of Courts Act, 1971, very clearly states that fair criticism of
any case which has been heard and decided is not contempt.
 In Arundhati Roy case, the Supreme Court observed that a fair criticism
of the conduct of a Judge, the institution of the judiciary and its
functioning may not amount to contempt if made in good faith and in
public interest.
Refer: https://www.insightsonindia.com/2019/11/16/contempt-of-court-2/

132. Which of the following acts as Insolvency regulator in India?


(a) NCLT
(b) IBBI
(c) RBI
(d) MoF
Ans: (b)
Explanation:
 Insolvency regulator: The Insolvency and Bankruptcy Code, 2016
establishes the Insolvency and Bankruptcy Board of India, to oversee

Telegram: https://t.me/insightsIAStips
113
Youtube: https://www.youtube.com/channel/UCpoccbCX9GEIwaiIe4HLjwA
Revision Through MCQs (RTM) Compilation (November 2019)

the insolvency proceedings in the country and regulate the entities


registered under it.
 The Board will have 10 members, including representatives from the
Ministries of Finance and Law, and the Reserve Bank of India.
Refer: https://www.insightsonindia.com/2019/11/16/rules-notified-to-bring-
financial-firms-under-ibc/
IBC Law and Features: https://www.insightsonindia.com/2019/08/05/rstv-the-
big-picture-insolvency-and-bankruptcy-code-bill/

133. Which of the following is responsible for refinance support and inspection of
Co-operative Banks in India?
(a) RBI
(b) NABARD
(c) State Government
(d) Union Government
Ans: (b)
Explanation:
 NABARD is responsible for regulating and supervising the functions of
Co-operative banks and RRBs.
 In this direction, Institutional Development Department of NABARD has
been taking several initiatives in association with Government of India
and RBI to improve the health of Co-operative banks and Regional Rural
Banks.
 NABARD support the following institutions:
o Regional Rural Banks (RRBs)
o State Cooperative Banks (StCBs)
o District Central Cooperative Banks (DCCBs)
o Primary Agricultural Credit Societies (PACS)
o State Cooperative Agriculture and Rural Development Banks
(SCARDBs)
o Primary Cooperative Agriculture and Rural Development Banks
(PCARDBs)
Refer: https://www.insightsonindia.com/2019/11/16/regulation-on-cooperative-
banks-soon/

134. Mukhyamantri Nishulk Dava Yojana has been in news for sometimes is
launched by
(a) Rajasthan
(b) Odisha
(c) Uttara Pradesh
(d) Delhi
Ans: (a)
Explanation: Rajasthan Mukhyamantri Nishulk Dava Yojana was launched
on 2nd October 2011.
 It has 2 components:
o Free Medicines: To provide commonly-used essential medicines
free of cost to patients visiting government healthcare institutions
(introduced on 2nd October 2011), and
o Free Tests: To provide free tests (introduced on 7th April 2013).

Telegram: https://t.me/insightsIAStips
114
Youtube: https://www.youtube.com/channel/UCpoccbCX9GEIwaiIe4HLjwA
Revision Through MCQs (RTM) Compilation (November 2019)

 For the successful implementation of the same, Rajasthan Medical


Services Corporation Limited (RMSCL) was incorporated as a Public
Limited Company.
 Since 2011, it has benefitted around 67 crore patients, and a record
number of 712 medicines are covered under it.
Refer: https://www.insightsonindia.com/2019/11/16/rajasthans-free-medicine-
scheme-ranked-first/

135. Consider the following statements


1. NISHTHA is the world’s largest teachers’ training programme
2. As per the Right to Education (RTE) Act, to be eligible for appointment as a
teacher in a State, a person would be required to possess the minimum
qualification laid down by the concerned State Council of Teacher
Education.
3. As per the RTE Act, for teaching primary classes, a candidate is required to
pass a Teacher Eligibility Test conducted in accordance with the National
Council of Teacher Education guidelines.
4. In India, more than 90% of teacher education institutions are directly under
the State Governments.
Which of the statements given above is/are correct?
(a) 2, 3 and 4
(b) 1 and 3
(c) 1, 2 and 4
(d) 1, 3 and 4
Ans: (b)
Explanation:
 Statement 1 is correct: NISHTHA is the world’s largest teachers’
training programme of its kind in the world. It has been launched to
improve Learning Outcomes at the Elementary level. The basic objective
of this massive training programme is to motivate and equip teachers to
encourage and foster critical thinking in students. The initiative is first of
its kind wherein standardized training modules are developed at national
level for all States and UTs.
 Statement 2 is not correct: In accordance with the provisions of sub-
section (1) of section 23 of the Right of Children to Free and Compulsory
Education (RTE) Act, 2009, the National Council for Teacher Education
(NCTE) has laid down the minimum qualifications for a person to be
eligible for appointment as a teacher in class I to VIII, vide its Notification
dated August 23, 2010.
 Statement 3 is correct: One of the essential qualifications for a person
to be eligible for appointment as a teacher in any of the primary schools
is that he/she should pass the Teacher Eligibility Test (TET) which will be
conducted by the appropriate Government.
 Statement 4 is not correct: From the mid-1960s to 1993, the number
of TEIs in India went up from about 1,200 to about 1,500. After the
NCTE was set up, the number of TEIs exploded, to about 16,000 (over
90% private) by 2011
Refer: https://www.insightsonindia.com/2019/11/16/national-mission-nishtha-
launched-in-jk/

Telegram: https://t.me/insightsIAStips
115
Youtube: https://www.youtube.com/channel/UCpoccbCX9GEIwaiIe4HLjwA
Revision Through MCQs (RTM) Compilation (November 2019)

136. Recently which of the following has set up an Online Child Sexual Abuse and
Exploitation (OCSAE) Prevention/Investigation Unit?
(a) CBI
(b) MHA
(c) MWCD
(d) Delhi Police
Ans: (a)
Explanation:
 The Central Bureau of Investigation (CBI) has set up an Online Child
Sexual Abuse and Exploitation (OCSAE) Prevention/Investigation
Unit.
 The unit, which will function under the agency’s Special Crime Zone in
Delhi, will collect and disseminate information on online child sexual
abuse and exploitation.
 It will also probe such offences covered under the IPC, the Protection of
Children from Sexual Offences Act and the Information Technology Act,
apart from other relevant laws.
Refer: Facts for Prelims: https://www.insightsonindia.com/2019/11/16/insights-
daily-current-affairs-pib-16-november-2019/

137. Consider the following statements about BRICS forum.


1. The Chairmanship of the forum is rotated annually among the members, in
accordance with the country that is hosting the conference.
2. BRICS forum is well known to engage all “tracks” of interaction such as
Track I, Track II and Track III.
Which of the given above statements is/are correct?
(a) 1 only
(b) 2 only
(c) Both 1 and 2
(d) Neither 1 Nor 2
Ans: (b)
Explanation:
 The Chairmanship of the forum is rotated annually among the
members, in accordance with the acronym B-R-I-C-S.
 Over and above the summit, BRICS cooperation in the past decade has
expanded to include an annual programme of over 100 sectoral meetings.
Cooperation among members is predicated on
 three levels or “tracks” of interaction, namely:
o Track I: Formal diplomatic engagement between the national
governments.
o Track II: Engagement through government-affiliated institutions,
e.g. state-owned enterprises and business councils.
o Track III: Civil society and “people-to-people” engagement.
 The First BRIC Summit was held in June 2009 in Yekaterinburg, Russia.
 The Ninth BRICS Summit was held in Xiamen, China under the theme
“BRICS: Stronger Partnership for a Brighter Future”. The leaders built
on achievements already made with a share vision for future development
of BRICS, discussed international and regional issues of common
concern and adopted the Xiamen Declaration.

Telegram: https://t.me/insightsIAStips
116
Youtube: https://www.youtube.com/channel/UCpoccbCX9GEIwaiIe4HLjwA
Revision Through MCQs (RTM) Compilation (November 2019)

 The 10th BRICS summit, is being held in South Africa under the theme:
“BRICS in Africa: Collaboration for Inclusive Growth and Shared
Prosperity in the 4th Industrial Revolution”.
 The 11th BRICS Summit took place from 13th-14th November 2019 in
Brasilia, Brazil. The theme of the 2019 BRICS summit is "Economic
Growth for an Innovative Future.”
o BRICS countries adopted the Brasilia Declaration
Refer: https://www.insightsonindia.com/2019/11/16/insights-into-editorial-india-
worlds-most-investment-friendly-economy-says-pm-modi/

138. Consider the following statements.


1. UNISPACE+50 is an initiative of the G-77 to coordinate effectively in the
domain of satellite supervision and disaster management.
2. The Space 2030 agenda of the U.N. will map out how spaceflight and space-
related activities can help achieve the Sustainable Development Goals
(SDGs).
Which of the given above statements is/are correct?
(a) 1 only
(b) 2 only
(c) Both 1 and 2
(d) Neither 1 Nor 2
Ans: (b)
Explanation:
 UNISPACE+50 will celebrate the fiftieth anniversary of the first United
Nations Conference on the Exploration and Peaceful Uses of Outer
Space.
 It will also be an opportunity for the international community to gather
and consider the future course of global space cooperation for the benefit
of humankind.
 The shared goal for UNISPACE+50 is to build, together with all
stakeholders, a comprehensive Space2030 agenda for the contribution of
space activities to the achievement of the Sustainable Development
Goals, addressing overarching, long-term development concerns, and
which is based on the peaceful exploration and uses of outer space.
 It was a chance to assess results following the three prior UNISPACE
conferences, held in 1968, 1982 and 1999, and consider how the future
course of global space cooperation can benefit everyone on Earth.
 The event aimed at shaping the UN’s comprehensive ‘Space2030’ agenda.
The Space2030 agenda will map out how spaceflight and space-related
activities can help achieve the 17 Sustainable Development Goals (SDGs),
addressing overarching, long-term development concerns, through the
peaceful exploration and uses of outer space.
Refer: https://www.unoosa.org/oosa/en/ourwork/unispaceplus50/index.html

139. The kingdom of Khurda is mainly associated to which state


(a) Maharashtra
(b) Bihar
(c) Odisha
(d) West Bengal
Ans: (c)
Explanation:

Telegram: https://t.me/insightsIAStips
117
Youtube: https://www.youtube.com/channel/UCpoccbCX9GEIwaiIe4HLjwA
Revision Through MCQs (RTM) Compilation (November 2019)

 Two-hundred years ago in 1817, a valiant uprising of soldiers led by Buxi


Jagabandhu (Bidyadhar Mohapatra) took place in Khurda of Odisha. This
is known as Paika rebellion.

140. Ease of Living Index is released by


(a) Niti Ayog
(b) World Bank
(c) Ministry of Rural Development
(d) Ministry of Housing and Urban Affairs
Ans : (d)
Explanation:
 Ease of Living Index is A Transformative Initiative of The Ministry of
Housing and Urban Affairs to Help Cities Assess their Liveability Vis-À-
Vis National & Global Benchmarks.
 “The ‘Ease of Living’ Index seeks to assist cities in undertaking a 360-
degree assessment of their strengths, weaknesses, opportunities, and
threats”.

RTM- REVISION THROUGH MCQS - 18th -Nov-2019

141. ‘England’s Debt to India’ and ‘India’s Will to Freedom’ are important works of
(a) Veer Savarkar
(b) Dadabhai Naoroji
(c) Ramesh Dutt
(d) None of the above
Ans: (d)
Explanation: Lala Lajpat Rai’s contributions, achievements and related key
facts
 Rai is remembered for his role during the Swadeshi movement and for his
advocacy of education.
 In 1885, Rai established the Dayanand Anglo-Vedic School in Lahore and
remained a committed educationist throughout his life.
 Rai, Tilak, and Bipin Chandra Pal (called Lal-Bal-Pal) fervently advocated
the use of Swadeshi goods and mass agitation in the aftermath of the
controversial Partition of Bengal in 1905 by Lord Curzon.
 He founded the Indian Home Rule League of America in New York City in
1917.
 His important works include: ‘The Arya Samaj’, ‘Young India’,
‘England’s Debt to India’, ‘Evolution of Japan’, ‘India’s Will to Freedom’,
‘Message of the Bhagwad Gita’, ‘Political Future of India’, ‘Problem of
National Education in India’, ‘The Depressed Glasses’, and the travelogue
‘United States of America’.
Refer: https://www.insightsonindia.com/2019/11/18/lala-lajpat-rai/

142. He wrote biographies of Mazzini, Garibaldi, Shivaji and Shrikrishna; stayed in


America for some time; and was also elected to the Central Assembly. He was
(a) Aurobindo Ghosh
(b) Bipin Chandra Pal
(c) Lala Lajpat Rai
(d) Motilal Nehru

Telegram: https://t.me/insightsIAStips
118
Youtube: https://www.youtube.com/channel/UCpoccbCX9GEIwaiIe4HLjwA
Revision Through MCQs (RTM) Compilation (November 2019)

Ans: (c)
Explanation:
 Lala Lajpat Rai wrote the biographies of Mazzini, Garibaldi, Shivaji,
Dayanand and Shri Krishna, besides other important works in English,
Hindi and Urdu.
Refer: https://www.insightsonindia.com/2019/11/18/lala-lajpat-rai/

143. With reference to Social Awareness and Action to Neutralise Pneumonia


Successfully (SAANS) Initiative, consider the following statements
1. Program is launched by MWCD to control Pneumonia
2. It is a central sector scheme
3. Under the scheme, child suffering from Pneumonia will be treated with pre-
referral dose of Amoxicillin by ASHA
4. The initiative also aims to create mass awareness about the most effective
solutions for pneumonia prevention like breast feeding, immunization, good
quality air
Which of the given above statements is/are correct?
(a) 1, 3 and 4
(b) 3 and 4
(c) 2, 3 and 4
(d) 1, 2, 3 and 4
Ans: (b)
Explanation:
 The Ministry of Health and Family Welfare has launched a campaign
in Gujarat named Social Awareness and Action to Neutralise Pneumonia
Successfully (SAANS) to control Pneumonia.
 This campaign will mobilize people's for protecting and preventing
children from pneumonia; and health personnel, governments & other
stakeholders to prioritize treatment towards the control of Pneumonia —
one of the most fatal childhood illnesses.
 Under the SAANS campaign, new interventions have been included like a
child suffering from Pneumonia will be treated with pre-referral dose of
Amoxicillin by ASHA; Pulse Oximeter will be used at the Health and
Wellness Centre for identification of low oxygen levels in the blood and if
required treated by use of Oxygen.
 The initiative also aims to create mass awareness about the most
effective solutions for pneumonia prevention like breast feeding, age
appropriate complementary feeding, immunization, good quality air etc.
Refer: https://www.insightsonindia.com/2019/11/18/social-awareness-and-
action-to-neutralise-pneumonia-successfully-saans/

144. Consider the following statements:


1. India have bilateral free trade agreement with all members of ASEAN nations
2. India has signed bilateral FTA with Sri Lanka, Bhutan, Nepal and
Bangladesh
Which of the given above statements is/are correct?
(a) 1 only
(b) 2 only
(c) Both 1 and 2
(d) Neither 1 Nor 2
Ans: (d)

Telegram: https://t.me/insightsIAStips
119
Youtube: https://www.youtube.com/channel/UCpoccbCX9GEIwaiIe4HLjwA
Revision Through MCQs (RTM) Compilation (November 2019)

Explanation:
 India has signed bilateral FTAs with Sri Lanka (1998), Afghanistan
(2003), Thailand (2004), Singapore (2005), Bhutan (2006), Nepal (2009),
Korea (2009), Malaysia (2011) and Japan (2011).
 FTA with Bangla Desh is under negotiation
 There have also been two regional trade agreements, the South Asian
Free Trade Agreement (SAFTA, 2004) and the India-Association of
Southeast Asian Nations Agreement (ASEAN, 2010).
Refer: https://www.insightsonindia.com/2019/11/18/indias-free-trade-
agreements-with-asean/

145. ADMM-Plus meeting has been in news for sometimes is related to


(a) ASEAN
(b) SAARC
(c) RCEP
(d) SCO
Ans: (a)
Explanation:
 The ADMM-Plus is a platform for ASEAN and its eight Dialogue
Partners to strengthen security and defence cooperation for peace,
stability, and development in the region.
 Agreed five areas of practical cooperation under this new mechanism,
namely maritime security, counter-terrorism, humanitarian assistance
and disaster relief, peacekeeping operations and military medicine. In
2013, a new priority area of humanitarian mine action was agreed.
Refer: https://www.insightsonindia.com/2019/11/18/admm-plus/

146. With reference to Bureau of Indian standards, consider the following


statements
1. BIS is the national Standards Body of India working under the aegis of
MoC&I
2. BIS head will be appointed by Cabinet secretary
3. BIS is a statutory body
Which of the given above statements is/are not correct?
(a) 1 and 2
(b) 2 only
(c) 1 and 3 only
(d) All of the above
Ans: (a)
Explanation: Here Directive word is NOT CORRECT!!
 Bureau of Indian Standards (BIS) is the national Standards Body of
India working under the aegis of Ministry of Consumer Affairs, Food &
Public Distribution
 It is established by the Bureau of Indian Standards Act, 1986 which
came into effect on 23 December 1986.
 The Minister in charge of the Ministry or Department having
administrative control of the BIS is the ex-officio President of the BIS.
Refer: https://www.insightsonindia.com/2019/11/18/water-quality-report/

Telegram: https://t.me/insightsIAStips
120
Youtube: https://www.youtube.com/channel/UCpoccbCX9GEIwaiIe4HLjwA
Revision Through MCQs (RTM) Compilation (November 2019)

147. Consider the following statements:


1. The Standard Mark of Bureau of Indian Standards (BIS) is mandatory for
automotive tyres and tubes.
2. AGMARK is a quality Certification Mark issues by the Food and Agriculture
Organisation (FAO).
Which of the statements given above is/are correct?
(a) 1only
(b) 2 only
(c) Both 1 and 2
(d) Neither 1 nor 2
Ans: (a)
Explanation:
 In 2010, a certification from the Bureau of Indian Standards (BIS) has
been made mandatory for all types of automotive tyres and tubes.
 AGMARK is a certification mark employed on agricultural products in
India, assuring that they conform to a set of standards approved by the
Directorate of Marketing and Inspection, an agency of the Government
of India.
 The AGMARK is legally enforced in India by the Agricultural Produce
(Grading and Marking) Act of 1937 (and amended in 1986)
Refer: https://www.insightsonindia.com/2019/11/18/water-quality-report/

148. Which of the following Oil Refinery/Refinery’s located in Assam?


1. Bongaigaon Refinery
2. Haldia Refinery
3. Guwahati Refinery
4. Barauni Refinery
Select the correct answer using the code below
(a) 3 only
(b) 1 and 3 only
(c) 2, 3 and 4 only
(d) All of the above
Ans: (b)
Explanation:
 Bongaigaon Refinery & Guwahati Refinery- Assam
 Haldia Refinery- West Bengal
 Barauni Refinery- Bihar
Refer: Facts for Prelims: https://www.insightsonindia.com/2019/11/18/insights-
daily-current-affairs-pib-18-november-2019/

149. Which of the following are the objectives of ‘National Nutrition Mission’?
1. To create awareness relating to malnutrition among pregnant women and
lactating mothers.
2. To reduce the incidence of anaemia among young children, adolescent girls
and women.
3. To promote the consumption of millets, coarse cereals and unpolished rice.
4. To promote the consumption of poultry eggs.
Select the correct answer using the code given below:
(a) 1 and 2 only
(b) 1, 2 and 3 only
(c) 1, 2 and 4 only

Telegram: https://t.me/insightsIAStips
121
Youtube: https://www.youtube.com/channel/UCpoccbCX9GEIwaiIe4HLjwA
Revision Through MCQs (RTM) Compilation (November 2019)

(d) 3 and 4 only


Ans: (a)
Explanation:
 There are two components of the National Nutrition Mission as
follows:
o Information, Education and Communication (IEC) Campaign
against malnutrition
o Multi-sectoral Nutrition Programme
 The key objectives of these programmes are as under:-
o To create awareness relating to malnutrition amongst pregnant
women, lactating mothers, promote healthy lactating practices and
importance of balanced nutrition
o To improve maternal and child under-nutrition in 200 high
burdened districts and to prevent and reduce the under-nutrition
prevalent among children below 3 years
o To reduce incidence of anaemia among young children, adolescent
girls and women.

150. Due to some reasons, if there is a huge fall in the population of species of
butterflies, what could be its likely consequence/consequences?
1. Pollination of some plants could be adversely affected.
2. There could be a drastic increase in the fungal infections of some cultivated
plants.
3. It could lead to a fall in the population of some species of wasps, spiders and
birds.
Select the correct using the code given below:
(a) 1 only
(b) 2 and 3 only
(c) 1 and 3 only
(d) 1, 2 and 3
Ans: (c)
Explanation:
 Statement 1 and 3 are correct. Butterflies are pollinating insects. They
help in pollination of many flowering plants.
 Butterflies also act as a lower member of the food chain.
 A number of animals, including birds and mice feed on butterfly.
 As populations of butterfly diminish, so will populations of birds and
other animals that rely on them as a food source.
 This loss of the butterfly is the beginning of the “butterfly effect”

RTM- REVISION THROUGH MCQS - 19th -Nov-2019

151. The Parliament of India acquires the power to legislate on any item in the
State List in the national interest if a resolution to that effect is passed by the
(a) Lok Sabha by a simple majority of its total membership
(b) Lok Sabha by a majority of not less than two thirds of its total membership
(c) Rajya Sabha by a simple majority of its total membership
(d) Raja Sabha by a majority of not less than two thirds of its members present
and voting

Telegram: https://t.me/insightsIAStips
122
Youtube: https://www.youtube.com/channel/UCpoccbCX9GEIwaiIe4HLjwA
Revision Through MCQs (RTM) Compilation (November 2019)

Ans: (d)
Explanation:
 If Rajya Sabha passes a resolution by a majority of not less than two-
thirds of members present and voting saying that it is “necessary or
expedient in the national interest” that Parliament should make a law on
a matter enumerated in the State List, Parliament becomes empowered to
make a law onthe subject specified in the resolution, for the whole or any
part of the territory of India.
 Such a resolution remains in force for a maximum period of one year but
this period can be extended by one year at a time by passing a similar
resolution further.
Refer: https://www.insightsonindia.com/2019/11/19/equal-representation-to-all-
states-in-rajya-sabha-sought/

152. Consider the following statements :


1. The Rajya Sabha has no power either to reject or to amend a Money Bill
2. The Rajya Sabha cannot vote on the Demands for Grants
3. The Rajya Sabha cannot discuss the Annual Financial Statement
Which of the statements given above is/are correct?
(a) 1 only
(b) 1 and 2 only
(c) 2 and 3 only
(d) 1, 2 and 3
Ans: (b)
Explanation:
 Statement 1 and 2 are right. 3rd statement is incorrect as Budget can be
discussed by Rajyasabha.
Refer: https://www.insightsonindia.com/2019/11/19/equal-representation-to-all-
states-in-rajya-sabha-sought/

153. With reference to Gram Panchayats, consider the following statements


1. Term Gram Sabha is defined in the Constitution of India under Article 243
2. The power to annul a decision of the Gram Sabha rests with the Gram
Sabha only
3. According to the State Panchayat Raj Acts, the Gram Sabha must meet at
least two to four times in a year
Which of the given above statements is/are correct?
(a) 1 and 2 only
(b) 3 only
(c) 2 and 3 only
(d) 1, 2 and 3
Ans: (d)
Explanation:
 What is Gram Sabha
o The term Gram Sabha is defined in the Constitution of India under
Article 243(b).
o Gram Sabha is the primary body of the Panchayati Raj system and
by far the largest.
o It is a permanent body.

Telegram: https://t.me/insightsIAStips
123
Youtube: https://www.youtube.com/channel/UCpoccbCX9GEIwaiIe4HLjwA
Revision Through MCQs (RTM) Compilation (November 2019)

Gram Sabha is the Sabha of the electorate. All other institutions of


o
the Panchayati Raj like the Gram Panchayat, Block Panchayat and
Zilla Parishad are constituted by elected representatives.
o The decisions taken by the Gram Sabha cannot be annulled by
any other body. The power to annul a decision of the Gram Sabha
rests with the Gram Sabha only.
 Who are the members of Gram Sabha
o Persons, those who are above 18 years of age and
o living in the village and
o whose names are included in the electoral rolls for the Panchayat
at the village level
 Scheduled meetings of the Gram Sabha
o According to the State Panchayat Raj Acts, the Gram Sabha must
meet at least two to four times in a year.
o For people's convenience, in most of the States, four national-
international days have been identified as reference dates for these
meetings. They are
 Republic Day (26th January)
 Labour day (1st May)
 Independence Day (15th August)
 Gandhi Jayanti (2nd October)
o Gram Panchayats are however free to convene Gram Sabha on
other dates according to their convenience.
Refer: https://www.insightsonindia.com/2019/11/19/gram-sabha/

154. Pradhan Mantri Matru Vandana Yojana (PMMVY) is a conditional cash transfer
scheme for pregnant and lactating women of 19 years of age or above for the
first live birth. The scheme falls under the ages of
(a) MWCD
(b) MoHFW
(c) MoSJE
(d) Both A and B
Ans: (a)
Explanation:
 Pradhan Mantri Matru Vandana Yojana (PMMVY) is a maternity benefit
rechristened from erstwhile Indira Gandhi Matritva Sahyog Yojana
(IGMSY). The IGMSY was launched in 2010.
 The scheme is a conditional cash transfer scheme for pregnant and
lactating women
 It provides a partial wage compensation to women for wage-loss during
childbirth and childcare and to provide conditions for safe delivery and
good nutrition and feeding practices.
 They receive a cash benefit of Rs. 5,000 in three instalments on fulfilling
the respective conditionality, viz. early registration of pregnancy, ante-
natal check-up and registration of the birth of the child and completion
of first cycle of vaccination for the first living child of the family.
 The eligible beneficiaries also receive cash incentive under Janani
Suraksha Yojana (JSY). Thus, on an average, a woman gets Rs. 6,000.
Refer: https://www.insightsonindia.com/2019/11/19/pradhan-mantri-matru-
vandana-yojana-pmmvy/

Telegram: https://t.me/insightsIAStips
124
Youtube: https://www.youtube.com/channel/UCpoccbCX9GEIwaiIe4HLjwA
Revision Through MCQs (RTM) Compilation (November 2019)

155. Recently Kimberley Process has been in news for sometimes is related to
(a) It is an international certification scheme that regulates trade in rough
diamonds
(b) It is an international certification scheme that regulates trade in Brent
Crude Oil
(c) It is an international certification scheme that regulates trade in 24 carat
Gold
(d) Both A and C
Ans: (a)
Explanation:
 The Kimberley Process is an international certification scheme that
regulates trade in rough diamonds. It aims to prevent the flow of conflict
diamonds, while helping to protect legitimate trade in rough diamonds.
Refer: https://www.insightsonindia.com/2019/11/19/kimberley-process-3/

156. The 2019 World Talent Ranking has been released by


(a) World Economic Forum
(b) World Intellectual Property Organization
(c) Quacquarelli Symonds Limited(QS)
(d) None of the above
Ans: (d)
Explanation: The 2019 World Talent Ranking has been released.
 It is released by the International Institute for Management
Development (IMD). IMD is a business education school based in
Switzerland.
 The ranking is based on the performance in three main categories —
investment and development, appeal and readiness.
Refer: https://www.insightsonindia.com/2019/11/19/imd-world-talent-ranking-
2019/

157. Za’ir-Al-Bahr (the Roar of the Sea)- It is the Joint Exercise between the Indian
Navy and
(a) Qatari Navy
(b) Royal Navy of Oman
(c) Royal Saudi Navy
(d) Yemeni Navy
Ans: (a)
Explanation: Za’ir-Al-Bahr (the Roar of the Sea):
 It is the Joint Exercise between the Qatari Emiri Navy and the Indian
Navy, Za’ir-Al-Bahr (the Roar of the Sea).
 The latest edition is being held in Doha.
 Aim: To strengthen cooperation and enhance interoperability between the
two navies.
Refer: Facts for prelims: https://www.insightsonindia.com/2019/11/19/insights-
daily-current-affairs-pib-19-november-2019/

158. Consider the following statements regarding Pradhan mantri awas yojana
gramin
1. Scheme implemented in rural areas throughout India
2. Beneficiary entitled to 100 days of unskilled labour under MGNREGA
3. Beneficiary can avail a loan of Rs.70000/- for construction of house

Telegram: https://t.me/insightsIAStips
125
Youtube: https://www.youtube.com/channel/UCpoccbCX9GEIwaiIe4HLjwA
Revision Through MCQs (RTM) Compilation (November 2019)

4. Unit size is 25 sq mts


Select the correct answer using the code below:
(a) 1, 2 and 3 only
(b) 2 and 3 only
(c) 3 and 4 only
(d) All of the above
Ans: (c)
Explanation:
 In pursuance to the goal – Housing for all by 2022, the rural housing
scheme Indira Awas Yojana has been revamped to Pradhan Mantri Awaas
Yojana – Gramin and approved during March 2016. Under the scheme,
financial assistance is provided for construction of pucca house to all
houseless and households living in dilapidated houses.
 It is proposed that one crore households would be provided assistance for
construction of pucca house under the project during the period from
2016-17 to 2018-19.
 The scheme would be implemented in rural areas throughout India
except Delhi and Chandigarh. The cost of houses would be shared
between Centre and States.
 Under PMAY, the cost of unit assistance is to be shared between Central
and State Governments in the ratio 60:40 in plain areas and 90:10 for
North Eastern and hilly states.
 The unit assistance given to beneficiaries under the programme is Rs
1,20,000 in plain areas and to Rs 1,30,000 in hilly states/difficult areas
/Integrated Action Plan (IAP) for Selected Tribal and Backward Districts.
Presently the NE States, States of HP, J&K and Uttarakhand and all 82
LWE districts are identified as difficult and hilly areas.
 The unit size is 25 sq.m including a dedicated area for hygienic cooking.
 The beneficiary is entitled to 90 days of unskilled labour from
MGNREGA.
 The beneficiary would be facilitated to avail loan of up to Rs.70,000/- for
construction of the house which is optional.
 Funds will be transferred electronically directly to the account of the
beneficiary.

159. Consider the following with respect to DNA technology bill


1. Bill provides DNA profiling for civil and criminal disputes
2. Written consent of individual is needed to collect DNA samples in all cases
3. Seeks to establish regional DNA data bank one for each state only
Select the correct answer using the code below:
(a) 1 and 2 only
(b) 2 and 3 only
(c) 1 only
(d) All of the above
Ans: (c)
Explanation:
 The Bill regulates the use of DNA technology for establishing the
identity of persons in respect of matters listed in a Schedule. These
include criminal matters (such as offences under the Indian Penal Code,
1860), and civil matters such as parentage disputes, emigration or
immigration, and transplantation of human organs.

Telegram: https://t.me/insightsIAStips
126
Youtube: https://www.youtube.com/channel/UCpoccbCX9GEIwaiIe4HLjwA
Revision Through MCQs (RTM) Compilation (November 2019)

 The Bill establishes a National DNA Data Bank and Regional DNA Data
Banks. Every Data Bank will maintain the following indices: (i) crime
scene index, (ii) suspects’ or undertrials’ index, (iii) offenders’ index, (iv)
missing persons’ index, and (v) unknown deceased persons’ index.
 The Bill establishes a DNA Regulatory Board. Every DNA laboratory that
analyses a DNA sample to establish the identity of an individual, has to
be accredited by the Board.
 Written consent by individuals is required to collect DNA samples
from them. Consent is not required for offences with punishment of
more than seven years of imprisonment or death.
 The Bill provides for the removal of DNA profiles of suspects on filing of a
police report or court order, and of undertrials on the basis of a court
order. Profiles in the crime scene and missing persons’ index will be
removed on a written request.
 The central government will establish a National DNA Data Bank and
Regional DNA Data Banks for each state, or two or more states, as it
may deem necessary.
Refer: https://www.insightsonindia.com/2019/06/27/dna-technology-bill-2/

160. Sepahijala Wildlife Sanctuary is in which state


(a) Assam
(b) Meghalaya
(c) Arunachal Pradesh
(d) Tripura
Ans: (d)
Explanation:
 The Sepahijala wildlife sanctuary carries the coveted ‘Clouded Leopard
National Park’ tag since 2011 thanks to its successful breeding of the
near-extinct animal. This is the only place where the endangered clouded
leopard is bred and conserved.
 Tripura has three other sanctuaries, Trishna in South Tripura, Rowa in
North Tripura and Gomati wildlife sanctuary in Gomati district. Among
these, Trishna was declared as a bison reserve.

RTM- REVISION THROUGH MCQS- 20th -Nov-2019

161. With reference to Indian Freedom Struggle, consider the following statements
1. Rani Laksmi Bhai rapidly organized her troops and assumed charge of the
rebels in the Bundelkhand region
2. Rani Laksmi Bhai died while fighting a squadron of the 8th Hussars under
Gen. Hugh Rose
Which of the given above statement is/are correct?
(a) 1 only
(b) 2 only
(c) Both 1 and 2
(d) Neither 1 Nor 2
Ans: (a)
Explanation:

Telegram: https://t.me/insightsIAStips
127
Youtube: https://www.youtube.com/channel/UCpoccbCX9GEIwaiIe4HLjwA
Revision Through MCQs (RTM) Compilation (November 2019)

 Rani Laksmi Bhai had a son Damodar Rao, who died within four months
of his birth. Following the death of the infant, her husband adopted a
cousin’s child Anand Rao, who was renamed Damodar Rao a day prior to
the death of the Maharaja.
 Lord Dalhousie refused to acknowledge the child and applied the
Doctrine of Lapse, and annexed the state. However, the Rani refused to
accept the Lord Dalhousie’s decision.
 Joining the uprising against the British, she rapidly organized her troops
and assumed charge of the rebels in the Bundelkhand region. Mutineers
in the neighbouring areas headed toward Jhansi to offer her support
 The queen died while fighting a squadron of the 8th Hussars under
Captain Heneage, on June 18, 1858, in Kotah-ki-Serai near the Phool
Bagh of Gwalior.
Refer: https://www.insightsonindia.com/2019/11/20/rani-lakshmibai/

162. Consider the following statements


1. Plague is a disease caused by the virus
2. Plague can be transmitted to humans through sexual contact
3. There is no treatment available to cure Plague
Which of the given above statements is/are not correct?
(a) 3 only
(b) 2 and 3 only
(c) 1 and 3 only
(d) 1, 2 and 3
Ans: (d)
Explanation: Here Directive Word Is Not Correct!!
 The plague is a disease caused by the bacteria Yersinia pestis, which is
found in animals, particularly rodents.
 It can be transmitted to humans through infected animals and fleas.
 The plague is a life-threatening disease but if caught early, can be
treated with antibiotics. However, without prompt treatment, the
disease can lead to serious illnesses and even death.
Refer: https://www.insightsonindia.com/2019/11/20/plague/

163. Consider the following temperature and rainfall data:

These are most likely to be found in the natural regions of:


(a) Humid sub-Temperate Monsoon Climate
(b) Humid sub-Tropical Monsoon Climate

Telegram: https://t.me/insightsIAStips
128
Youtube: https://www.youtube.com/channel/UCpoccbCX9GEIwaiIe4HLjwA
Revision Through MCQs (RTM) Compilation (November 2019)

(c) Dry sub-Tropical Monsoon Climate


(d) Dry sub-Temperate Monsoon Climate
Ans: (b)
Explanation:
 Above Climate data is for Aizawl, the capital of Mizoram
(https://web.archive.org/web/20150701183234/http://www.imd.gov.in
/doc/climateimp.pdf)
 Mizoram has a mild climate, being relatively cool in summer 20 to 29 °C
(68 to 84 °F) but progressively warmer, most probably due to climate
change, with summer temperatures crossing 30 degrees Celsius and
winter temperatures ranging from 7 to 22 °C (45 to 72 °F).
 The region is influenced by monsoons, raining heavily from May to
September with little rain in the dry (cold) season.
 The climate pattern is moist tropical to moist sub-tropical, with
average state rainfall 254 centimetres (100 in) per annum.
 The state is in a region where cyclones and landslides can cause
weather-related emergencies.
Refer: https://www.insightsonindia.com/2019/11/20/mizoram-revokes-forest-
rights-act/

164. Article 371(G) of the Constitution states that the Parliament cannot decide on
the matters of the religious and social practices of the
(a) Mizos Tribe
(b) Naga Tribe
(c) Garo Tribe
(d) None of the above
Ans: (a)
Explanation:
 Article 371(G) of the Constitution states that the Parliament cannot
decide on the matters of the religious and social practices of the Mizos,
civil and criminal law of the land, land ownership transfer, and
customary law procedure without the consent of the Assembly.
Refer: https://www.insightsonindia.com/2019/11/20/mizoram-revokes-forest-
rights-act/

165. “The term entered English in the late 15th century from French, it has been
described as the "crossroads of western Asia, the eastern Mediterranean, and
northeast Africa also region is an approximate historical geographical term
referring to a large area in the Eastern Mediterranean, primarily in Western Asia.
In its narrowest sense, it is equivalent to the historical region of Syria.”
The above passage describes which of the following geographical region?
(a) West bank
(b) Levant
(c) Transcaucasia
(d) Sinai Peninsula
Ans: (b)
Explanation:
 The Levant is a French word. It is the present participle of the French
word for to rise "lever," and its use in geography refers to the direction
that the sun comes up. The geographic term means "the countries of

Telegram: https://t.me/insightsIAStips
129
Youtube: https://www.youtube.com/channel/UCpoccbCX9GEIwaiIe4HLjwA
Revision Through MCQs (RTM) Compilation (November 2019)

the east." The east, in this case, means the eastern Mediterranean
region, meaning the islands and the adjoining countries.
 The Levant region called the Fertile Crescent saw some of the earliest
use of domesticated plants and animals during the Neolithic period; and
some of the earliest urban sites arose here in Mesopotamia, what is today
Iraq. Judaism got its start here, and from it, Christianity developed a few
thousand years later.
 Term is often used in reference to the ancient lands in the Old Testament
of the Bible (Bronze Age): the kingdoms of Israel, Ammon, Moab, Judah,
Edom, and Aram; and the Phoenician and Philistine states. Important
cities include Jerusalem, Jericho, Petra, Beersheba, Rabbath-Ammon,
Ashkelon, Tyre, and Damascus.

Refer: https://www.insightsonindia.com/2019/11/20/west-bank-and-issues-
associated/

166. The term "two-state solution" is sometimes mentioned in the news in the
context of the affairs of
(a) China
(b) Israel
(c) Greece
(d) Yemen
Ans: (b)
Explanation: Two-State Solution of Israel Palestine Issue:
 It envisages an independent State of Palestine alongside the State of
Israel west of Jordan River.
 The UNSC Resolution 1397 agreed in 2000 with support from USA and
becoming first UNSC resolution to agree on two state solution
Refer: https://www.insightsonindia.com/2019/11/20/west-bank-and-issues-
associated/

167. Xinjiang shares borders with


1. Mongolia
2. Russia

Telegram: https://t.me/insightsIAStips
130
Youtube: https://www.youtube.com/channel/UCpoccbCX9GEIwaiIe4HLjwA
Revision Through MCQs (RTM) Compilation (November 2019)

3. Kazakhstan
4. Nepal
5. Afghanistan
6. Pakistan
7. India
Select the correct answer using the below code:
(a) 1, 2, 3, 4 and 6
(b) 1, 2, 3, 5 and 6
(c) 2, 3, 4, 5 and 6
(d) 1, 2, 3, 5, 6 and 7
Ans: (d)
Explanation:
 Xinjiang shares borders with Mongolia, Russia, Kazakhstan, Kyrgyzstan,
Tajikistan, Afghanistan, Pakistan and India.


Refer: https://www.insightsonindia.com/2019/11/20/uighur-detention-camps/

168. Consider the following statements


1. Deposit Insurance and Credit Guarantee Corporation (DICGC) under IRDAI
2. In India, every insured bank pays premium amounting to 10% of its deposits
to DICGC every year
3. When a bank is liquidated, depositors are entitled to receive an insurance
amount of ₹1 lakh per individual from DICGC
Which of the given above statements is/are correct?
(a) 3 only
(b) 2 and 3 only
(c) 1 only
(d) 1 and 2 only
Ans: (a)
Explanation:
 Deposit insurance is providing insurance protection to the depositor’s
money by receiving a premium.
 The government has set up Deposit Insurance and Credit Guarantee
Corporation (DICGC) under RBI to protect depositors if a bank fails.

Telegram: https://t.me/insightsIAStips
131
Youtube: https://www.youtube.com/channel/UCpoccbCX9GEIwaiIe4HLjwA
Revision Through MCQs (RTM) Compilation (November 2019)

 Every insured bank pays premium amounting to 0.001% of its deposits


to DICGC every year.
 When a bank is liquidated, depositors are entitled to receive an insurance
amount of ₹1 lakh per individual from the Deposit Insurance and
Credit Guarantee Corporation of India (DICGC).
 The ₹1 lakh insurance limit includes both principal and interest dues
across your savings bank accounts, current accounts, fixed deposits and
recurring deposits held with the bank.
Refer: https://www.insightsonindia.com/2019/11/20/what-is-deposit-insurance/

169. Consider the following statements with respect to NATGRID


1. NATGRID is a post Mumbai 26/11 attack measure
2. NATGRID, like a number of other government initiatives (UIDAI), established
through governmental notifications rather than legislation passed in
Parliament
3. Initially, no state agencies will be given direct access to the NATGRID data
4. The data recovery centre of the NATGRID has been constructed in Bengaluru
Which of the given above statements is/are correct?
(a) 1, 2 and 4
(b) 1 and 4
(c) 1, 2 and 3
(d) 1, 2, 3 and 4
Ans: (d)
Explanation:
 NATGRID is a post Mumbai 26/11 attack measure.
 NATGRID, like a number of other government initiatives (UIDAI), is being
established through governmental notifications rather than legislation
passed in Parliament.
 Initially, no state agencies will be given direct access to NATGRID
data but in case any relevant information is required, they can approach
NATGRID through any of 10 user agencies.
o In the first phase, 10 user’s agencies and 21 service providers
will be connected with the NATGRID while in the later stages 950
organizations and in subsequent years another 1,000
organization’s will be connected with it.
o The 10 user agencies which will be able to access the NATGRID
data on real-time basis are: Intelligence Bureau (IB), Research &
Analysis Wing (R&AW), Central Bureau of Investigation (CBI),
Enforcement Directorate (ED), Directorate of Revenue Intelligence
(DRI), Financial Intelligence Unit (FIU), Central Board of Direct Taxes
(CBDT), Central Board of Excise and Customs (CBEC), Directorate
General of Central Excise and Intelligence (DGCEI) and Narcotics
Control Bureau (NCB
 The data recovery centre of the NATGRID has been constructed in
Bengaluru and its headquarters is nearing completion in Delhi.
Refer: https://www.insightsonindia.com/2019/11/20/natgrid-2/

170. Consider the following in respect of Indian Ocean Naval Symposium (IONS):
1. Inaugural IONS was held in India in 2015 under the chairmanship of the
Indian Navy.

Telegram: https://t.me/insightsIAStips
132
Youtube: https://www.youtube.com/channel/UCpoccbCX9GEIwaiIe4HLjwA
Revision Through MCQs (RTM) Compilation (November 2019)

2. IONS is a voluntary initiative that seeks to increase maritime co-operation


among navy’s of the littoral states of the Indian Ocean Region.
Which of the above statements is/are correct?
(a) 1 only
(b) 2 only
(c) Both 1 and 2
(d) Neither 1 nor 2
Ans: (b)
Explanation:
 The ‘Indian Ocean Naval Symposium’ (IONS) is a voluntary initiative
that seeks to increase maritime co-operation among navies of the littoral
states of the Indian Ocean Region by providing an open and inclusive
forum for discussion of regionally relevant maritime issues.
 There are 35 members -navies of the IONS which have been
geographically grouped into the following four sub-regions:-South Asian
Littorals, West Asian Littorals, East African Littorals, South East Asian
and Australian Littorals.
 The inaugural IONS-2008 was held in New Delhi, India on 14 Feb 08.
 Indian Navy was designated the Chairman IONS for the period 2008-10.
 The theme of the IONS-2008 was “Contemporary Trans-national and
International Maritime Connectivity’s”.
 List of symposiums:

Refer: https://www.insightsonindia.com/2019/01/26/bandar-abbas-to-host-ions-
next-month/

RTM- REVISION THROUGH MCQS - 21th -Nov-2019

171. With reference to National Register of Citizens (NRC), consider the following
statements
1. The register was first prepared after the 9th in a series of census held in India
2. One of the eligibility criteria under NRC, D-Voters can apply for inclusion of
their names in the updated NRC only when the appropriate Foreigner
Tribunal declares them as non-foreigners
3. Initially, NRC was specifically made for a state, which was famous for Shirui
Lily and Cheiraoba festival
Which of the given above statements is/are not correct?
(a) 1 only

Telegram: https://t.me/insightsIAStips
133
Youtube: https://www.youtube.com/channel/UCpoccbCX9GEIwaiIe4HLjwA
Revision Through MCQs (RTM) Compilation (November 2019)

(b) 2 only
(c) 3 only
(d) 1 and 3 only
Ans: (c)
Explanation: Here Directive Word is Not Correct!!
 The NRC is the list of Indian citizens and was prepared in 1951,
following the census of 1951
o 1951 Census of India was the 9th in a series of censuses held
in India every decade since 1871.
o It is also the first census after independence and Partition of India.
o 1951 census was also the first census to be conducted under 1948
Census of India Act.
 The process of NRC update was taken up in Assam as per a Supreme
Court order in 2013.
 In order to wean out cases of illegal migration from Bangladesh and other
adjoining areas, NRC updation was carried out under The Citizenship
Act, 1955, and according to rules framed in the Assam Accord.
 ‘D’ voters can apply for inclusion of their names in the updated NRC.
However, their names will be finally included only when the appropriate
Foreigner Tribunal declares them as non-foreigners.
 festivals of Manipur are Lui-ngai-ni Ningol Chakouba, Shirui Lily
festival, Yaoshang, Gan-ngai, Chumpha, Cheiraoba, Kang and Heikru
Hidongba, as well as the broader religious festivals Eid-Ul-Fitr, Eid-Ul-
Adha and Christmas.
Refer: https://www.insightsonindia.com/2019/11/21/national-register-of-citizens-
nrc-7/

172. The Hong Kong International Convention for the Safe and Environmentally
Sound Recycling of Ships, 2009 (the Hong Kong Convention), was adopted by
(a) Directorate General of Shipping
(b) International Association of Ports and Harbors
(c) World Association for Waterborne Transport Infrastructure
(d) None of the above
Ans: (d)
Explanation:
 It was adopted by the International Maritime Organization (IMO) in
2009.
 IMO – the International Maritime Organization – is the United Nations
specialized agency with responsibility for the safety and security of
shipping and the prevention of marine and atmospheric pollution by
ships. IMO's work supports the UN SDGs.
Refer: https://www.insightsonindia.com/2019/11/21/recycling-of-ships-bill-
2019/

173. Arrange the following events in chronological order


1. Montagu-Chelmsford Report
2. Jallianwala Bagh massacre
3. Rowlatt Acts
4. Rabindranath Tagore renounced the knighthood
Select the correct answer using the code below:
(a) 1-2-3-4

Telegram: https://t.me/insightsIAStips
134
Youtube: https://www.youtube.com/channel/UCpoccbCX9GEIwaiIe4HLjwA
Revision Through MCQs (RTM) Compilation (November 2019)

(b) 3-1-2-4
(c) 3-2-1-4
(d) 1-3-2-4
Ans: (d)
Explanation:
 Montagu-Chelmsford Report went before Cabinet on 24 May and 7 June
1918 and was embodied in the Government of India Act of 1919
 Rowlatt Acts, (February 1919), legislation passed by the Imperial
Legislative Council, the legislature of British India. The acts allowed
certain political cases to be tried without juries and permitted internment
of suspects without trial
 13 April 1919-Jallianwala Bagh massacre
 Tagore returned his Knighthood for Services to Literature, which he was
awarded in 1915, in protest against the 1919 Amritsar Massacre
Refer: https://www.insightsonindia.com/2019/11/21/jallianwala-bagh-national-
memorial-amendment-bill-2019-2/

174. Code on Wages, 2019 will amalgamate


1. Payment of Wages Act, 1936
2. Minimum Wages Act, 1948
3. Payment of Bonus Act, 1965
4. Payment of Gratuity Act 1972
Select the correct answer using the code below:
(a) 1, 2 and 3
(b) 1, 3 and 4
(c) 2, 3 and 4
(d) All of the above
Ans: (a)
Explanation:
 The Centre will soon notify the rules that will create the mechanisms to
fix a floor wage that would then undergird the minimum wages for
different categories of workers — unskilled, semi-skilled, skilled and
highly skilled — that the States and Central government would have to
set and enforce.
 This is in accordance with the Code on Wages, 2019.
 The new code will amalgamate the Payment of Wages Act, 1936, the
Minimum Wages Act, 1948, the Payment of Bonus Act, 1965, and
the Equal Remuneration Act, 1976.
Refer: https://www.insightsonindia.com/2019/11/21/new-code-on-wages-4/

175. Recently 3S programme has been in news for sometimes is primarily related to
(a) Banking reform’s
(b) Skill development
(c) Data management
(d) Public Healthcare
Ans: (d)
Explanation:
 The 3S project was recommended by the World Health Organization
(WHO), considering the limited safety data on vaccines introduced in
India.

Telegram: https://t.me/insightsIAStips
135
Youtube: https://www.youtube.com/channel/UCpoccbCX9GEIwaiIe4HLjwA
Revision Through MCQs (RTM) Compilation (November 2019)

 As part of the 3S project, India is evaluating the recently-introduced


rotavirus vaccines. It is also trying to strengthen the collaboration among
key stakeholders, such as ministry of health and Central Drugs Standard
Control Organisation (CDSCO), to ensure high levels of vigilance.
Refer: https://www.insightsonindia.com/2019/11/21/smart-safety-surveillance-
or-3s-programme/

176. Consider the following statements


1. National Institute of Sowa-Rigpa (NISR) is autonomous national institute
under MoH&FW
2. Sowa –Rigpa is a traditional system of medicine originated in Bhutan
Which of the given above statements is/are correct?
(a) 1 only
(b) 2 only
(c) Both1 and 2
(d) Neither 1 Nor 2
Ans: (d)
Explanation:
 National Institute of Sowa-Rigpa (NISR): It will be an autonomous
national institute under the Ministry of AYUSH with the mandate to
undertake interdisciplinary education and research programmes in
Sowa-Rigpa in collaboration with national and international institutes.
 It will act as an apex institute for Sowa-Rigpa system. The existing Sowa-
Rigpa institutions work under the Ministry of Culture.
 What is Sowa -Rigpa?
o It is a traditional system of medicine practised in the Himalayan
belt of India.
o It originated in Tibet and popularly practiced in countries
namely, India, Nepal, Bhutan, Mongolia, and Russia.
o The majority of theory and practice of Sowa-Rigpa is similar to
“Ayurveda”.
o Yuthog Yonten Gonpo from Tibet is believed to be the father of
Sowa Rigpa
Refer: facts For Prelims: https://www.insightsonindia.com/2019/11/21/insights-
daily-current-affairs-pib-21-november-2019/

177. Which of the following statements can help in water conservation in


agriculture?
1. Reduced or zero tillage of the land
2. Applying gypsum before irrigating the field
3. Allowing crop residue to remain in the field
Select the correct answer using the code given below:
(a) 1 and 2 only
(b) 3 only
(c) 1 and 3 only
(d) 1, 2 and 3
Ans: (d)
Explanation:
 No-till farming (also called zero tillage or direct drilling) is a way of
growing crops or pasture from year to year without disturbing the soil
through tillage.

Telegram: https://t.me/insightsIAStips
136
Youtube: https://www.youtube.com/channel/UCpoccbCX9GEIwaiIe4HLjwA
Revision Through MCQs (RTM) Compilation (November 2019)

 No-till is an agricultural technique which increases the amount of water


that infiltrates into the soil and increases organic matter retention and
cycling of nutrients in the soil.
 Gypsum improves the ability of soil to drain and not become waterlogged
thus increasing water-use efficiency of crops.

178. What is the application of Somatic Cell Nuclear Transfer Technology?


(a) Production of biolarvicides
(b) Manufacture of biodegradable plastics
(c) Reproductive cloning of animals
(d) Production of organisms free of diseases
Ans: (c)
Explanation:
 Somatic-cell nuclear transfer involves removing the nucleus, which
contains the DNA, and implanting it into an unfertilised egg, whose
nucleus has been removed. It is used in reproductive cloning of
organisms.
Refer: https://www.ifst.org/resources/information-statements/insight-animal-
cloning

179. Ministry of Earth Sciences has unveiled state-of-the-art Air Quality and
Weather Forecast System–SAFAR (System of Air Quality and Weather
Forecasting). It covers which of the following pollutants?
1. PM2.5 and PM10
2. Ozone
3. Carbon dioxide
4. Methane
Select the correct answer using the codes below.
(a) 1, 2 and 3 only
(b) 1, 2, 3 and 4
(c) 1, 2 and 4 only
(d) 1 and 4 only
Ans: (c)
Explanation:
 It is first of its kind and most advanced system in India.
 SAFAR was developed indigenously by Indian Institute of Tropical
Meteorology (IITM), Pune and operationalized by India Meteorological
Department (IMD).
 It has been introduced for greater metropolitan cities of India to provide
location-specific information on air quality in near real time and its
forecast 1-3 days in advance for the first time in India.
 It is integral part of India’s first Air Quality Early Warning System
operational in Delhi. It will monitor all weather parameters like
temperature, rainfall, humidity, wind speed and wind direction.
 Pollutants monitored: PM1, PM2.5, PM10, Ozone, CO, NOx (NO, NO2),
SO2, BC, Methane (CH4),Non-methane hydrocarbons (NMHC), VOC’s,
Benzene, Mercury.
 Monitored Meteorological Parameters: UV Radiation, Rainfall,
Temperature, Humidity, Wind speed, Wind direction, solar radiation.
 Significance of the system:

Telegram: https://t.me/insightsIAStips
137
Youtube: https://www.youtube.com/channel/UCpoccbCX9GEIwaiIe4HLjwA
Revision Through MCQs (RTM) Compilation (November 2019)

oIt has been introduced for greater metropolitan cities of India to


provide location-specific information on air quality in near real
time and its forecast 1-3 days in advance for the first time inIndia.
o The World Meteorological Organization has recognised SAFAR as a
prototype activity on the basis of the high quality control and
standards maintained in its implementation.
o The ultimate objective of the project is to increase awareness
among general public regarding the air quality in their city well in
advance so that appropriate mitigation measures and systematic
action can be taken up for betterment of air quality and related
health issues.
Refer: https://www.insightsonindia.com/2019/11/02/public-health-emergency-
declared-in-delhi/

180. The Rajya Sabha does not have a procedure for moving of
1. Adjournment motion
2. Censure motion
3. No-confidence motion
Select the correct answer using the codes below.
(a) 1 only
(b) 2 and 3 only
(c) 1, 2 and 3
(d) 1 and 3 only
Ans: (c)
Explanation:
 S1 and S2 are not possible because these are motions that censure the
government, and Rajya Sabha does not enjoy this power.
 Rule 198 of the Rules of Procedure and Conduct of Business in Lok
Sabha lays down the procedure for moving a Motion of No-Confidence
in the Council of Ministers.
o There is no mention of a no-confidence motion in the constitution.
o A no-confidence motion is a parliamentary motion which is moved
in the Lok Sabha against the entire council of ministers, stating
that they are no longer deemed fit to hold positions of
responsibility due to their inadequacy in some respect or their
failure to carry out their obligations.
o No prior reason needs to be stated for its adoption in the Lok
Sabha
Refer: https://www.insightsonindia.com/2018/07/20/insights-daily-current-
affairs-20-july-2018/

RTM- REVISION THROUGH MCQS - 22th -Nov-2019

181. “Rights of Citizenship of Certain Persons of Indian Origin Residing Outside


India” has been enshrined in the constitution of India under the article
(a) Article 5
(b) Article 6
(c) Article 7
(d) Article 8
Ans: (d)

Telegram: https://t.me/insightsIAStips
138
Youtube: https://www.youtube.com/channel/UCpoccbCX9GEIwaiIe4HLjwA
Revision Through MCQs (RTM) Compilation (November 2019)

Explanation:
 The eighth article reads that any person who or either of whose parents
or grandparents were born in India as stated in the Government of India
Act 1955 and who is residing ordinarily in any country outside India
shall be considered to be a citizen of India.
 If he has registered as a Citizen of India by an Indian diplomatic or
consular representative in that country on an application made by him or
her in the prescribed document form to such diplomatic or consular
representative, whether before or after the commencement of the
Constitution.
Refer: https://www.insightsonindia.com/2019/11/22/mlas-citizenship-revoked-
for-hiding-foreign-visits/

182. With reference to K12 Education Transformation Framework, consider the


following statements
1. It was rolled out by Microsoft with intention to improve high school
education
2. It comprises four pillars strategy including parents
Which of the given above statements is/are not correct?
(a) 1 only
(b) 2 only
(c) Both 1 and 2
(d) Neither 1 Nor 2
Ans: (c)
Explanation: Here Directive Word is Not Correct!!
K12 Education Transformation Framework by Microsoft
 The Framework provides a holistic framework to facilitate a
comprehensive digital transformation of schools.
 The framework is a flexible platform based on the latest research and
input from hundreds of academics, experts, and policymakers.
 It comprises four pillars — leadership and policy, modern teaching and
learning, intelligent environments and technology blueprint.
Refer: https://www.insightsonindia.com/2019/11/22/k12-education-
transformation-framework/

183. Recently Arundhati Swarna Yojana has been rolled out by


(a) State of Assam
(b) State of Gujarat
(c) State of Maharashtra
(d) State of Karnataka
Ans: (a)
Explanation: Assam government has announced the launch of ‘Arundhati
Swarna Yojana’
 Under the scheme, the state government will offer 10 grams of gold as a
gift to every adult bride who has completed at least 10th standard and
has registered her marriage.
 Note: The government will not give the gold directly but Rs. 30,000 to
purchase 10 gms of gold.
Refer: https://www.insightsonindia.com/2019/11/22/arundhati-swarna-yojana/

Telegram: https://t.me/insightsIAStips
139
Youtube: https://www.youtube.com/channel/UCpoccbCX9GEIwaiIe4HLjwA
Revision Through MCQs (RTM) Compilation (November 2019)

184. IMI 2.0 has been in news for sometimes is primarily related to
(a) Restructuring of stressed assets in banking sector
(b) Debt swap agreement with Japan
(c) Renaissance of ISIS terrorist group
(d) Public Health care
Ans: (d)
Explanation: Intensified Mission Indradhanush 2.0 (IMI 2.0):
 To ensure that not a single child in the country misses out on
vaccination, the government has launched the ‘Intensified Mission
Indradhanush 2.0’ with a special focus on improving coverage in areas
with “low” immunization .
 Through ‘IMI 2.0’, the health ministry aims to reach each and every child
below the age of two years and all pregnant women still
uncovered/partially covered in 271 districts of the country and 652
blocks of Uttar Pradesh and Bihar.
Refer: https://www.insightsonindia.com/2019/11/22/in-news-intensified-
mission-indradhanush/

185. With reference to Avian botulism, consider the following statements


1. It refers to strain of botulism that affects wild and captive bird populations,
most notably waterfowl
2. Humans are generally considered resistant to type E avian botulism
3. It is a paralytic disease brought on by the the Botulinum neurotoxin (BoNt)
of the bacterium Clostridium botulinum
Which of the given above statements is/are correct?
(a) 1 and 3
(b) 1 only
(c) 2 and 3
(d) 3 only
Ans: (a)
Explanation:
 Avian Botulism is a strain of botulism that affects wild and captive bird
populations, most notably waterfowl.
 This is a paralytic disease brought on by the Botulinum neurotoxin
(BoNt) of the bacterium Clostridium botulinum.
 Type C BoNt is most frequently associated with waterfowl mortality.
 Humans are generally considered resistant to type C avian botulism.
 Type E avian botulism can cause illness in humans, but this is very
rare.
Refer: https://www.insightsonindia.com/2019/11/22/avian-botulism/

186. Consider the following statements regarding Convention on Cybercrime


1. It is the first international treaty seeking to address Internet and computer
crime
2. It is open for ratification even to states that are not members of the Council
of Europe
3. It was drawn up International Telecommunications Union (ITU)
Which of the given above statements is/are correct?
(a) 1 and 2
(b) 1 only
(c) 2 and 3

Telegram: https://t.me/insightsIAStips
140
Youtube: https://www.youtube.com/channel/UCpoccbCX9GEIwaiIe4HLjwA
Revision Through MCQs (RTM) Compilation (November 2019)

(d) 3 only
Ans: (a)
Explanation:
 The Convention on Cybercrime, also known as the Budapest
Convention on Cybercrime or the Budapest Convention, is the first
international treaty seeking to address Internet and computer crime by
harmonizing national laws, improving investigative techniques, and
increasing cooperation among nations.
 It was drawn up by the Council of Europe in Strasbourg, France, with
the active participation of the Council of Europe’s observer states
Canada, Japan, South Africa and the United States.
 It is open for ratification even to states that are not members of the
Council of Europe.
 As of September 2019, 64 states have ratified the convention.
Refer: https://www.insightsonindia.com/2019/11/22/budapest-convention-on-
cyber-security/

187. Which of the following are correctly matched


1. Prithvi-II missile- Short-range ballistic missile
2. Agni missiles- Medium to Intercontinental range ballistic missiles
3. K family of missiles - Submarine-launched ballistic missiles
Select the correct answer using the code below:
(a) 1 and 2 only
(b) 2 and 3 only
(c) 1, 2 and 3
(d) 2 only
Ans: (c)
Explanation:
 All the options are correct
 K family of missiles named after Indian scientist and former president A.
P. J. Abdul Kalam, is a series of submarine-launched ballistic missiles
developed by India to boost its second strike capabilities and thus
augment its nuclear deterrence
Refer: Facts for Prelims: https://www.insightsonindia.com/2019/11/22/insights-
daily-current-affairs-pib-22-november-2019/

188. With reference to Bacteriophage, consider the following statements


1. A bacteriophage is a type of virus that infects bacteria
2. All bacteriophages are composed of a nucleic acid molecule that is
surrounded by a protein structure
3. Bacteriophages occasionally remove a portion of their host cells' bacterial
DNA during the infection process and then transfer this DNA into the
genome of new host cells, the process is known as transcription
Which of the given above statements is/are correct?
(a) 1 and 3
(b) 1 and 2
(c) 1 only
(d) 1, 2 and 3
Ans: (b)
Explanation:
 A bacteriophage is a type of virus that infects bacteria(bacteria eater)

Telegram: https://t.me/insightsIAStips
141
Youtube: https://www.youtube.com/channel/UCpoccbCX9GEIwaiIe4HLjwA
Revision Through MCQs (RTM) Compilation (November 2019)

 All bacteriophages are composed of a nucleic acid molecule that is


surrounded by a protein structure.
 A bacteriophage attaches itself to a susceptible bacterium and infects the
host cell.
 Bacteriophages occasionally remove a portion of their host cells' bacterial
DNA during the infection process and then transfer this DNA into the
genome of new host cells. This process is known as transduction.
 Transcription is the first of several steps of DNA based gene expression,
in which a particular segment of DNA is copied into RNA by the enzyme
RNA polymerase
Refer: Facts for Prelims: https://www.insightsonindia.com/2019/11/22/insights-
daily-current-affairs-pib-22-november-2019/

189. Western Asia is located directly south of Eastern Europe. The region is
surrounded by seven major seas which include?
1. Black Sea
2. Red Sea
3. Aegean Sea
4. Caspian Sea
Select the correct answer using the codes below.
(a) 1 and 2 only
(b) 1, 2 and 4 only
(c) 3 and 4 only
(d) 1, 2, 3 and 4
Ans: (d)
Explanation:
 These are the Aegean Sea, the Black Sea, the Caspian Sea, the
Persian Gulf, the Arabian Sea, the Red Sea, and the Mediterranean
Sea.
 To the north, the region is delimited from Europe by the Caucasus
Mountains, to the southwest, it is delimited from Africa by the Isthmus of
Suez, while to the east, the region adjoins Central Asia and South Asia.

Telegram: https://t.me/insightsIAStips
142
Youtube: https://www.youtube.com/channel/UCpoccbCX9GEIwaiIe4HLjwA
Revision Through MCQs (RTM) Compilation (November 2019)

190. Consider a triangle that is formed from the following;


1. Southernmost point of Mediterranean Sea
2. Northernmost point of Black Sea
3. Southernmost point of Caspian Sea
Which of these nations would fall partially or fully in the triangle?
(1) Syria (2) Turkey (3) Iran (4) Kuwait
Select the correct answer using the codes below.
(a) 1 only
(b) 1, 2 and 3
(c) 2 and 3 only
(d) 1, 2, 3 and 4
Ans: (b)
Explanation:

Telegram: https://t.me/insightsIAStips
143
Youtube: https://www.youtube.com/channel/UCpoccbCX9GEIwaiIe4HLjwA
Revision Through MCQs (RTM) Compilation (November 2019)

RTM- REVISION THROUGH MCQS - 23th -Nov-2019

191. With reference to Indian Parliament, consider the following statements


1. Any MP who is not a member of lok sabha is referred to as a private member
2. Admissibility of a private member’s bill is the sole prerogative of speaker
3. While government Bills can be introduced and discussed on any day, private
member’s Bills can be introduced and discussed only on Fridays
Which of the give above statements is/are correct?
(a) 3 only
(b) 2 and 3 only
(c) 1 and 3 only
(d) 1, 2 and 3
Ans: (a)
Explanation:
 Any MP who is not a Minister is referred to as a private member.
 The admissibility is decided by the Chairman for Rajya Sabha and
Speaker in the case of Lok Sabha.The procedure is roughly the same for
both Houses:
 While government Bills can be introduced and discussed on any day,
private member’s Bills can be introduced and discussed only on Fridays
 Till date, Parliament has passed 15 Private Members' Bills. Five of
these were passed in 1956 alone and After 47 years of passing the last
bill, the Supreme Court (Enlargement of Criminal Appellate Jurisdiction)
Bill, 1968 that became an Act on 9 August 1970; Rajya Sabha has
passed the Rights of Transgender Persons Bill, 2014 on April 24, 2015,

Telegram: https://t.me/insightsIAStips
144
Youtube: https://www.youtube.com/channel/UCpoccbCX9GEIwaiIe4HLjwA
Revision Through MCQs (RTM) Compilation (November 2019)

which was introduced in the lower house, Lok Sabha on 26 February


2016
Refer: https://www.insightsonindia.com/2019/11/23/private-members-bill-2/

192. With reference to GST Council & its 36th meeting, consider the following
statements
1. GST rate on plates and cups made of flowers, leaves and bark reduced from
12% to 5 %
2. There is no GST payable on tamarind
3. 18% GST rate payable on Caffeinated Beverages
4. Woven/ Non-woven Polyethylene Packaging bags attracts 12% of GST rate
Select the correct answer using the code below:
(a) 1, 2 and 3
(b) 2 and 4
(c) 1, 3 and 4
(d) 3 and 4
Ans: (b)
Explanation: The following are the rate cuts announced at the 36th GST Council
meeting:

Telegram: https://t.me/insightsIAStips
145
Youtube: https://www.youtube.com/channel/UCpoccbCX9GEIwaiIe4HLjwA
Revision Through MCQs (RTM) Compilation (November 2019)


Refer: https://www.insightsonindia.com/2019/11/23/gst-council-3/

193. Which of the following statements is/are correct?


1. State of Jammu and Kashmir joined GST earlier this year
2. Under the GST law, Delhi and Pondicherry have State Legislatures, they are
treated as States under GST
3. GSTC is a constitutional body
4. Under GST law, Centre will have one-third weightage of the total votes cast
and the States together will have two-thirds weightage of the total votes cast.
Passing of a resolution requires the support of at least three-fourth
weightage of the total votes cast
Select the correct answer suing the code below:
(a) 1 and 3
(b) 2, 3 and 4
(c) 1, 3 and 4
(d) 3 and 4 only
Ans: (b)
Explanation:
 The Jammu and Kashmir Legislative Assembly passed the state GST Bill
2017 making it the last state of the country to join the new tax regime (
joined in 2017)
o Since Govt scraps 370 and splits in J&k into 2 UT’s- The state
goods and services tax (GST) will continue, says the Jammu and
Kashmir reorganisation legislation, but according to officials and
experts, it would be for the time being.
o Later, the UT GST Act has to be made for Ladakh and state GST
Act for J&K.
o According to experts, the reorganisation Act says the GST Act will
continue because in its absence there will be problems in
enforcing the indirect tax on the two union territories that will now
replace the state of Jammu and Kashmir.

Telegram: https://t.me/insightsIAStips
146
Youtube: https://www.youtube.com/channel/UCpoccbCX9GEIwaiIe4HLjwA
Revision Through MCQs (RTM) Compilation (November 2019)

o “Although J&K will continue with the SGST since it has an


assembly despite being union territory, Ladakh will require
UTGST, for which the Centre will pass an ordinance to quicken the
process,” an official said.
 Under the GST law, a State has been defined to include a Union
Territory having a Legislature. Given that both, Delhi and Pondicherry
have State Legislatures, they are treated as States under GST
 GSTC is a constitutional body for making recommendations to the
Union and State Government on issues related to Goods and Service Tax.
 The GST Council is chaired by the Union Finance Minister and other
members are the Union State Minister of Revenue or Finance and
Ministers in-charge of Finance or Taxation of all the State
 As per Article 279A of the amended Constitution, the GST Council
which will be a joint forum of the Centre and the States, shall consist of
the following members: -
o the Union Finance Minister, Union Minister of State in charge of
Revenue or Finance, the Minister in charge of Finance or Taxation
or any other Minister nominated by each State Government
 Presently, the law specifies that all decisions in the GST Council will be
taken by a weighted voting procedure where the Centre will have one-
third weightage of the total votes cast and the States will have two-thirds
weightage of the total votes cast. Passing of a resolution requires the
support of at least three-fourth weightage of the total votes cast.
Refer: https://www.insightsonindia.com/2019/11/23/gst-council-3/

194. Consider the following statements related to One Stop Centre (OSC) scheme
1. It is a Centrally Sponsored Scheme
2. It falls under the MHA
3. OSC will support all women excluding girls below 18 years of age affected by
violence, irrespective of caste, class, religion, region, sexual orientation or
marital status
Which of the given above statements is/are correct?
(a) 1 only
(b) 1 and 2
(c) 1 and 3
(d) 1, 2 and 3
Ans: (a)
Explanation: One Stop Centre (OSC) scheme
 Popularly known as Sakhi, Ministry of Women and Child Development
(MWCD) has formulated this Centrally Sponsored Scheme.
 It is a sub – scheme of Umbrella Scheme for National Mission for
Empowerment of women including Indira Gandhi Mattritav Sahyaog
Yojana.
 Under the scheme, One Stop Centres are being established across the
country to provide integrated support and assistance under one roof to
women affected by violence, both in private and public spaces in phased
manner.

Telegram: https://t.me/insightsIAStips
147
Youtube: https://www.youtube.com/channel/UCpoccbCX9GEIwaiIe4HLjwA
Revision Through MCQs (RTM) Compilation (November 2019)

 Target group: The OSC will support all women including girls below 18
years of ageaffected by violence, irrespective of caste, class, religion,
region, sexual orientation or marital status.
Refer: https://www.insightsonindia.com/2019/11/23/one-stop-centre-scheme-3/

195. What are the issues associated with Reverse Osmosis Plants
1. Deposition of brine along the shores
2. Reduced membrane performance by fouling
3. It can significantly reduce good minerals that can aid in overall heart and
muscle health
Which of the given above statements is/are correct?
(a) 1 and 3
(b) 1 only
(c) 1 and 2
(d) 1, 2 and 3
Ans: (d)
Explanation:
 All the given statements are considered as drawback of Reverse Osmosis
Plants
Refer: https://www.insightsonindia.com/2019/11/23/reverse-osmosis-ro/

196. Recently Living root bridges are in news for sometimes related to
(a) They are handmade from the aerial roots of Ficus elastic trees
(b) They are handmade from the aerial roots of Ocimum tenuiflorum
(c) They are handmade from the aerial roots of Azadirachta indica
(d) They are handmade from the aerial roots of Mangifera indica
Ans: (a)
Explanation:
 Living root bridges are a form of tree shaping common in the southern
part of the Northeast Indian state of Meghalaya.
 They are handmade from the aerial roots of rubber fig trees (Ficus
elastic) by the Khasi and Jaintia peoples of the mountainous terrain
along the southern part of the Shillong Plateau.
 Root bridges have also been observed in the Indian state of Nagaland
Refer: Facts for Prelims: (https://www.insightsonindia.com/2019/11/23/insights-
daily-current-affairs-pib-23-november-2019/)

197. Common Services Centers (CSCs) are a strategic cornerstone of which of the
following flagship
Programmes of the Government of India?
(a) MGNREGA
(b) Digital India programme
(c) National Nutrition Mission
(d) Sarva Shiksha Abhiyan (SSA)
Ans: (b)
Explanation:
 Even though it might be useful in all major schemes, it was seen as a
strategic intervention for Digital India to succeed.

Telegram: https://t.me/insightsIAStips
148
Youtube: https://www.youtube.com/channel/UCpoccbCX9GEIwaiIe4HLjwA
Revision Through MCQs (RTM) Compilation (November 2019)

 CSCs are the access points for delivery of various electronic services to
villages in India, thereby contributing to a digitally and financially
inclusive society.
 CSCs enable the three vision areas of the Digital India programme:
o Digital infrastructure as a core utility to every citizen.
o Governance and services on demand.
o Digital empowerment of citizens.
 Significance of CSCs:
 CSCs are more than service delivery points in rural India. They are
positioned as change agents, promoting rural entrepreneurship and
building rural capacities and livelihoods. They are enablers of community
participation and collective action for engendering social change through
a bottom-up approach with key focus on the rural citizen.
 The CSC project, which forms a strategic component of the National e-
Governance Plan was approved by the Government in May 2006, as part
of its commitment in the National Common Minimum Programme to
introduce e-governance on a massive scale.

198. Development expenditure of the central government includes;


1. Expenditure on social and community services
2. Defence expenditure
3. Grants to states
4. Expenditure on economic services
Select the correct answer using the codes below;
(a) 1, 3 and 4 only
(b) 1 and 3 only
(c) 1 only
(d) 1, 2, 3 and 4
Ans: (a)
Explanation:
 Development expenditure: It is that expenditure which is incurred on
economic and social development of the country. Example: expenditure
on education, health, employment, industry etc
 Defence expenditure is not treated as developmental expenditure.

199. Which of these seas is NOT included in the China Seas?


(a) The Yellow Sea
(b) The East China Sea
(c) The South China Sea
(d) The Cametos Sea
Ans: (d)
Explanation:
 The China Seas consist of a series of marginal seas in the Western Pacific
Ocean, around China. They are the major components signifying the
transition from the continent of Asia to the Pacific Ocean. They have been
described in terms of their collective vastness and complexity: The four
seas of China, the Bohai Sea, the Huanghai Sea, the East China Sea, and

Telegram: https://t.me/insightsIAStips
149
Youtube: https://www.youtube.com/channel/UCpoccbCX9GEIwaiIe4HLjwA
Revision Through MCQs (RTM) Compilation (November 2019)

the South China Sea, occupy a total area of about 4.7 million km2, half
of the area of China mainland.
 These seas are located in the southeastern margin of the Eurasian
continent and subject to the interactions between the Eurasian, Pacific,
and Indian-Australian plates. The seas have complicated geology and rich
natural resources.

 ;

200. The mercantilist theory of Economics growth is primarily concerned with an


increase in the;
(a) Volume of money in circulation through import of gold
(b) Size of population
(c) State intervention in the economy
(d) Surplus in the balance of trade
Ans: (d)
Explanation:
 Mercantilism is an economic theory that advocates government
regulation of international trade to generate wealth and strengthen
national power.

Telegram: https://t.me/insightsIAStips
150
Youtube: https://www.youtube.com/channel/UCpoccbCX9GEIwaiIe4HLjwA
Revision Through MCQs (RTM) Compilation (November 2019)

 Merchants and the government work together to reduce the trade deficit
and create a surplus. It funds corporate, military, and national growth.
Mercantilism is a form of economic nationalism.
 It advocates trade policies that protect domestic industries.
 The mercantilist theory of economic growth is primarily concerned with
an increase in the surplus in the balance of trade.

RTM- REVISION THROUGH MCQS - 25th -Nov-2019

201. Consider the following statements


1. Judicial review shall be applicable to the proclamation of the president calling
for President’s rule in a state.
2. Relevant material is necessary for the satisfaction of the president to apply
president’s rule. The satisfaction of the President must be based on relevant
material. In case the court find out that the President’s action was based on
extraneous grounds, it could be struck down or declared mala fide.
3. The centre shall bear the burden of proving that sufficient and relevant
materials exist for justification of imposition of President’s Rule.
In which of the following case Supreme Court issues above mentioned verdict?
(a) S. R. Bommai case of 1994
(b) Kihoto Hollohan case of 1992
(c) Waman Rao case of 1981
(d) Indira Gandhi case of 1975
Ans: (a)
Explanation:
 Supreme Court issued the historic order in 1994 in SR Bommai case,
which in a way put an end to the arbitrary dismissal of State
governments under Article 356 by spelling out restrictions.
 The verdict concluded that the power of the President to dismiss a State
government is not absolute.
 The verdict said the President should exercise the power only after his
proclamation (imposing his/her rule) is approved by both Houses of
Parliament.
 Till then, the Court said, the President can only suspend the Legislative
Assembly by suspending the provisions of Constitution relating to the
Legislative Assembly.
 Also, the concept of floor test was introduced by the SC’s in this
judgement and referred to Article 164 of the constitution.
Refer: https://www.insightsonindia.com/2019/11/25/maharashtra-floor-test-
plea-and-supreme-courts-demands/

202. Which of the following article enables the Supreme Court to review its own
judgments, subject to the provisions of any law made by Parliament?
(a) 137
(b) 145
(c) 142
(d) 138

Telegram: https://t.me/insightsIAStips
151
Youtube: https://www.youtube.com/channel/UCpoccbCX9GEIwaiIe4HLjwA
Revision Through MCQs (RTM) Compilation (November 2019)

Ans: (a)
Explanation:
 Under Article 137, the Supreme Court has the power to review any of its
judgments or orders.
 Scope for review:
o When a review takes place, the law is that it is allowed not to take
fresh stock of the case but to correct grave errors that have
resulted in the miscarriage of justice.
o The court has the power to review its rulings to correct a “patent
error” and not “minor mistakes of inconsequential import”.
Refer: https://www.insightsonindia.com/2019/11/25/review-petition/

203. Every person including the disabled has his life and liberty guaranteed under
(a) Article 15
(b) Article 19
(c) Article 20
(d) Article 21
Ans: (d)
Explanation: Under the Constitution the disabled have been guaranteed the
following fundamental rights:
 Article 15(1) enjoins on the Government not to discriminate against any
citizen of India (including disabled) on the ground of religion, race, caste,
sex or place of birth.
 Article 15 (2) States that no citizen (including the disabled) shall be
subjected to any disability, liability, restriction or condition on any of the
above grounds in the matter of their access to shops, public restaurants,
hotels and places of public entertainment or in the use of wells, tanks,
bathing ghats, roads and places of public resort maintained wholly or
partly out of government funds or dedicated to the use of the general
public. Women and children and those belonging to any socially and
educationally backward classes or the Scheduled Castes & Tribes can be
given the benefit of special laws or special provisions made by the State.
 There shall be equality of opportunity for all citizens (including the
disabled) in matters relating to employment or appointment to any office
under the State.
 No person including the disabled irrespective of his belonging can be
treated as an untouchable. It would be an offence punishable in
accordance with law as provided by Article 17 of the Constitution.
 Every person including the disabled has his life and liberty
guaranteed under Article 21 of the Constitution.
 There can be no traffic in human beings (including the disabled), and
beggar and other forms of forced labour is prohibited and the same is
made punishable in accordance with law (Article 23).
 Article 24 prohibits employment of children (including the disabled)
below the age of 14 years to work in any factory or mine or to be engaged
in any other hazardous employment. Even a private contractor acting for

Telegram: https://t.me/insightsIAStips
152
Youtube: https://www.youtube.com/channel/UCpoccbCX9GEIwaiIe4HLjwA
Revision Through MCQs (RTM) Compilation (November 2019)

the Government cannot engage children below 14 years of age in such


employment.
 Article 25 guarantees to every citizen (including the disabled) the right to
freedom of religion. Every disabled person (like the non-disabled) has the
freedom of conscience to practice and propagate his religion subject to
proper order, morality and health.
 Every disabled person can move the Supreme Court of India to enforce
his fundamental rights and the rights to move the Supreme Court is itself
guaranteed by Article 32.
Refer: https://www.insightsonindia.com/2019/11/25/chhattisgarh-panchayats-
to-have-disabled-quota/

204. With reference to Exchange Traded Funds, consider the following statements
1. Unlike regular mutual funds, an ETF trades like a common stock on a stock
exchange
2. Traded prices of ETF don’t change throughout the day like any other stock
3. Trading value of an ETF is based on the net asset value of the underlying
stocks that an ETF represents
Which of the above given statements is/are correct?
(a) 1 and 3 only
(b) 3 only
(c) 1 and 2 only
(d) 2 and 3 only
Ans: (a)
Explanation:
 Exchange Traded Funds (ETFs) are mutual funds listed and traded on
stock exchanges like shares.
 Typically, an ETF mirrors a particular index, which means the group of
stocks in the ETF would be similar to those in the index that it is
benchmarked to.
 Usually, ETFs are passive funds where the fund manager doesn’t select
stocks on your behalf. Instead, the ETF simply copies an index and
endeavours to accurately reflect its performance.
 The traded price of an ETF changes throughout the day like any other
stock, as it is bought and sold on the stock exchange.

Refer: https://www.insightsonindia.com/2019/11/25/exchange-traded-funds-2/

205. Construction of Daudhan dam will result into submergence of 10% of critical
tiger habitat of MP’s. It is referring to
(a) Kanha Tiger Reserve
(b) Pench Tiger Reserve
(c) Panna Tiger Reserve
(d) Betla Tiger Reserve
Ans: (c)
Explanation:
 About Ken- Betwa project:

Telegram: https://t.me/insightsIAStips
153
Youtube: https://www.youtube.com/channel/UCpoccbCX9GEIwaiIe4HLjwA
Revision Through MCQs (RTM) Compilation (November 2019)

 Conceived as a two-part project, this is the country’s first river


interlinking project. It is perceived as a model plan for similar interstate
river transfer missions.
 The ₹18,000-crore project has been mired in several hurdles. The most
recent one is a disagreement between the States on the share of water.
There are outstanding environmental obstructions too. It involves
deforesting a portion of the Panna Tiger Reserve.

Refer: https://www.insightsonindia.com/2019/11/25/ken-betwa-river-
interlinking-project/

206. “It is a community-owned traditional harvested rainwater storage wetland


principally used for effectively harnessing water resources in the states of
Haryana, Rajasthan, Punjab, and western Uttar Pradesh of North India, that
collects and stores water throughout the year, to be used for the purpose of
recharging the groundwater in the nearby water wells, washing, bathing and
drinking by humans and cattle”
The above given passage describes, which of the following rain water harvesting
system?
(a) Suranga Bawadi
(b) Johad
(c) Kere
(d) Diggi
Ans: (b)
Explanation:
 Johads are community-owned rainwater storage wetland mainly used for
harnessing water resources.
 Haryana state government has come out with a plan of rehabilitating over
16,400 ponds in rural areas across the state in order to analyze pond
water to ascertain its suitability for irrigation and other uses.

Telegram: https://t.me/insightsIAStips
154
Youtube: https://www.youtube.com/channel/UCpoccbCX9GEIwaiIe4HLjwA
Revision Through MCQs (RTM) Compilation (November 2019)

Refer: Facts for Prelims: https://www.insightsonindia.com/2019/11/25/insights-


daily-current-affairs-pib-25-november-2019/

207. Which of the following rhino is the smallest of the five extant rhino species in
the world?
(a) White Rhino
(b) Black Rhino
(c) Greater One-Horned Rhinoceros
(d) None of the above
Ans: (d)
Explanation:
 The Sumatran rhino (IUCN-Critically Endangered) is the smallest of the
five extant rhino species in the world. The other species include the White
Rhino, the Black Rhino, the Greater One-Horned Rhinoceros and the
Javan Rhino.
Refer: Facts for Prelims: https://www.insightsonindia.com/2019/11/25/insights-
daily-current-affairs-pib-25-november-2019/

208. If you travel by road from Leh (Ladakh) to Yanam (Puducherry), what is the
minimum number of states within India through which you can travel, excluding
origin and destination?
(a) 4
(b) 5
(c) 6
(d) 7
Ans: (a)
Explanation:

Telegram: https://t.me/insightsIAStips
155
Youtube: https://www.youtube.com/channel/UCpoccbCX9GEIwaiIe4HLjwA
Revision Through MCQs (RTM) Compilation (November 2019)


Refer: Facts for Prelims: https://www.insightsonindia.com/2019/11/25/insights-
daily-current-affairs-pib-25-november-2019/

209. P-Series Fuels are a blend of


1. Ethanol
2. Natural gas liquids
3. Methyl tetrahydrofuran (MeTHF)
4. Biodiesel
Select the correct answer using the codes below.
(a) 1 and 2 only
(b) 3 only
(c) 2, 3 and 4 only
(d) 1, 2 and 3 only
Ans: (d)
Explanation:
 P-series fuels are a family of renewable, non-petroleum, liquid fuels that
can substitute for gasoline.
 P-Series fuels (alternative fuels) are a blend of ethanol, natural gas
liquids and methyl tetrahydrofuran (MeTHF), a co-solvent derived from
biomass.

Telegram: https://t.me/insightsIAStips
156
Youtube: https://www.youtube.com/channel/UCpoccbCX9GEIwaiIe4HLjwA
Revision Through MCQs (RTM) Compilation (November 2019)

 P-Series fuels are clear, high-octane alternative fuels that can be used in
flexible fuel vehicles.
 P-Series fuel addresses three problems: the need for non-petroleum
energy sources, solid waste management, and affordability. Using
feedstock with a negative cost - that means waste that municipalities
would otherwise pay to have hauled away - allows the fuel's selling price
to be about the same as mid-grade gasoline. It also gives urban areas
control over a large portion of the generated trash stream without relying
on burning, burying, or bequeathing it to other states. The feedstock is
not incinerated, but chemically digested, so there is no combustion with
the accompanying toxic air emissions.

210. Consider the following statements.


1. Common Risk Mitigation Mechanism (CRMM), established under the Sendai
framework, intends to enlarge the total size of the renewable energy market
by making investments in renewables widely available to institutional
investors.
2. The Cities Climate Finance Leadership Alliance’s mission aims to catalyse
investment in low-carbon and climate-resilient infrastructure in cities across
the World.
3. Climate Finance Leadership Initiative works to fulfill the private financing
objectives included in the 2015 Paris Climate Change Agreement.
Select the correct answer using the codes below.
(a) 1 and 3 only
(b) 2 and 3 only
(c) 2 only
(d) 1, 2 and 3
Ans: (b)
Explanation:
 Statement 1: The historic Paris Agreement of 2015 stated “Making
finance flows consistent with a pathway towards low greenhouse gas
emissions and climate-resilient development“(article 2.c). To achieve this
objective, on May 18th 2017 in Delhi, understanding that one of the
biggest obstacles to solar deployment is the cost of capital (~70% of total
installation costs), 16 countries of the ISA jointly commissioned of a
study to define and structure the Common Risk Mitigation
Mechanism (CRMM) for solar power generation projects. This
instrument, if implemented, will dramatically lower the cost of finance for
renewable energy and the overall price of electricity. Even more
significantly, CRMM will vastly enlarge the total size of the renewable
energy market by making investments in renewables widely available to
institutional investors.
 Statement 3: The Climate Finance Leadership Initiative will work to
fulfill the private financing objectives included in the landmark 2015
Paris Climate Change Agreement, which reaffirmed the goal of
mobilizing at least $100 billion per year by 2020 through a combination
of public development finance and private foreign direct investment.
Recognizing that the global response to climate change must include

Telegram: https://t.me/insightsIAStips
157
Youtube: https://www.youtube.com/channel/UCpoccbCX9GEIwaiIe4HLjwA
Revision Through MCQs (RTM) Compilation (November 2019)

robust sub-national action, particularly in urban areas, coalitions of


cities, banks, national governments and civil society organizations
launched the Cities Climate Finance Leadership Alliance at the UN
Secretary- General’s Climate Summit on 23 September 2014.
 Statement 2: The Cities Climate Finance Leadership Alliance’s
mission is to catalyse and accelerate additional capital flows to cities,
maximize investment in low-carbon and climate-resilient infrastructure,
and close the investment gap in urban areas over the next fifteen years.
 The Alliance is a coalition of thirty six leading institutions that are
committed to mobilize finance for low-emission and climate-resilient
infrastructure, in particular for the benefit of poor and vulnerable
communities in cities. Alliance members commit to reinforce and expand
their existing programs to support city leaders and city institutions, as
well as to share information, coordinate activities, and seek synergies
among their individual efforts.

RTM- REVISION THROUGH MCQS - 26th -Nov-2019

211. Consider the following statements


1. Samvidhan Divas is celebrated every year on November 26 to mark the day
on which the Constitution of India was adopted
2. Former Prime Minister of India Manmohan Singh announced that 26
November will be celebrated every year as "Constitution Day"
Which of the above given statements is/are correct?
(a) 1 only
(b) 2 only
(c) Both 1 and 2
(d) Neither 1 Nor 2
Ans: (a)
Explanation:
 Government of India declared 26 November as Constitution Day on 19
November 2015 by a gazette notification.
 The Prime Minister of India Narendra Modi made the declaration on
11 October 2015 while laying the foundation stone of the B. R.
Ambedkar's Statue of Equality memorial in Mumbai.
 The year of 2015 was the 125th birth anniversary of Ambedkar, who had
chaired the drafting committee of the Constituent Assembly and played
a pivotal role in the drafting of the constitution.
 Previously this day was celebrated as Law Day.
 26 November was chosen to spread the importance of the constitution
and to spread thoughts and ideas of Ambedkar.
Refer: https://www.insightsonindia.com/2019/11/26/constitution-day-of-india-3/

212. Consider the following statements with reference to Rules of Procedure and
Conduct of Business in parliament
1. Article 118(1) of the Constitution empowers each House of Parliament to
make rules for regulating its Procedure and the Conduct of its business

Telegram: https://t.me/insightsIAStips
158
Youtube: https://www.youtube.com/channel/UCpoccbCX9GEIwaiIe4HLjwA
Revision Through MCQs (RTM) Compilation (November 2019)

2. Rules of Procedure and Conduct of Business in the House of the People were
amended by the Speaker from time to time on the recommendations of the
subordinate legislation committee of the House until 1954
Which of the above given statements is/are correct?
(a) 1 only
(b) 2 only
(c) Both 1 and 2
(d) Neither 1 Nor 2
Ans: (a)
Explanation:
 Article 118(1) of the Constitution empowers each House of Parliament to
make rules for regulating its Procedure and the Conduct of its business
 The Constituent Assembly (Legislative) Rules of Procedure and
Conduct of Business in force immediately before the commencement
of the Constitution of India were modified and adopted by the
Speaker of Lok Sabha in exercise of the powers conferred on the
Speaker by article 118(2) of the Constitution and published under the
title “Rules of Procedure and Conduct of Business in the House of
the People” in the Gazette of India Extraordinary dated the 17th
April, 1952.
 Those Rules were amended by the Speaker from time to time on the
recommendations of the Rules Committee of the House until
September, 1954
Refer: https://www.insightsonindia.com/2019/11/26/rules-for-the-conduct-of-
business/

213. “This Committee consists of 15 members including the Speaker who is the ex-
officio Chairman of the Committee. The members are nominated by the Speaker.
The Committee considers matters of procedure and conduct of business in the
House and recommends any amendments or additions to the Rules of Procedure
and Conduct of Business in Lok Sabha that are considered necessary”
The above given passage describes which of the following parliamentary
committee?
(a) Business Advisory Committee
(b) Committee on Papers Laid on the Table
(c) Committee on Subordinate Legislation
(d) Rules Committee
Ans: (d)
Explanation:
 There shall be a Committee on Rules to consider matters of procedure
and conduct of business in the House and to recommend any
amendments or additions to these rules that may be deemed necessary.
 The Committee on Rules shall be nominated by the Speaker and shall
consist of fifteen members including the Chairperson of the Committee.
 The Speaker shall be the ex-officio Chairperson of the Committee.
Refer: https://www.insightsonindia.com/2019/11/26/rules-for-the-conduct-of-
business/

Telegram: https://t.me/insightsIAStips
159
Youtube: https://www.youtube.com/channel/UCpoccbCX9GEIwaiIe4HLjwA
Revision Through MCQs (RTM) Compilation (November 2019)

214. With reference to Credit-linked Subsidy Services Awas Portal, which of the
following are its significance
1. The scheme will expand institutional credit flow to the housing needs of
urban poor
2. Interest subsidy will be credited upfront to the loan account of beneficiaries
through Primary Lending Institutions
3. Credit linked subsidy will be available only for loan amounts upto Rs 6 lakhs
4. HUDCO is the Central Nodal Agency (CNA) to channelize this subsidy to the
lending institutions
Which of the above given statements is/are correct?
(a) 2 and 3
(b) 2 only
(c) 1, 2 and 4
(d) 1, 2, 3 and 4
Ans: (d)
Explanation:
 The Mission, in order to expand institutional credit flow to the housing
needs of urban poor will implement credit linked subsidy component as
a demand side intervention.
 Interest subsidy will be credited upfront to the loan account of
beneficiaries through Primary Lending Institutions resulting in reduced
effective housing loan and Equated Monthly Instalment (EMI). The Net
Present Value (NPV) of the interest subsidy will be calculated at a
discount rate of 9 %.
 The credit linked subsidy will be available only for loan amounts
upto Rs 6 lakhs and additional loans beyond Rs. 6 lakhs, if any, will be
at nonsubsidized rate.
 Housing and Urban Development Corporation (HUDCO) and National
Housing Bank (NHB) have been identified as Central Nodal Agencies
(CNAs) to channelize this subsidy to the lending institutions and for
monitoring the progress of this component. Ministry may notify other
institutions as CNA in future.
Refer: https://www.insightsonindia.com/2019/11/26/global-housing-technology-
challenge-2/

215. With reference to Coalbed Methane (CBM), consider the following statements
1. CBM is an conventional form of natural gas found in coal deposits or coal
seams
2. CBM can be used as feedstock for fertilisers
3. Best prospective areas for CBM development are in eastern India
Which of the above given statements is/are correct?
(a) 1 only
(b) 2 and 3
(c) 2 only
(d) 1, 2 and 3
Ans: (b)
Explanation:

Telegram: https://t.me/insightsIAStips
160
Youtube: https://www.youtube.com/channel/UCpoccbCX9GEIwaiIe4HLjwA
Revision Through MCQs (RTM) Compilation (November 2019)

 Coalbed Methane (CBM), an unconventional source of natural gas is


now considered as an alternative source for augmenting India’s energy
resource.
 India has the fifth largest proven coal reserves in the world and thus
holds significant prospects for exploration and exploitation of CBM.
 The Gondwana sediments of eastern India host the bulk of India’s coal
reserves and all the current CBM producing blocks. The vast majority of
the best prospective areas for CBM development are in eastern India,
situated in Damodar Koel valley and Son valley
Refer: https://www.insightsonindia.com/2019/11/26/coalbed-methane-cbm/

216. Consider the following statements


1. largest consumers of coal in India are electricity generation
2. Gondwana coal is of low calorific value and high ash content
3. India has the fourth-largest coal reserves in the world
Which of the above given statements is/are correct?
(a) 1 only
(b) 2 and 3
(c) 1 and 2
(d) 1, 2 and 3
Ans: (c)
Explanation:
 India has the fifth-largest coal reserves in the world, and CBM has been
looked at as a clean alternative fuel with significant prospects.
 A large part of Indian coal reserve is similar to Gondwana coal. It is of
low calorific value and high ash content. The carbon content is low in
India's coal, and toxic trace element concentrations are negligible. The
natural fuel value of Indian coal is poor. On average, the Indian power
plants using India's coal supply consume about 0.7 kg of coal to generate
a kWh, whereas United States thermal power plants consume about 0.45
kg of coal per kWh
 The largest consumers of coal in India are electricity generation
(576.19 MT), the steel and washery industries (58.50 MT), the sponge
iron industry (8.51 MT) and the cement industry (7.70 MT). Consumption
of lignite stood at 45.82 MT in 2017–18. Electricity generation alone
accounts for 83.7% of the total lignite consumption
Refer: https://www.insightsonindia.com/2019/11/26/coalbed-methane-cbm/

217. What is common to the clans known as Swarga-Aroi, Basumati-Aroi, Ramsa-


Aroi?
(a) Monpa
(b) Apatani
(c) Munda
(d) None of the above
Ans: (d)
Explanation: The important clans of Bodos are:

Telegram: https://t.me/insightsIAStips
161
Youtube: https://www.youtube.com/channel/UCpoccbCX9GEIwaiIe4HLjwA
Revision Through MCQs (RTM) Compilation (November 2019)

 Swarga-Aroi; In Sanskrit, Swarga means heaven. The clan is heaven folk.


The clan never worked as cultivators. They were also known as Deoris
and Ojhas.
 Basumati-Aroi; In Sanskrit, Basumati means earth. The clan is earth
folk. The clan had certain privileges over land not possessed by others.
 Ramsa-Aroi; The clan is Ramsa folk. Ramsa is a village in Betna Mouza,
Undivided Kamrup. Ramsa is a hill in Kharguli, Kamrup. Ram-sa is the
name by which Kacharis living in the plains were known to their brethren
in the hills. Hojai Kacharis called Bodos Rangsa or Ramsa.
Refer: https://www.insightsonindia.com/2019/11/26/what-is-the-bodoland-
dispute/

218. The largest number of Hindu temples under the ASI’s protection is in
(a) Karnataka
(b) Tamil Nadu
(c) Odisha
(d) Uttara Pradesh
Ans: (a)
Explanation:


Refer: Facts for Prelims: https://www.insightsonindia.com/2019/11/26/insights-
daily-current-affairs-pib-26-november-2019/

219. “The 8.8 km long Rohtang Tunnel will cut through X. When complete, it will
become the world’s longest highway tunnel above 10,000 feet. The tunnel
provides a temporary winter link to the outside world not only to residents of
Lahaul and Spiti and the tunnel will be one of the longest road tunnels in India
and is expected to reduce the distance between Manali and Keylong by about 46
km.”
With reference to above passage, Identify the X from the following given options:
(a) Karakoram Range
(b) Pir Panjal Range

Telegram: https://t.me/insightsIAStips
162
Youtube: https://www.youtube.com/channel/UCpoccbCX9GEIwaiIe4HLjwA
Revision Through MCQs (RTM) Compilation (November 2019)

(c) Aravali Range


(d) None of the above
Ans: (b)
Explanation:
 Rohtang Tunnel is a highway tunnel being built under the Rohtang Pass
in the eastern Pir Panjal range of the Himalayas on the Leh-Manali
Highway.

Refer: Facts for Prelims: https://www.insightsonindia.com/2019/11/26/insights-


daily-current-affairs-pib-26-november-2019/

220. Consider the following statements.


1. Rajasthan is the first state to have India’s first wildlife conservation reserve
dedicated exclusively to the blackbuck
2. Blackbucks are native to the Indian subcontinent that has been classified as
endangered (IUCN) in 2017.
Which of the above statements is/are correct?
(a) 1 only
(b) 2 only
(c) Both 1 and 2
(d) Neither 1 Nor 2

Telegram: https://t.me/insightsIAStips
163
Youtube: https://www.youtube.com/channel/UCpoccbCX9GEIwaiIe4HLjwA
Revision Through MCQs (RTM) Compilation (November 2019)

Ans: (d)
Explanation:
 India’s first wildlife conservation reserve dedicated exclusively to the
blackbuck has been approved in 2017 by the state government of Uttar
Pradesh under Wildlife Protection Act, 1972.
 It is the “first ever conservation reserve” of any kind in U.P.
 They are native to the Indian subcontinent that has been classified as
Least Concerned in 2017 (earlier status was near threatened by IUCN
since 2003). They are now extinct in Bangladesh and Pakistan.

RTM- REVISION THROUGH MCQS - 27th -Nov-2019

221. With reference to National Remote Sensing Centre (NRSC), consider the
following statements
1. NRSC has a data reception station at Master Hassan
2. NRSC ground station acquire the earth observation data from Indian as well
as foreign remote sensing satellite
3. It monitor and analyze the Cartosat series satellite transferred data
Which of the above given statements is/are correct?
(a) 1 and 3
(b) 2 and 3
(c) 3 only
(d) 1 only
Ans: (b)
Explanation:
 National Remote Sensing Centre (NRSC) at Hyderabad is responsible for
remote sensing satellite data acquisition and processing, data
dissemination, aerial remote sensing and decision support for disaster
management.
 NRSC has a data reception station at Shadnagar near Hyderabad for
acquiring data from Indian remote sensing satellites as well as
others.
 NRSC Ground station at Shadnagar acquires Earth Observation data
from Indian remote-sensing satellites as well as from different foreign
satellites.
 NRSC is also engaged in executing remote sensing application projects
in collaboration with the users.
 ISRO has established the Antarctica Ground Station for Earth
Observation Satellites (AGEOS), at Bharati Station, Larsemann Hills,
Antarctica, for receiving Indian Remote sensing Satellite (IRS) data. This
state-of-the-art advanced Ground station was commissioned during
August 2013 and is receiving data from IRS satellites (like CARTOSAT-2
Series, SCATSAT-1, RESOURCESAT-2/2A, CARTOSAT-1) and
transferring the same to NRSC, Shadnagar near Hyderabad.
Refer: https://www.isro.gov.in/about-isro/national-remote-sensing-centre-nrsc

Telegram: https://t.me/insightsIAStips
164
Youtube: https://www.youtube.com/channel/UCpoccbCX9GEIwaiIe4HLjwA
Revision Through MCQs (RTM) Compilation (November 2019)

222. Consider the following statements about National Remote Sensing Centre
(NRSC)
1. It is a part of Indian Space Research Organization (ISRO)
2. It is the focal point for distribution of remote sensing satellite data products
in India and its neighboring countries
Select the correct answer using the code below:
(a) 1 Only
(b) 2 Only
(c) Both 1 and 2
(d) Neither 1 nor 2
Ans: (c)
Explanation:
 National Remote Sensing Centre (NRSC), is a full-fledged centers of ISRO.
 NRSC was functioning as an autonomous body called National Remote
Sensing Agency (NRSA) under Department of Space (DOS) till August,
2008. The Centre is responsible for remote sensing satellite data
acquisition and processing, data dissemination, aerial remote sensing
and decision support for disaster management.
 NRSA was transferred from DST to new created Department of Space in
1980 as an autonomous centre. Indian Photo-interpretation Institute was
renamed as Indian Institute of Remote Sensing in 1983. On 1st
November 1995, the UN affiliated Centre for Space Science and
Technology Education in Asia and the Pacific (CSSTE-AP) was created
and hosted at Indian
 Institute of Remote Sensing (IIRS), Dehradun. On September 1, 2008
NRSA was converted from an autonomous organisation to a fully
Government organisation under ISRO and renamed National Remote
Sensing Centre (NRSC).
 National Remote Sensing Centre (NRSC) is the focal point for distribution
of remote sensing satellite data products in India and its neighboring
countries. NRSC has an earth station at Shadnagar about 55Km from
Hyderabad, to receive data.
Refer: https://www.isro.gov.in/about-isro/national-remote-sensing-centre-nrsc

223. In June-2019 IMD reported that Churu (Rajastan) recorded the highest
temperature of 48.4°C due to increasing heat wave in the north-India. In the
context of “Heat wave” consider the following statements
1. Heat waves form when a warm, high pressure system stalls in a region
2. Heat is able to persist if there aren't rain and clouds to cool things off
3. Heat-trapping ability of cities is called urban heat island effect
4. According to IPCC 4th Assessment Report, Global warming is causing more
heat waves
Select the correct answer using the code given below.
(a) 1 and 3
(b) 1, 2 and 3
(c) 2 and 3
(d) All of the above
Ans: (d)

Telegram: https://t.me/insightsIAStips
165
Youtube: https://www.youtube.com/channel/UCpoccbCX9GEIwaiIe4HLjwA
Revision Through MCQs (RTM) Compilation (November 2019)

Explanation:
 Heat waves form when a warm, high pressure system stalls in a region.
 Variations in the location of the jet stream (flow of air that steers the
movements of high and low pressure areas through the mid-latitudes)
can bring the unusually warm air into a region, causing a heat wave.
 Heat is able to persist if there aren't rain and clouds to cool things off.
 heat-trapping ability of cities is called urban heat island effect
 Global warming is causing more heat waves. According to IPCC 4th
Assessment Report
Refer: http://www.imd.gov.in/pages/heatwave.php

224. According to IMD, the criteria for consideration of “Heat Wave” is


1. When normal maximum temperature of a station is less than or equal to
400C
2. When normal maximum temperature of a station is more than 400C
3. When actual maximum temperature remains 450C or more irrespective of
normal maximum temperature, heat wave should be declared
Select the correct answer using the code given below.
(a) 1 and 3
(b) 1 and 2
(c) 2 and 3
(d) All of the above
Ans: (d)
Explanation:
 IMD classification (PageNo-10)
Refer: http://imd.gov.in/section/nhac/termglossary.pdf

225. Recently pre historic rock paintings are found in the X village of Tamil Nadu,
which resemble the script found in Indus civilization sites. Here X refers to
(a) Aluva
(b) Chengannur
(c) Kunnathunad
(d) Karikiyoor
Ans: (d)
Explanation:
 Members of the Irula tribe, one of India’s oldest indigenous groups, are
furious at the recent destruction of their ancient rock art at Karikiyoor
in Kil Kotagiri village, Tamilnadu.
 Apathetic trekkers have defaced the paintings—an ancestral site for the
tribals—with religious and political messages, etched out in chalk and
whitener pens over 5,000-year-old imagery.
Refer: https://www.thehindu.com/news/national/tamil-nadu/nilgiri-tribals-tense-
as-trekkers-trash-priceless-rock-art/article27407136.ece

226. Oussudu Lake sometimes seen in the news recently, located in


(a) Andhra Pradesh
(b) Kerala

Telegram: https://t.me/insightsIAStips
166
Youtube: https://www.youtube.com/channel/UCpoccbCX9GEIwaiIe4HLjwA
Revision Through MCQs (RTM) Compilation (November 2019)

(c) Karnataka
(d) Puducherry
Ans: (d)
Explanation:
 An important bird area, located between the Pulicat and Point Calimere
wetlands, the about 800-acre Oussudu lake spreads equally in
Puducherry and Tamil Nadu territories. About 390 acres on the
Puducherry were declared as a bird sanctuary.
 Designated as one of the important wetlands of Asia by the International
Union for Conservation of Nature (IUCN), the wetland will have protection
from both sides
 It is an artificial (man-made) lake, it has been a haven for a variety of
migratory and inland birds. The areas in and around the lake are abused
by several industries producing glass, rubber, coir, cosmetic industry, a
brewery, dairy and plastic-molding factory. Poaching also has been
rampant
Refer: https://www.thehindu.com/news/cities/puducherry/artificial-islands-
coming-up/article28494107.ece

227. Talley Valley Wild life sanctuary located in


(a) Assam
(b) Arunachal Pradesh
(c) Odisha
(d) Sikkim
Ans: (b)
Explanation:
 Talley Valley Wildlife Sanctuary is a wildlife sanctuary in Arunachal
Pradesh, India.
 Talley is a plateau with dense forest of silver fir trees, pine clad plateau of
beautiful grandeur, and a vast wasteland. The area has some of the most
important endangered species including the clouded leopard.
Refer: https://www.thehindu.com/sci-tech/energy-and-environment/new-snake-
species-found-in-arunachal/article30099823.ece

228. “E-2020 initiative” has been in news for sometimes was launched by
(a) WHO
(b) UNFCCC
(c) UNDP
(d) WTO
Ans: (a)
Explanation:
 Four countries from Asia — China, Iran, Malaysia and Timor-Leste —
and one from Central America — El Salvador — reported no indigenous
cases of malaria in 2018, according to the World Health Organzation
(WHO).

Telegram: https://t.me/insightsIAStips
167
Youtube: https://www.youtube.com/channel/UCpoccbCX9GEIwaiIe4HLjwA
Revision Through MCQs (RTM) Compilation (November 2019)

 The countries were part of the global health body’s E-2020 initiative,
launched in 2016, working in 21 countries, spanning five regions, to
scale up efforts to achieve malaria elimination by 2020.
Refer: https://www.insightsonindia.com/2019/06/22/e-2020-initiative/

229. Which of the following is the first Arab country to get FATF membership?
(a) Qatar
(b) Kuwait
(c) Iran
(d) Saudi Arabia
Ans: (d)
Explanation:
 Saudi Arabia has become the first Arab country to be granted full
membership of the Financial Action Task Force (FATF) following the
group’s annual general meeting in the US.
 The kingdom’s accession came as the global money laundering watchdog
celebrated the 30th anniversary of its first meeting held in Paris in 1989.
 Saudi Arabia had received an invitation from the FATF at the beginning
of 2015 to join as an “observer member“.
Refer: https://www.insightsonindia.com/2019/06/24/saudi-arabia-becomes-1st-
arab-country-to-get-fatf-membership/

230. Centrally-controlled Air Traffic Flow Management (C-AFTM) system has been in
news for sometimes is primarily related to
(a) Air pollution control management
(b) Airspace and airport management
(c) Weather control and management
(d) None of the above
Ans: (b)
Explanation:
 Airports Authority of India (AAI) is working towards the
implementation of a Centrally-controlled Air Traffic Flow Management
(C-AFTM) system across country’s airspace and major airports,
particularly those with high traffic density, in its bid to cut travel time for
fliers.
 The new system would integrate data from airlines, airports and air-
traffic agencies and inform airlines about the time planes will be
parked at the destination airport well before departure.
 The system is primarily meant to address the balancing of capacity
against the demand to achieve optimum utilization of the major
resources such as airport, airspace and aircraft at every Indian airport
where there is a capacity constraint.
Refer: facts for Prelims: https://www.insightsonindia.com/2019/06/24/insights-
daily-current-affairs-pib-24-june-2019/

Telegram: https://t.me/insightsIAStips
168
Youtube: https://www.youtube.com/channel/UCpoccbCX9GEIwaiIe4HLjwA
Revision Through MCQs (RTM) Compilation (November 2019)

RTM- REVISION THROUGH MCQS - 28th -Nov-2019

231. Consider the following statements


1. Question Hour is the first hour of a sitting session of India's Lok Sabha
2. Star question is a one to which member desires an oral answer from minister
3. Un starred question is one to which written answer is desired by the member
from minster
Which of the above given statements is/are correct?
(a) 1 only
(b) 2 and 3 only
(c) 1 and 2 only
(d) 1, 2 and 3
Ans: (d)
Explanation: The number of starred questions was fixed at 20 per Question
Hour from the fourth session of the fifth Lok Sabha in 1972.
 Starred Questions: A Starred Question is one to which a member desires
an oral answer from the Minister in the House and is required to be
distinguished by him/her with an asterisk. Answer to such a question
may be followed by supplementary questions by members.
 Unstarred Questions: An Unstarred Question is one to which written
answer is desired by the member and is deemed to be laid on the Table of
the House by Minister. Thus it is not called for oral answer in the House
and no supplementary question can be asked thereon.
Refer: https://www.insightsonindia.com/2019/11/28/what-are-starred-
questions/

232. Common But Differentiated Responsibilities and Respective Capability (CBDR-


RC) and the annex classifications were codified under
(a) Montreal Protocol
(b) Cancun Summit
(c) Kyoto Protocol
(d) Parris Summit
Ans: (c)
Explanation:
 Common but Differentiated Responsibilities and Respective
Capabilities (CBDR–RC) is a principle within the United Nations
Framework Convention on Climate Change (UNFCCC).
 It acknowledges the different capabilities and differing responsibilities of
individual countries in addressing climate change.
 Reflecting CBDR-RC, the Convention divided countries into “Annex I”
and “non-Annex I,” the former generally referring to developed countries
and the latter to developing countries.
 Under the Convention Annex I countries have a greater mitigation role
than non-Annex-I countries.
 CBDR-RC and the annex classifications were codified in the 1997
Kyoto Protocol, and Annex I country emissions reductions were legally
bound.

Telegram: https://t.me/insightsIAStips
169
Youtube: https://www.youtube.com/channel/UCpoccbCX9GEIwaiIe4HLjwA
Revision Through MCQs (RTM) Compilation (November 2019)

Refer: https://www.insightsonindia.com/2019/11/28/common-but-differentiated-
responsibilities-and-respective-capabilities-cbdr-rc/

233. With reference to Contract for Web, Consider the following statements
1. Contract for web is an initiative of International Telecommunication Union
under UN leadership
2. Contract is binding on all the agreed nations
3. It is used for to create standard policy on web for the benefits of the world
Which of the above given statements is/are correct?
(a) 1 only
(b) 3 only
(c) 1 and 3
(d) 2 and 3
Ans: (b)
Explanation: Contract for the Web
 The idea is to create a global plan of action for all stakeholders to
together commit to building a “better” Web. The goal is to create a
standard policy for a Web that benefits all.
o The Contract consists of nine principles — three each for
governments, private companies, and individuals and civil society
to endorse.
o It has been created by representatives from over 80 organisations,
including governments, companies, civil society activists, and
academics.
 The Contract is not meant to be “simply aspirational”, or just a
“declaration”. “It’s actually meant to be implemented, and it’s meant to be
a plan of action.
 ‘Contract for the Web’ is not a legal document, or a United Nations
document — though the organisation is in talks with the UN. It cannot
currently bend governments or companies — even those that are on
board — to its will.
Refer: https://www.insightsonindia.com/2019/11/28/contract-for-the-web/

234. The annual flagship Emissions Gap Report released by


(a) German Watch
(b) UNEP
(c) IPCC
(d) UNFCCC
Ans: (b)
Explanation: The annual United Nations Environment Programme (UNEP)
flagship Emissions Gap Report has been released.
 It is also called as the “Commitment Gap”, it is the difference between the
low levels of emissions that the world needs to drop to, compared with
the projected level of emissions based on countries’ current commitments
to decarbonization.
 It measures the gap between what we need to do and what we are
actually doing to tackle climate change.
Refer: https://www.insightsonindia.com/2019/11/28/emission-gap-report/

Telegram: https://t.me/insightsIAStips
170
Youtube: https://www.youtube.com/channel/UCpoccbCX9GEIwaiIe4HLjwA
Revision Through MCQs (RTM) Compilation (November 2019)

235. With reference to SATAT initiative, consider the following statements


1. It was launched with the main objective of addressing environmental
problems arising from stubble burning
2. It falls under the ages of MoEFCC
3. Under the initiative natural gas plants are proposed to be set up mainly
through independent entrepreneurs
Which of the above given statements is/are not correct?
(a) 1 and 2
(b) 2 only
(c) 3 only
(d) All of the above
Ans: (d)
Explanation: Here Directive word is not Correct!!
Sustainable Alternative towards Affordable Transportation (SATAT)
 The initiative is aimed at providing a Sustainable Alternative towards
Affordable Transportation (SATAT) as a developmental effort that would
benefit both vehicle-users as well as farmers and entrepreneurs.
 Compressed Bio-Gas plants are proposed to be set up mainly through
independent entrepreneurs.
 Falls under Ministry of Petroleum & Natural Gas
Refer: https://www.insightsonindia.com/2019/11/28/sustainable-alternative-
towards-affordable-transportation-satat-initiative-3/

236. Consider the following statements


1. Bio-gas is produced naturally through a process of aerobic decomposition
from waste
2. Natural gas is a naturally occurring hydrocarbon gas mixture consisting
primarily of methane
3. Compressed Bio-Gas is exactly similar to the commercially available natural
gas in its composition and energy potential
Which of the above given statements is/are correct?
(a) 1 and 3
(b) 2 and 3
(c) 1 and 2
(d) All of the above
Ans: (b)
Explanation:
 Bio-gas is produced naturally through a process of anaerobic
decomposition from waste / bio-mass sources like agriculture residue,
cattle dung, sugarcane press mud, municipal solid waste, sewage
treatment plant waste, etc. After purification, it is compressed and called
CBG, which has pure methane content of over 95%.
 Natural gas is a naturally occurring hydrocarbon gas mixture consisting
primarily of methane, but commonly including varying amounts of other
higher alkanes, and sometimes a small percentage of carbon dioxide,
nitrogen, hydrogen sulfide, or helium
 Compressed Bio-Gas is exactly similar to the commercially available
natural gas in its composition and energy potential. With calorific value

Telegram: https://t.me/insightsIAStips
171
Youtube: https://www.youtube.com/channel/UCpoccbCX9GEIwaiIe4HLjwA
Revision Through MCQs (RTM) Compilation (November 2019)

(~52,000 KJ/kg) and other properties similar to CNG, Compressed Bio-


Gas can be used as an alternative, renewable automotive fuel.
Refer: https://www.insightsonindia.com/2019/11/28/sustainable-alternative-
towards-affordable-transportation-satat-initiative-3/

237. Which of the following statements is/are correct


(a) HIM VIJAY Military Exercise conducted to validate operational capabilities of
our combat formations
(b) HIM VIJAY Military Exercise was an all Arms exercise
(c) Both A and B
(d) None of the above
Ans: (c)
Explanation:
 The Indian Army Conducts routine military exercises every year.
Exercise HIM VIJAY was one such endeavour.
 It was conducted to validate operational capabilities of our combat
formations. Training exercises are conducted from within available
resources and funds.
 The exercise was an all Arms exercise and approximately a Division
strength participated.
Refer: facts for prelims: https://www.insightsonindia.com/2019/11/28/insights-
daily-current-affairs-pib-28-november-2019/

238. Which of the following is/are the discretionary powers of Governor?


1. Can appoint anybody as chief minister if there is no political party with a
clear-cut majority in the assembly.
2. Can refuse to sign to an ordinary bill passed by the state legislature
3. Can reserve a bill passed by the state legislature for president’s assent
4. Can seek information from the chief minister with regard to the
administrative and legislative matters of the state
Select the correct answer using the code below:
(a) 1, 3 and 4
(b) 1, 2 and 3
(c) 2 and 4
(d) All of the above
Ans: (d)
Explanation: Some discretionary powers are as follows:
 Can dissolve the legislative assembly if the chief minister advices him to
do following a vote of no confidence. Following which, it is up to the
Governor what he/ she would like to do.
 Can recommend the president about the failure of the constitutional
machinery in the state.
 Can reserve a bill passed by the state legislature for president’s assent.
 Can appoint anybody as chief minister If there is no political party with a
clear-cut majority in the assembly.

Telegram: https://t.me/insightsIAStips
172
Youtube: https://www.youtube.com/channel/UCpoccbCX9GEIwaiIe4HLjwA
Revision Through MCQs (RTM) Compilation (November 2019)

 Determines the amount payable by the Government of Assam,


Meghalaya, Tripura and Mizoram to an autonomous Tribal District
Council as royalty accruing from licenses for mineral exploration.
 Can seek information from the chief minister with regard to the
administrative and legislative matters of the state.
 Can refuse to sign to an ordinary bill passed by the state legislature.
Refer: https://www.insightsonindia.com/2019/09/02/president-appoints-
governors-2/

239. Automatic Exchange of Information (AEOI) Regime is an international deal


signed between
(a) India and Japan
(b) India and USA
(c) India and Russia
(d) None of the above
Ans: (d)
Explanation:
 Automatic Exchange of Information (AEOI) Regime between
Switzerland and India kicked off from September 1, 2019.
 Under this mechanism, India will start receiving information on all
financial accounts held by Indian residents in Switzerland, for the year
2018.
Refer: https://www.insightsonindia.com/2019/09/02/automatic-exchange-of-
information-aeoi/

240. India’s longest electrified rail tunnel was recently inaugurated in


(a) Ladakh
(b) Assam
(c) Andhra Pradesh
(d) Karnataka
Ans: (c)
Explanation:
 India’s longest electrified rail tunnel was recently inaugurated in Andhra
Pradesh.
 The 6.7 km long tunnel was constructed at a cost of Rs 437 crore and
connects Cherlopalli and Rapuru.
Refer: Facts for Prelims: https://www.insightsonindia.com/2019/09/02/insights-
daily-current-affairs-pib-02-september-2019/

RTM- REVISION THROUGH MCQS- 29th -Nov-2019

241. Recently a bill has been introduced in Lok Sabha proposes setting up of a Re-
skilling fund, which is primarily related to
(a) For training of retrenched employees
(b) For training of central government employees
(c) For training of unorganized sector workers
(d) For training of state government employees

Telegram: https://t.me/insightsIAStips
173
Youtube: https://www.youtube.com/channel/UCpoccbCX9GEIwaiIe4HLjwA
Revision Through MCQs (RTM) Compilation (November 2019)

Ans: (a)
Explanation:
 Labour Code on Industrial Relations, 2019 bill has been introduced in
the Lok Sabha.
 The bill aims to streamline industrial relations and help India improve on
the ease of doing business index
 Proposes setting up of a “re-skilling fund” for training of retrenched
employees. The retrenched employee would be paid 15 days’ wages from
the fund within 45 days of retrenchment.
Refer: https://www.insightsonindia.com/2019/11/29/industrial-relations-code-
bill-2019/

242. Which of the following organization has launched Accelerator Lab in India, to
work on pollution, water issues?
(a) UNICEF
(b) UNESCO
(c) UNFCCC
(d) UNDP
Ans: (d)
Explanation:
 The Accelerator Lab is an innovative new initiative by the UNDP,
Germany and Qatar, to find 21st century solutions to today”s complex
new challenges.
Refer: https://www.insightsonindia.com/2019/11/29/undp-accelerator-labs/

243. Consider the following statements


1. Supplementary grants are the additional grant required to meet the required
expenditure of the government
2. Excess Grant is granted for an exceptional purpose which forms no part of
the current service of any financial year
3. Token Grant is granted when money has been spent on any service during a
financial year in excess of the amount granted for that year
Which of the above given statements is/are correct?
(a) 1 only
(b) 1 and 2 only
(c) 1, 2 and 3
(d) 2 and 3 only
Ans: (a)
Explanation:
 Supplementary grants are the additional grant required to meet the
required expenditure of the government.
 Additional Grant: It is granted when a need has arisen during the
current financial year for supplementary or additional expenditure upon
some new service not contemplated in the Budget for that year.
 Excess Grant: It is granted when money has been spent on any service
during a financial year in excess of the amount granted for that year. The

Telegram: https://t.me/insightsIAStips
174
Youtube: https://www.youtube.com/channel/UCpoccbCX9GEIwaiIe4HLjwA
Revision Through MCQs (RTM) Compilation (November 2019)

demands for excess grants are made after the expenditure has actually
been incurred and after the financial year to which it relates, has expired.
 Exceptional Grants: It is granted for an exceptional purpose which
forms no part of the current service of any financial year
 Token Grant: It is granted when funds to meet proposed expenditure on
a new service can be made available by re-appropriation, a demand for
the grant of a token sum may be submitted to the vote of the House and,
if the House assents to the demand, funds may be so made available.
Refer: https://www.insightsonindia.com/2019/11/29/supplementary-grants/

244. With reference to FASTags, Consider the following statements


1. All lanes, except one on each side, at all NHAI toll plazas were to be declared
as dedicated FASTag lanes
2. After its commencement, any motorist entering the dedicated FASTag lane
without the tag would be charged double the toll fee
3. One of the lanes may be kept as hybrid lane accepting FASTag and other
modes of payment even after its commencement
Which of the above given statements is/are not correct?
(a) 3 only
(b) 2 only
(c) 1 only
(d) None of the above
Ans: (d)
Explanation: Here Directive word is Not Corrcet!!
 FASTag is a prepaid tag, affixed on vehicle's windscreen that enables
automatic deduction of toll charges. The aim is to promote digital
payments at toll plaza and decongest national highways.
 Govt said it will now charge double user fee from vehicles entering
FASTag lane without FASTag from Dec 15, instead of Dec 1
 However, one lane in each direction was decided to be kept as 'hybrid
lane' which will accept FASTag and other modes of payment
Refer: https://www.insightsonindia.com/2019/11/29/fastags/

245. Which of the following is the first nation to declare climate emergency?
(a) Scotland
(b) United Kingdom
(c) EU
(d) France
Ans: (a)
Explanation:
 On 28th April 2019, the Scottish Parliament declared a climate
emergency, making Scotland the first country to do so.
 This was quickly followed by the National Assembly for Wales on the
29th April and then the Parliament of the United Kingdom for the UK
as whole in 1st May.
 United Kingdom, made up of England, Scotland, Wales and Northern
Ireland, is an island nation in northwestern Europe.

Telegram: https://t.me/insightsIAStips
175
Youtube: https://www.youtube.com/channel/UCpoccbCX9GEIwaiIe4HLjwA
Revision Through MCQs (RTM) Compilation (November 2019)

Refer: https://www.insightsonindia.com/2019/11/29/eu-declares-climate-
emergency/

246. Recently 6th Asian Dendrochronology Conference being held at the Birbal Sahni
Institute of Palaeosciences in Lucknow. This is the first time that the conference
is being held in India. In this context what do you mean by Dendrochronology?
(a) Dendrochronology is the study of tree rings
(b) Dendrochronology is the study of tree branch
(c) Dendrochronology is the study of tree stem
(d) Dendrochronology is the study of tree roots
Ans: (a)
Explanation:
 Dendrochronology is the study of tree rings that hold a wealth of
information about not only a tree’s past but also that of the ecosystem in
which it lives.
Refer: Facts for Prelims:https://www.insightsonindia.com/2019/11/29/insights-
daily-current-affairs-pib-29-november-2019/

247. ‘YuWaah’- Generation Unlimited initiative is launched by


(a) Ministry of Skill Development and Entrepreneurship
(b) Ministry of Youth
(c) NITI
(d) None of the above
Ans: (d)
Explanation:
 YuWaah’- Generation Unlimited initiative: Launched by UNICEF.
 It brings young people together with the private sector, governments,
international and local organisations
Refer: Facts for Prelims:https://www.insightsonindia.com/2019/11/29/insights-
daily-current-affairs-pib-29-november-2019/

248. Consider the following statements regarding cold wave in India


1. A cold wave is a rapid fall in temperature within a 24 hour period
2. The core of a cold wave at the surface is a strong high-pressure center that
forms during winter in high latitudes.
3. In India, Core cold wave zone covers Kerala, Tamil Nadu and Karnataka
Which of the above given statements is/are correct?
(a) 1 and 2
(b) 2 only
(c) 1 only
(d) 1, 2 and 3
Ans: (a)
Explanation:
 A cold wave is a weather phenomenon that is distinguished by a cooling
of the air.
 A cold wave is a rapid fall in temperature within a 24 hour period.
 The precise criterion for a cold wave is determined by the rate at which
the temperature falls, and the minimum to which it falls.

Telegram: https://t.me/insightsIAStips
176
Youtube: https://www.youtube.com/channel/UCpoccbCX9GEIwaiIe4HLjwA
Revision Through MCQs (RTM) Compilation (November 2019)

 This minimum temperature is dependent on the geographical region and


time of year. Usually cold waves are measured by the difference from the
normal temperature.
 Cold waves generally are capable of occurring at any geological location
and are formed by large cool air masses that accumulate over certain
regions, caused by movements of air streams.
 Cold waves affect much larger areas than blizzards, ice storms, and other
winter hazards.
 The “wave” in cold wave is apparent in the upper-air flow (the jet stream),
which is usually amplified into a strong ridge-trough pattern during a
major cold outbreak.
 most significant issues associated with cold wave:
o Ice and frost accretion on the critical external surfaces of an
aircraft.
o Extremely cold temperatures can stress the metal aircraft
surfaces.
o Vehicle batteries are strained and diesel fuel gels in extreme cold
weather which could have a negative impact on ground operations,
etc.
 In India, The core cold wave zone covers Punjab, Himachal Pradesh,
Uttarakhand, Delhi, Haryana, Rajasthan, Uttar Pradesh, Gujarat,
Madhya Pradesh, Chhattisgarh, Bihar, Jharkhand, West Bengal, Odisha
and Telangana, and meteorological subdivisions of Jammu, Kashmir and
Ladakh, Marathawada, Vidharbha, Saurashtra (Gujarat) and central
Maharashtra.

249. 42nd constitutional amendment, which was passed during emergency, has
moved which of the following subjects from state to concurrent list?
1. Population control
2. Forest
3. Education
4. Administration of justice
Select the correct answer using the code below:
(a) 1, 3 and 4
(b) 1 and 3 only
(c) 1, 2 and 4
(d) All of the above
Ans: (d)
Explanation:
 42nd amendment, 1976: It was called as Mini constitution of India and
was brought during national emergency.
 It transferred five subjects to Concurrent List from State List, that is,
(a) education, (b) forests, (c) weights and measures, (d) protection of wild
animals and birds, and (e) administration of justice; constitution and
organisation of all courts except the Supreme Court and the high courts.
 Also inserted words Socialist, Secular and Integrity in the preamble.

Telegram: https://t.me/insightsIAStips
177
Youtube: https://www.youtube.com/channel/UCpoccbCX9GEIwaiIe4HLjwA
Revision Through MCQs (RTM) Compilation (November 2019)

250. Which of the following is/are the primary requirement for a person to practice
medicine in India with foreign medical qualification?
(a) Clearing National exit exam
(b) One year internship in a hospital recognised by the Medical Council of India
(c) Recommendation letter by the local MP
(d) All of the above
Ans: (a)
Explanation:
 Students who study MBBS from abroad have to qualify MCI Screening
test in India, only after which they are granted the licence to practice
medicine in India.
 Now, these students from abroad will also be appearing with Indian
students in the proposed National Exit Exam (NEXT), after clearing which
they will be able to practice in India.
 This National Exit Test (NEXT) will serve as the MCI Screening
Test/Foreign Medical Graduate Examination (FMGE) for abroad
students to get licence to practice in India.

RTM- REVISION THROUGH MCQS - 30th -Nov-2019

251. “He was considered as Unofficial Ambassador of India; he was one of the
founding members of the Indian National Congress; he was a first Indian to be a
British MP and his major works includes Admission of educated natives into the
Indian Civil Service and The wants and means of India”. He was
(a) Lala Lajpat Rai
(b) Gopal Krishna Gokhale
(c) Udham Singh
(d) Dadabhai Naoroji
Ans: (d)
Explanation:
 Dadabhai Naoroji Dordi (4 September 1825 – 30 June 1917) also known
as the "Grand Old Man of India" and "Unofficial Ambassador of India"
was a British Parsi scholar, trader and politician who was a Liberal Party
member of Parliament (MP) in the United Kingdom House of Commons
between 1892 and 1895, and the first Indian to be a British MP.
 Naoroji was one of the founding members of the Indian National
Congress.
 His book Poverty and Un-British Rule in India brought attention to the
draining of India's wealth into Britain. In it he explained his wealth drain
theory.
 He was also a member of the Second International along with Kautsky
and Plekhanov.
 The East India Association was founded by Dadabhai Naoroji in 1866,
in collaboration with Indians and retired British officials in London. It
superseded the London Indian Society and was a platform for

Telegram: https://t.me/insightsIAStips
178
Youtube: https://www.youtube.com/channel/UCpoccbCX9GEIwaiIe4HLjwA
Revision Through MCQs (RTM) Compilation (November 2019)

discussing matters and ideas about India, and to provide representation


for Indians to the Government.
 Major works
o The manners and customs of the Parsees (Bombay, 1864)
o The European and Asiatic races (London, 1866)
o Admission of educated natives into the Indian Civil Service (London,
1868)
o The wants and means of India (London, 1876)
o Condition of India (Madras, 1882)
o Poverty of India
Refer: https://www.insightsonindia.com/2019/11/30/who-was-udham-singh/

252. With reference to Ghadar Party ,Consider the following statements


1. After the outbreak of World War I, Ghadar party members returned to
Punjab to agitate for rebellion for freedom Movement
2. The founding president of Ghadar Party was Sohan Singh Bhakna and
Udham Singh was the co-founder of this party
3. Komagata Maru tragedy associated with Ghadar Party
Which of the given above statements is/are not correct?
(a) 1 and 2 only
(b) 2 only
(c) 1 and 3 only
(d) 1, 2 and 3
Ans: (b)
Explanation: Here Directive Word is Not correct!!
 Ghadar Party was an Indian revolutionary organisation primarily founded
by Indians
 The party was multi-ethnic and had Sikh, Hindu and Muslim leaders
 The founding president of Ghadar Party was Sohan Singh Bhakna and
Lala Hardayal was the co-founder of this party.
 Headquarters of the movement were set up at Yugantar Ashram in San
Francisco, United States.
 Key members included Bhai Parmanand, Sohan Singh Bhakna, Bhagwan
Singh Gyanee, Har Dayal, Tarak Nath Das, Kartar Singh Sarabha, Abdul
Hafiz Mohamed Barakatullah, Sulaman Choudhary, Aamir Choudhary,
Rashbehari Bose and Gulab Kaur.
 After the outbreak of World War I, Ghadar party members returned to
Punjab to agitate for rebellion for freedom Movement. In 1915 they
conducted revolutionary activities in central Punjab and organised
uprisings.
 The party is known for setting the foundation for future Indian
revolutionary movements and served as a stepping stone for
independence
 Udham Singh was a revolutionary belonging to the Ghadar Party best
known for his assassination in London of Michael O' Dwyer, the former
lieutenant governor of the Punjab in India, on 13 March 1940.
Refer: https://www.insightsonindia.com/2019/11/30/who-was-udham-singh/

Telegram: https://t.me/insightsIAStips
179
Youtube: https://www.youtube.com/channel/UCpoccbCX9GEIwaiIe4HLjwA
Revision Through MCQs (RTM) Compilation (November 2019)

253. Which of the following section of the Representation of the People Act, 1951
governs the registration of political parties in India?
(a) Section 26A
(b) Section 27A
(c) Section 28A
(d) Section 29A
Ans: (d)
Explanation:
 Registration of Political parties is governed by the provisions of Section
29A of the Representation of the People Act, 1951.
 To be eligible for a ‘State Political Party,’ the Election Commission has
set the following criteria:
For any political party to be eligible for recognition as a State Party in a
state, it has to satisfy any of the five conditions listed below:
o Secure at least 6% of the valid vote & win at least 2 seats in an
Assembly General Election.
o Secure at least 6% of the valid vote & win at least 1 seats in a Lok
Sabha General Election
o Win at least 3% of the seats or at least 3 seats , whichever is more,
in an Assembly General Election
o Win at least 1 out of every 25 seats from a state in a Lok Sabha
General Election
o Secure at least 8% of the total valid vote in an Assembly or a Lok
Sabha General Election.
Refer: https://www.insightsonindia.com/2019/11/30/registration-of-political-
parties-under-section-29a-of-the-rp-act-1951-3/

254. Chagos Islands, a small archipelago is located in


(a) Indian Ocean
(b) Pacific Ocean
(c) Atlantic Ocean
(d) Arctic Ocean
Ans: (a)
Explanation:
 Mauritius called the UK an “illegal colonial occupier”, after it ignored a
UN mandated deadline to return the Chagos Islands, a small
archipelago in the Indian Ocean, to Mauritius.

Telegram: https://t.me/insightsIAStips
180
Youtube: https://www.youtube.com/channel/UCpoccbCX9GEIwaiIe4HLjwA
Revision Through MCQs (RTM) Compilation (November 2019)

Refer: https://www.insightsonindia.com/2019/11/30/the-dispute-between-
britain-and-mauritius-over-chagos-islands-3/

255. Consider the following statements:


1. Global Migration Report 2020 has been released by International Organization
for Migration (IOM)
2. IOM is part of the United Nations Secretariat and it provides services and
advice concerning migration to governments and migrants, including
internally displaced persons, refugees, and migrant workers
Which of the given above statements is/are correct?
(a) 1 only
(b) 2 only
(c) Both 1 and 2
(d) Neither 1 nor 2
Ans: (a)
Explanation:
 ‘Global Migration Report 2020’ has been released by International
Organisation for Migration’s (IOM).
 The International Organization for Migration is an intergovernmental
organization that provides services and advice concerning migration to
governments and migrants, including internally displaced persons,
refugees, and migrant workers. In September 2016, IOM became a
related organization of the United Nations.
 It was initially established in 1951 as the Intergovernmental Committee
for European Migration (ICEM) to help resettle people displaced by World
War II. As of March 2019, the International Organization for Migration
has 173 member states and eight observer states.
 United Nations Department of Economic and Social Affairs (UN
DESA) is part of the United Nations Secretariat and is responsible for
the follow-up to major United Nations Summits and Conferences, as well

Telegram: https://t.me/insightsIAStips
181
Youtube: https://www.youtube.com/channel/UCpoccbCX9GEIwaiIe4HLjwA
Revision Through MCQs (RTM) Compilation (November 2019)

as services to the United Nations Economic and Social Council and the
Second and Third Committees of the United Nations General Assembly.
 UN DESA assists countries around the world in agenda-setting and
decision-making with the goal of meeting their economic, social and
environmental challenges
 'The World Population Prospects 2019: Highlights', published by the
Population Division of the UN Department of Economic and Social
Affairs, said the world's population is expected to increase by two billion
people in the next 30 years, from 7.7 billion currently to 9.7 billion in
2050.
Refer: https://www.insightsonindia.com/2019/11/30/global-migration-report-
2020/

256. With reference to North Atlantic Treaty Organization(NATO), Consider the


following statements
1. It is an intergovernmental military alliance
2. NATO constitutes a system of collective defence whereby its independent
member states agree to mutual defence in response to an attack by any
external party
3. Under the individual Membership Action Plans, NATO currently has two
candidate countries ie Bosnia and India, are in the process of joining the
alliance
Which of the given above statements is/are Incorrect?
(a) 3 only
(b) 1 and 2 only
(c) 1 and 3 only
(d) 1, 2 and 3
Ans: (a)
Explanation: Here Directive word is Incorrect!!
 New membership in the alliance has been largely from Central and
Eastern Europe, including former members of the Warsaw Pact.
 Accession to the alliance is governed with individual Membership
Action Plans, and requires approval by each current member. NATO
currently has two candidate countries that are in the process of joining
the alliance: Bosnia and Herzegovina and North Macedonia. North
Macedonia signed an accession protocol to become a NATO member
state in February 2019, which is undergoing ratification by the member
states.
 NATO is an intergovernmental military alliance.
 Established by Washington treaty.
 Treaty that was signed on 4 April 1949.
Refer: https://www.insightsonindia.com/2019/11/30/north-atlantic-treaty-
organisations-nato/

257. With reference to difference between Censure motion and No-confidence


motion, consider the following statements

Telegram: https://t.me/insightsIAStips
182
Youtube: https://www.youtube.com/channel/UCpoccbCX9GEIwaiIe4HLjwA
Revision Through MCQs (RTM) Compilation (November 2019)

1. Both censure motion and no-confidence motion can be moved in Lok Sabha
or lower house in states
2. According to procedures, a no-confidence motion can be moved only in Lok
Sabha
3. No-confidence motion moved against the entire Council of Ministers and not
individual ministers or private members, while censure motion can be moved
against individual ministers or members
Which of the given above statements is/are correct?
(a) 1 and 3
(b) 2 only
(c) 1 and 2
(d) 1, 2 and 3
Ans: (d)
Explanation:
 No-confidence motion
o In a parliamentary system a no-confidence motion is a statement
or vote that the government is no longer deemed fit to remain in
office while a censure motion carries no such threat.
o A no-confidence motion demonstrates to the head of state that the
elected Parliament no longer has confidence in the government.
o In India, the Council of Ministers is collectively responsible to Lok
Sabha and it remains in office till it enjoys confidence of majority
of the members in the Lok Sabha.
o Thus, a motion of no-confidence is moved to remove the council of
ministers and oust the government from office.
o According to procedures, a no-confidence motion can be moved
only in Lok Sabha (or state assembly as the case may be). It is
not allowed in Rajya Sabha (or state legislative council).
o It is moved against the entire Council of Ministers and not
individual ministers or private members.
o As per Rule 198 of the Rules of Procedure and Conduct of
Business in the Lok Sabha, any member of the house can move a
no-confidence motion. The written notice of the motion has to be
given to the secretary general of the House by 10 am on any day of
sitting of the House.
o It needs support of at least 50 members when introduced in Lok
Sabha. If the motion carries, the house debates and votes on the
motion. If a majority of the members of the house vote in favour of
the motion, the motion is passed and the Government is bound to
vacate the office.
 Censure Motion
o A censure literally means expression of strong disapproval or
harsh criticism. It can be a stern rebuke by a legislature, generally
opposition against the policies of Government or an individual
minister. However, it can also be passed to criticize, condemn
some act. A censure motion can be moved in Lok Sabha or in a
state assembly.

Telegram: https://t.me/insightsIAStips
183
Youtube: https://www.youtube.com/channel/UCpoccbCX9GEIwaiIe4HLjwA
Revision Through MCQs (RTM) Compilation (November 2019)

o Both censure motion and no-confidence motion can be moved


in Lok Sabha or lower house in states.
o While censure motion can be moved against individual
ministers or members, no-confidence motion is moved against
the entire council of ministers.
o There is no impact on the government when a censure motion is
passed, but the council of ministers have to resign and
government collapses when a no-confidence motion is passed.

258. Which of the following is the world’s largest cotton producer in 2019-2020
season?
(a) China
(b) India
(c) Brazil
(d) Pakistan
Ans: (b)
Explanation:
 India will surpass China to become the world’s largest cotton
producer in 2019-20 season, according to latest estimates from the US
Department of Agriculture (USDA).
 India’s cotton production during the season is forecast to increase by 3-
million-bales to 28.5 million bales, while China is likely to produce 27.75
million bales. One bale is equivalent to 170 kilograms.
 “For 2019-20, India’s cotton area is forecast to rise slightly to 12.35
million hectares (30.5 million acres), as price expectations remain
favourable for cotton. The national yield is also projected to rebound from
last season’s below-average level, supporting the 3-million-bale increase
in cotton production to 28.5 million bales in 2019-20,” the USDA said in
its latest ‘Cotton and Wool Outlook’ report.
259. Consider the following statements
1. Nominal GDP is GDP evaluated at current market prices
2. Real GDP is a measure of growth without according inflation
Which of the given above statements is/are correct?
(a) 1 only
(b) 2 only
(c) Both 1 and 2
(d) Neither 1 nor 2
Ans: (a)
Explanation:
 Nominal gross domestic product is a measurement of economic output
that doesn't adjust for inflation. GDP measures everything produced by
all the people and companies within a country's borders.
 Nominal GDP is GDP evaluated at current market prices
 Real gross domestic product is a measurement of economic output that
accounts for the effects of inflation or deflation. It provides a more
realistic assessment of growth than nominal GDP.

Telegram: https://t.me/insightsIAStips
184
Youtube: https://www.youtube.com/channel/UCpoccbCX9GEIwaiIe4HLjwA
Revision Through MCQs (RTM) Compilation (November 2019)

 Nominal GDP includes both prices and growth, while real GDP is pure
growth
260. “This island is in the Sundarban Delta complex of the Bay of Bengal has been
facing the devastating effects of climate change. With rising sea levels, high tides
and floods have been swallowing portions of the island. In the last two decades,
the island's size has reduced by several times its original land mass”
The above given passage refers to which of the following island?
(a) Ghoramara island
(b) Bhasan char island
(c) Baratang Island
(d) Chatham Island
Ans: (a)
Explanation:
 Once sprawling across 8.51 sq km, Ghoramara, part of Hooghly river's
estuary, has now been reduced to 4.45 sq km. “Continual rise in sea level
due to climate change, will continue to create havoc in the estuary
causing the island to erode away,”
 According to a 2015 report in The Indian Express, the island, also
referred to as the "sinking island", "was spread over 22,000 bighas" but
now has been "reduced to approximately 5,000 bighas". The effect is such
that there is apparently a possibility that the island may be wiped off of
the map very soon.
 At one point, the island had some 40,000 people living in it. Now the
numbers have reduced to just about 3,000.

Telegram: https://t.me/insightsIAStips
185
Youtube: https://www.youtube.com/channel/UCpoccbCX9GEIwaiIe4HLjwA
Revision Through MCQs (RTM) Compilation (November 2019)

Telegram: https://t.me/insightsIAStips
186
Youtube: https://www.youtube.com/channel/UCpoccbCX9GEIwaiIe4HLjwA

S-ar putea să vă placă și